Segment 2 Of 3     Previous Hearing Segment(1)   Next Hearing Segment(3)

SPEAKERS       CONTENTS       INSERTS    Tables

 Page 252       PREV PAGE       TOP OF DOC    Segment 2 Of 3  
    Question. Is it correct that funds in the Family Housing Improvement Fund remain available until expended, and that any funds set aside for the cost of guarantees will remain unobligated forever, if there are no defaults?

    Answer. Funds in the Family Housing Improvement Fund are available until expended. It is no year money until 2001 when our test period is over. The government's cost of a loan guarantee (a subsidy estimate) is obligated from the Family Housing Improvement Fund at the time a loan guarantee commitment is made. When the lending institution issues the loan to a contractor, the amount obligated by the government is disbursed into a financing account, where it is held, collecting interest, pending any default. If no defaults occur, or no defaults are anticipated, a downward subsidy re-estimate is made and the amount held in the financing account is moved to a receipt account, where is becomes available for appropriation.

    Question. The Army is proceeding very aggressively to execute ''whole installation'' type projects. The Navy policy envisions a ''regional scope.'' What is the OSD policy with regard to privatizing all family housing in a large geographic area?

    Answer. Requirements determination should consider regional needs. OSD's policy is to encourage the Services to use the authorities in ways that best fit their needs.

    Question. Under current authorities, family housing privatization involves government contribution of land, facilities, infrastructure, mortgage guarantees, and differential lease payments to developers and financiers. Wouldn't it be prudent to gain some experience with how well this program works, before making such a large commitment to turn over so many assets for a fifty year term?
 Page 253       PREV PAGE       TOP OF DOC    Segment 2 Of 3  

    Answer. The Services and OSD are working closely and cautiously to find the best ways to implement these new privatization authorities. As we test the authorities, we find that long term, large scale deals offer some of the most powerful leveraging. We are bringing the best private sector expertise to the table to help us apply the authorities wisely as we test these deals. Balancing caution with expeditious improvement of housing for our servicemembers is the challenge we are striving to meet.

    Question. It took us many, many years to build up these housing assets. Tell us about some of the steps OSD is taking to protect its investments under this program.

    Answer. The original genesis of this program was based on two thirds of our family housing assets being in disrepair. Projects, such as Fort Carson, under this program are allowing us to renovate and replace those assets more quickly than can be done under traditional methods. Providing good housing units, along with the bad, greatly enhances our ability to accomplish this. When the contracts are put in place, DOD retains numerous incentives to ensure contractor compliance.

    Question. Provide for the record a list of all the locations that are under consideration for such projects, including the number of units in the current inventory at each location, as well as some indication of the value of the total government contribution at each location.

    Answer. The Services have nominated nearly 70 sites for housing privatization. The following 11 sites totaling 24,149 housing units, have approved concept plans and have either been procured or have solicitations being developed. Their status is listed as follows:

Table 36


 Page 254       PREV PAGE       TOP OF DOC    Segment 2 Of 3  

    The costs of these projects will be determined by scoring. Sufficient appropriations must be available to cover the amount obligated for each contract. The Department, with OMB concurrence, will determine the amount of funds to be obligated to cover future costs that are associated with the use of the tools provided in the Military Housing Privatization Initiative. These amounts are not finalized until contract award.

    Question. Describe for us the reasons privatization probably won't work at some of the places that the Services have reviewed.

    Answer. There are several reasons, but large inventory of badly dilapidated houses combined with low BAH levels makes privatization difficult financially. The cost of construction in a given geographic location also has a major impact on the financing required to make privatization work.

OVERSEAS HOUSING AUTHORITY

    Question. Is OSD interested in a legislative proposal for the establishment of an ''Overseas Housing Authority'' as a ''Non-Appropriated Fund Instrumentality'', which the Army is pursuing, and which is now under review at the Office of Management and Budget?

    Answer. Yes.

FISCAL YEAR 1999 PRESIDENT'S BUDGET REQUEST

 Page 255       PREV PAGE       TOP OF DOC    Segment 2 Of 3  
    Question. Is it correct that the only transfer of funds to date (3/12/98) into the Family Housing Improvement Fund has been the transfer of $5,900,000 from the account ''Family Housing, Navy and Marine Corps'' for the project at Everett, Washington, and that this amount is included in the 1997 column of the fiscal year 1999 program and financing statement?

    Answer. Yes.

    Question. Why does the Everett transfer appear as two separate amounts on two separate lines $3,000,000 on line 22.2001 and $2,900,000 on line 42.0001?

    Answer. The source of the Everett transfer was two different Navy Family Housing construction appropriations. The $3 million was transferred from FY 96 appropriations and the $2.9 million was transferred from FY 97 appropriations. Since the transfer occurred in FY 97 the amounts are shown on separate lines to delineate between the transfer of current year authority (the $2.9 million of FY 97 funds shown on lines 42.0001) and the transfer of prior year unobligated balances (the $3.0 million of FY 96 funds shown on line 22.2001).

    Question. Is it correct that line 22.22 of the program and financing statement shows that no further transfers are expected during fiscal years 1998 and 1999?

    Answer. Since the specific sources, timing, and amounts of transfers required for privatization projects were unknown during the formulation of the budget, no transfers are displayed in the program and financing statement. Congressional notification of our intent to transfer funds will be made during execution when specific sources, timing, and amounts for transfer have been identified.
 Page 256       PREV PAGE       TOP OF DOC    Segment 2 Of 3  

    Question. If further transfers are planned, line 22.22 notwithstanding, what are the dates and amounts of such transfers, and for which projects?

    Answer. See Answer #69 for project timelines. Budget scoring requirements are finalized during the procurement.

    Question. Is it correct that the program and financing statement indicates that the unobligated balance available to the Family Housing Improvement Fund at the end of fiscal year 1999 will be zero, that is, the Fund will be broke?

    Answer. The program and financing statement assumes all prior year unobligated balances and the $7 million requested for the administrative expenses of HRSO during FY 99 will be obligated by the end of FY 99. Transfers from the family housing construction accounts will fund any additional privatization costs. An additional appropriation request will be made for HRSO administrative expenses in FY 00.

    Question. How was the $7,000,000 budget request for fiscal year 1999 formulated, and what workload measurement justifies this request?

    Answer. As noted in Answer 43, expected obligations which justify the request are as follows:

Table 37


    Question. Is it correct that the program and financing statement for the Military Unaccompanied Housing Improvement Fund indicates that all balances in the Fund ($5,000,000) will be obligated during fiscal year 1998, that $1,000,000 will outlay during fiscal year 1998, and that another $1,000,000 will outlay during fiscal year 1999?
 Page 257       PREV PAGE       TOP OF DOC    Segment 2 Of 3  

    Answer. Yes. However, the budget projections for obligations and outlays from the Unaccompanied Housing Improvement Fund during FYs 1998 and 1999 are dependent upon stimulating private developers to build, operate and maintain barracks. The extent to which market incentives materialize or do not materialize for private development will determine the execution of these funds during the FY 1998 and FY 1999 timeframe.

    Question. Why does the President's Budget Request include a proposal to broaden last year's General Provision Section 123, to allow transfers into the Family Housing Improvement Fund from the Base Realignment and Closure Accounts?

    Answer. DOD wants to be able to use funds earmarked for military family housing at receiving locations but budgeted in the BRAC account to pursue privatization where it is cost effective. There are currently no specific locations under active development, however.

    [CLERK'S NOTE.—End of questions for the record submitted by chairman Packard.]

    [CLERK'S NOTE.—Questions for the record submitted by Congressman Hobson.]

ENERGY SAVINGS

    As you remember, Mr. Goodman, I started out working with Assistant Secretary of Defense (Economic Security) Josh Gotbaum on a bill to establish a non-government, not-for-profit institute to assist installations in upgrading energy infrastructure and expediting cost-effective project contracts. That ''Forrestal initiative'' was put on hold to see if DOD could achieve the same results without new legislation. When we talked a year ago, you testified that DOD would be willing to come back to Congress if legislation is needed.
 Page 258       PREV PAGE       TOP OF DOC    Segment 2 Of 3  

    Question. Have you been able to achieve energy savings and cost reductions, and if so, how much?

    Answer. The Department reduced energy consumption in our buildings and facilities by 15.5 percent between FY 85 and the end of FY 96, measured by British Thermal Units per square foot. We are on track with the requirements of the Energy Policy Act and Executive Order 12902 to achieve a 30 percent reduction by 2005.

    The Department has also established Energy Savings Performance Contracts (ESPCs) covering all fifty states, the District of Columbia and Puerto Rico as another method of executing energy and water conservation measures. ESPCs are indefinite delivery, indefinite quantity service contracts that enable bases to contract more easily with one or more energy services contractors. We are redoubling our efforts to use ESPCs to partner with the private sector and enter into mutually beneficial energy and water conservation projects. The Department currently has seventeen active ESPCs with over $2 billion in contract capacity. This includes the two Super Regional ESPC contracts administered by the Army Corps of Engineers, Engineering and Support Center, Huntsville, Alabama.

FEDERAL ENERGY MANAGEMENT PROGRAM

    Last year, I worked through the National Security Appropriations Subcommittee to provide $15M for Budget Activity 18550, the ''Federal Energy Management Program.'' The money survived the appropriations conference and have been available to you.

 Page 259       PREV PAGE       TOP OF DOC    Segment 2 Of 3  
    Question. What have you done with these funds? Do you need more in FY 99, and were they requested? Is there a dispute over where the work is done—on the individual base vs. centrally managed? What is the right answer and where is this work done now?

    Answer. The DOD wide Federal Energy Management Program (FEMP) funds will be used to leverage ESPC contracts and fund additional high payback initiatives. To date, we have allocated FY98 DOD-wide FEMP funds to the following categories:

Table 38


    [CLERK'S NOTE.—End of questions for the record submitted by Congressman Hobson.]
Thursday, March 5, 1998.

QUALITY OF LIFE IN THE MILITARY

WITNESSES

SMA ROBERT E. HALL, SERGEANT MAJOR OF THE ARMY

MCPON JOHN HAGAN, MASTER CHIEF PETTY OFFICER OF THE NAVY

SGTMAJ LEWIS LEE, SERGEANT MAJOR OF THE MARINE CORPS

CMSAF ERIC W. BENKEN, CHIEF MASTER SERGEANT OF THE AIR FORCE

Statement of the Chairman
 Page 260       PREV PAGE       TOP OF DOC    Segment 2 Of 3  

    Mr. PACKARD. Ladies and gentlemen, we would like to call this hearing to order.

    This hearing this morning will concentrate on the quality of life issue as it relates to mostly our enlisted men and women in the military, which, of course, is the heart and soul of our services. We are extremely grateful to have with us the chief enlisted men from each branch of the services. This morning we have with us from the Army Sergeant Major Robert Hall, the Sergeant Major of the Army; John Hagan, the Master Chief Petty Officer of the Navy; and Mr. Lewis G. Lee, who is Sergeant Major of the Marine Corps; and Mr. Eric Benken, who is the Chief Master Sergeant of the Air Force.

    Three of these brethren have been with us before, some of them several times before this subcommittee. One is new. We want to welcome Segeant Major Hall of the Army, who is new this year.

    This is a good hearing. This is one that I think most members of the subcommittee look forward to because, again, we are really hearing from those that represent the sailors and the marines and the airmen and the soldiers that are in the field, those that really are the heart and soul of our services.

    It is a different kind of testimony, and I have read their testimony, and I am sure many of the committee members have.

    We want to welcome a group of students from Los Angeles. They are here just as observers. We welcome them here, and anyone else that is here, we appreciate you all being with us.
 Page 261       PREV PAGE       TOP OF DOC    Segment 2 Of 3  

    I will simply make one or two introductory comments before we hear from the ranking member, Mr. Hefner, and then hear from each of the witnesses.

    We have expressed concern and disappointment in the level of budgeting this year for military construction. We feel that we have taken—three years in a row, we have taken a hit in terms of the level of funding. If the President's budget submission this year was implemented, we would be somewhere over a 35 percent reduction over the last 3 years, and that is simply unacceptable. We can't live with that and continue to provide the facilities that are necessary not only for a good qualify of life for our men and women in the services, but that reduces us to the point where we perhaps may be dipping into readiness and into safety and a variety of other things that are even more important than quality-of-life issues.

    Retention is a major problem, and each of you has addressed that in your testimonies. We have got probably the highest level and quality of men and women in our services than we have ever had in my lifetime, and yet that is being threatened, I think, because we may lose some of these very fine, well-trained people simply because the facilities that they have to work with and the facilities they have to live in and those auxiliary facilities that provide services to their families are inadequate and sub-standard. And that is of great concern.

    We have made a deliberate effort to try to catch up and close the gap in these facilities, but the fact is if we don't have the money, we can't do it. I know that that is of concern to each of you.

    I guess I also usually announce our next meeting, and our next hearing will be the 12th of March, next Thursday. That will be our final hearing before we start to write the bill. And so the members of the committee ought to be aware of that. It is at 9:30 in the morning.
 Page 262       PREV PAGE       TOP OF DOC    Segment 2 Of 3  

    With that, I am going to turn some time over to Mr. Hefner, who has chaired this committee in the past and is certainly well experienced on all of the issues that we deal with on military construction, and so we are delighted to have you with us, Mr. Hefner.

    Mr. HEFNER. Thank you, Mr. Chairman. And welcome, gentlemen. It is good to see you again and again and again.

    It is the same old story that we have every year and we have worked so hard over the years to try to build quality of life for our men and women in the service. It seems as the budget gets squeezed, we get squeezed just a little bit harder. I would imagine that we are the only budget in the House that over the last few years in real dollars has seen a decrease in our budget.

    The need is still there, and, of course, retention is sort of a double-edged sword. I would imagine that a very good expanding economy puts more pressure to try to get people to come into the armed forces. Of course, we may not be able to offer the incentives that we did in the past simply because of budgetary constraints. But we are happy to see you here today, and we are anxious to hear what you have to say today, and you are always welcome before this committee.

    Thank you, Mr. Chairman.

    Mr. PACKARD. Thank you very much, Mr. Hefner.

 Page 263       PREV PAGE       TOP OF DOC    Segment 2 Of 3  
    Mr. Wamp, do you have a statement you would like to make?

    Mr. WAMP. No, sir.

    Mr. PACKARD. Mr. Hobson?

    Mr. HOBSON. No, sir.

    Mr. PACKARD. Thank you.

    With that, then, we will proceed with the Army first, the Navy, and then the Marines and then the Air Force, in that order, if we could. Again, we have your testimony. Each of you has written a very thorough and a very complete and a very well organized written testimony. That will be entered into the record in its completeness. We would prefer that you not read it. Most of us have read it. We don't need to have you read it to us again, so if you will summarize and hit the points that you feel are most important for this subcommittee to be reminded of, we would appreciate it.

    We will move to you, Sergeant Major Hall. Inasmuch as you are new on the committee, we hate to have you go first, but that is the way we have got it set up, and so we will have you speak first.

STATEMENT OF SERGEANT MAJOR ROBERT E. HALL

    Sergeant Major HALL. Sir, it is my honor to go first.
 Page 264       PREV PAGE       TOP OF DOC    Segment 2 Of 3  

    First of all, Mr. Chairman and distinguished members of the subcommittee, good morning. I am honored and privileged to speak before you today to discuss quality-of-life issues, and to speak on behalf of our Army, our soldiers, and their families. Before I talk about that, I think the first thing I would like to do is thank you for the additional funds that you have given us over the past two years for quality-of-life improvements. We have used those funds to bring over 9,000 billet spaces to the DOD 1 plus 1 standard in more than 100 barracks worldwide. We do really appreciate everything you do to help us to enhance the morale of our soldiers. They are very busy. As you know, deployments haven't eased during the past year, and today soldiers serve around the world in many capacities. Over 100,000 are forward deployed, and on any given day, we average 30,000 soldiers deployed away from home station in over 70 countries. In May of last year, we hit a high watermark with soldiers deployed in 100 countries.

    I didn't know there were 100 countries in the world, sir, so I had them pull up the list, and we really did have soldiers in 100 countries in May of 1997. Those soldiers were keeping the peace in Bosnia. They were deterring Iraqi aggression in Southwest Asia. They also supported local authorities following hurricanes, floods, wildfires, ice storms, and in numerous other capacities.

    Sir, as you said, I am only in my fifth month as the Sergeant Major of the Army. I am the newest member of this senior enlisted team before you today. But that doesn't make me a rookie because I do bring over 30 years of Army experience to this job, and I have been a noncommissioned leader for 29 of those 30 years. So I do come from the soldiers in our Army.

    During the past 4 months throughout my travels, I have talked with soldiers who were preparing to deploy, who were returning from deployments, and those who were doing their job every day. I am proud to represent those young men and women. They are great soldiers. They are trained. They are motivated. They are dedicated to the country.
 Page 265       PREV PAGE       TOP OF DOC    Segment 2 Of 3  

    I think they understand that there is only so much money that the American people can afford to provide for its men and women in uniform. They do not expect to become wealthy. They just want to serve. But, above all, they want to ensure an adequate standard of living for themselves and for their families.

    We as leaders recognize that the strength of our Army lies in the quality of our soldiers. They are first-class Americans, and we absolutely need them.

    I am heartened by the fact that they have not lost faith. They have not lost faith in the Army. They have not lost faith in the leadership. And they have not lost faith in themselves to do what is right. But they do have concerns, and those concerns run the full spectrum of quality-of-life issues.

    If you would ask me to list the top four issues this morning, sir, I would tell you that the top four things that soldiers worry about are pay and entitlements, housing, medical care for themselves, but especially for their families, and a stable retirement system.

    We know we have to take care of these soldiers if we hope to recruit and retain the quality individuals that are coming in today, because they are the key to ensuring us the best Army in the world.

    We know we have to take care of the families, also, because they are tremendously important and have a tremendous influence on a soldier's decision to leave or to stay in the Army. It is really true that the Army enlists the soldier, but it re-enlists the family.
 Page 266       PREV PAGE       TOP OF DOC    Segment 2 Of 3  

    I would just like to say that we very much appreciate your help. I ask for your continued assistance to provide for our Army, our Nation, our military, and our soldiers.

    Sir, I don't have any profound statement to end this with, so just let me say again that I consider myself very lucky to represent my soldiers this morning, and I welcome any questions that the subcommittee has.

    [Prepared statement of Sergeant Major Robert E. Hall follows:]
    "The Official Committee record contains additional material here."

    Mr. PACKARD. Thank you very much, Sergeant Major Hall.

    We will have all of the testimony before we get into the question-and-answer period, so we will come back to the Army.

    Sergeant Major HALL. Yes, sir.

    Mr. PACKARD. And all of you, with questions.

    Mr. Hagan, we welcome you back to this subcommittee again. You have served for a long time, and my understanding is that this may be your last time, at least in this capacity, before the subcommittee. So we welcome you and are looking forward to your testimony. You may proceed as you wish.

 Page 267       PREV PAGE       TOP OF DOC    Segment 2 Of 3  
STATEMENT OF MASTER CHIEF PETTY OFFICER JOHN HAGAN

    Chief HAGAN. Chairman Packard and members of the committee, thank you very much for the opportunity. It is true that today represents my final opportunity to testify before this committee. I am privileged to have an additional opportunity before other committees throughout the rest of this month, and then will be leaving the Navy after 33 years and nearly 6 years in this job.

    I have submitted written testimony which is possibly too lengthy. In fact, some who chopped on my testimony within the Department of the Navy indicated that I spent too much time saying thank you and giving you proof or evidence of the good potential that has been fulfilled by previous MILCON. I realize that you probably get that information through a variety of sources. It was important to me, though, for you to know from me personally how grateful I am at how much improvement we have made in our quality-of-life infrastructure over the 33 years I have been in the Navy, but particularly over the 6 years that I have been in this job and keeping track of it.

    I express to you this morning the gratitude of all the sailors I visit with regularly for your wisdom and foresight in addressing our needs and for the continuing dramatic improvement in our infrastructure. Like Sergeant Major Hall said, the Navy is expeditionary forward deployed, and today, generally speaking, 190-plus ships are underway today and over half of those are forward deployed for 6 months or longer.

    It may seem unusual that in my written testimony my personal priorities for acceleration of programmed MILCON, if at all possible, and my priorities for insertion of unprogrammed MILCON lie in the training world, specifically at our Recruit Training Command at Great Lakes, Illinois. But within the charter I am authorized to speak to and within my area of competence, I prioritize that as not only our number one MILCON need but far ahead of whatever is number two.
 Page 268       PREV PAGE       TOP OF DOC    Segment 2 Of 3  

    As important as family housing, barracks, single-Sailor construction, single-Sailor facilities for Sailors that live aboard ships, child development centers, family service centers, as important as any of those things are, we need a quality training infrastructure where recruits enter the front door of the Navy, and we don't have it.

    Currently we operate without a live fire range because the live fire range has been condemned environmentally, and although one is programmed, I ask for your consideration for accelerating that construction by one year so that we put fewer Sailors into the fleet who have not received live fire training during their initial recruit phase.

    I would point out that at RTC Great Lakes we have basically what is known in the Navy as heads and beds. We have berthing and galley facilities and some training infrastructure, but not enough. My own personal priority is to have a series of obstacle/challenge competence courses that can be used as a true training environment. We currently have one competence course which can only be used as a diversion from training rather than a training device because its capacity allows us to put Sailors through the course a maximum of two times in their entire 9 weeks of training. During the summer surge, recruits only have one shot at that facility. Those are the sorts of needs we have at our Recruit Training Command at Great Lakes, and that is my true and genuine and passionate desire for MILCOM priority.

    I would only share with you one other concern related to MILCON, and that is my personal concern, not shared by all of my leaders within the Department of the Navy. I feel deeply that the Private Project Venture initiatives, while good and offering us options to provide more facilities more quickly, particularly in the area of family housing, but as I leave the Navy I worry that PPVs, if used exclusively, might lead to an environment which we would give away the traditional family housing structure that I cherish and think is too valuable to allow that to happen. And, again, I am grateful for what the Private Project Venture has provided. I acknowledge that the increased cost to the Sailor is offset often by the fact that we get these facilities quicker, and I am willing to live with that, gratefully, so long as PPVs don't become the exclusive way we provide these facilities. And so I have pointed that out at some length as articulately as I am able to in my written testimony.
 Page 269       PREV PAGE       TOP OF DOC    Segment 2 Of 3  

    I appreciate this opportunity, and I look forward to your questions.

    [The prepared statement of Master Chief Petty Officer John Hagan follows:]
    "The Official Committee record contains additional material here."

    Mr. PACKARD. Thank you very much, Mr. Hagan.

    We will now have Mr. Lee from the Marine Corps.

SERGEANT MAJOR LEWIS G. LEE

    Sergeant Maj. LEE. Mr. Chairman and committee members, I too am honored to be here today, and I want to report that our Corps of Marines is in very good shape overall. And on behalf of them, I want to thank you for your support, past and present, and I ask on their behalf your continued support for the future.

    Our MILCON budget this year will be considerably less than it was in 1998. However, 77 percent of the programs we have planned for FY 1999 will positively impact the quality of life of our marines and their families.

    Our emphasis will be on bachelor housing and family housing, with further emphasis on fixing what we already own. After that, comes community and family programs and, finally, infrastructure reinvestment.

 Page 270       PREV PAGE       TOP OF DOC    Segment 2 Of 3  
    Of the 97,000 bachelor spaces we have, over 10,000 are considered inadequate, and we continue to have a backlog of maintenance. However, we will eliminate by the year 2005 all inadequate spaces, and our backlog of maintenance will be corrected by the year 2004. That is attributable to your all support and the commitments that we have made.

    Again, this is the result of new BEQs that are coming on line and that continuing commitment to fix our existing problems.

    We maintain approximately 25,000 family housing units. We will in fiscal year 99 rehab some 600 existing units. But we will not be able to build any new units.

    Our programs to attack maintenance backlog will be corrected by the year 2001. Since we cannot build additional units, our ability to provide more families with housing will not improve.

    To find other ways to assist our families, we are pursuing various programs to include set-aside housing, 801-type housing, and public/private ventures. I too want you all to know that while we are aggressive in our pursuits of such programs, we remain very cautious and will ensure our assets are spent in ways that not only improve the quality of life of our families, but also meet with congressional and departmental approval.

    The BAH program seems to be a good step in the right direction. We intend to monitor it very carefully and seek the maximum support for monetary increases to our BAH in the future—that is Basic Allowance for Housing—as well as the total compensation for all Marines.
 Page 271       PREV PAGE       TOP OF DOC    Segment 2 Of 3  

    We will fund other QOL projects in FY 1999 such as a child care center, a new mess hall, and a new fire station. Additionally, the bachelor rooms continue to receive new furniture, and we are on a course for replacing furniture every seven years vice the current 13.6 year cycle, beginning in FY 2002.

    In closing, adequate facilities in which to train, to work, to live, and to recreate are crucial to morale and readiness. Your marines appreciate everything they have, and they will continue to make the most of it, and I assure you they will take care of what they have.

    Mr. Chairman, things are not perfect, and they are not near perfect. But, in closing, I want to make sure you all understand that your marines are ready, they are capable, and they remain very relevant. And I will answer any questions you have to the extent of my knowledge.

    [The prepared statement of Lewis G. Lee follows:]
    "The Official Committee record contains additional material here."

    Mr. PACKARD. Thank you very much, Sergeant Major.

    Chief Master Sergeant of the Air Force, Mr. Benken, we are pleased to hear from you again.

STATEMENT OF CHIEF MASTER SERGEANT ERIC W. BENKEN

 Page 272       PREV PAGE       TOP OF DOC    Segment 2 Of 3  
    Sergeant BENKEN. Good morning, Mr. Chairman and committee members. Having submitted a written statement for the record, I would just like to say that I certainly appreciate the opportunity to speak to the Committee today on behalf of the thousands of enlisted men and women serving around the world and to thank the Members of Congress for all of the work that you have done to take care of them and their families.

    As you alluded to before, Mr. Chairman, our services have a tough job these days balancing the needs of our people with readiness and modernization to meet mission requirements of today and of the future. For the people part of that equation, there is no doubt that family housing, dormitories, child development, and fitness centers play an extremely important part in our effort to recruit and retain high-quality people in this all-volunteer force that we have today.

    The direct connection between Quality of Life and readiness is indisputable. Taking care of our service members and their families allows those who must deploy at ever-increasing rates to do their job without distraction. It is essential, now more than ever, that we continue to support Quality-of-Life efforts for our troops.

    Again, I would like to thank the Committee for the support in the past, and I look forward to the hearing today. Thank you.

    [The prepared statement of Chief Master Sergeant Eric W. Benken follows:]
    "The Official Committee record contains additional material here."

    Mr. PACKARD. Thank you very much, Mr. Benken. I think we will start with you, again, Mr. Hefner.
 Page 273       PREV PAGE       TOP OF DOC    Segment 2 Of 3  

RETENTION

    Mr. HEFNER. Thank you, Mr. Chairman.

    I don't have any new questions. The same questions keep popping up all the time. We know that we are short of money. You mentioned the things that the soldiers marines and sailors are concerned about is basically what all Americans are concerned about, making an adequate living and health care and one thing or another.

    I was looking here and I remember back in, I guess, the Reagan years when there were some surveys done why people were going into the services. Of course, you know, on first blush, you would have thought it was strictly patriotism. But we found out through the survey that a majority of the people that were going in the service were going in because of the educational benefits. I would imagine that still holds true to a point. As I mentioned earlier, we don't have the incentives for re-upping as we did before.

    On other side of the coin how big a part does quality of life play in the fact that people are leaving the service?

    Sergeant Major HALL. Sir, if I could take that one on first. First of all, we are doing okay in retention. We think it is a concern, and the caution light is certainly flashing. But about 70 percent of our mid-career force is re-enlisting.

    The ones who get out, get out for a number of reasons: the perceived lack of benefits, such as worry about a stable retirement system. I mean, these are staff sergeants, mid-managers, who have been in the Army about 10 to 12 years who are now beginning to question: Is the military a viable career? Will I still have a retirement benefit after 20 years? That plays a tremendous role in the decision to stay. They have a right to worry. They are on the least generous of three retirement systems. There have been 17 attacks on the retirement system in the past 4 years. That is a real concern.
 Page 274       PREV PAGE       TOP OF DOC    Segment 2 Of 3  

    But even without that, I have to say that they are still staying with us. Now, I pray every night that they will keep staying with us, but somewhere down the road, we will have to deal with that. Quality of life is a whole package, and it is all we have to offer our soldiers for doing the deployments that they have to do, and those deployments are not going to ease up. In fact, they tell me that they are okay with the deployments. As long as they know that the family support activities are at the same level they are at now, they intend to stay in.

    When family support drops, the danger is that they will vote with their feet.

    Mr. HEFNER. Does anybody else have anything?

    Sergeant Maj. LEE. I will say, sir, again, I think it is because of the uniqueness of the way the Marine Corps is organized and structured and our reliance upon what we call the first-term organization. I honestly have more marines, enlisted marines that want to stay in the Marine Corps than I can keep. I honestly have that, and I have that year in and year out. But I do have the same concerns that the ones Sergeant Major Hall talked about, the staff sergeants with 8 to 10 years are willing to get out, because I have a lot invested in that person, and I have a small career force that I depend on, and I don't want to lose that person.

    Now, if you ask why they are leaving the Marine Corps at 10 years of service, they will all give you 10 separate answers. But needless to say, they are smart people, they are bright people, and they want something good for themselves and their families, and they want something good for the future. So I reiterate, too, whatever affects the quality of life of those people is a retention factor for the individual, and the future, what they are going to do when they leave the service for good, it is very important to them, i.e., the benefits associated with retirement and/or veterans' benefits.
 Page 275       PREV PAGE       TOP OF DOC    Segment 2 Of 3  

    But, beyond that, I do want to reiterate to you I do not have what you would call a retention problem. I don't have that.

    Chief HAGAN. Sir, I wouldn't disagree with either answer. In terms of quality of life, I believe we are holding our own, and your specific question, I would respond by saying it isn't very high on the list of reasons given for leaving the service after one or two terms of service. It is occasionally on there, and depending upon where you have served, the answer can vary. But the quality of life that we offer and provide in so many of our fleet concentration areas now is so superior to what it has been in the past that quality of life doesn't figure very high.

    But, honestly, the number one reason in the Navy is clearly family separation. Sailors serve their first tour after the training environment, generally speaking, at sea. The OPTEMO/PERSTEMPO in the ships and squadrons and the seabee battalions has been high. In some cases it has been higher than we would have it, beyond our control. The number one issue I have put in my subsequent testimony before another committee this month to stem that is to adjust career sea pay. This is a unique Navy pay tied to shipboard life. Adjusting career sea pay for inflation is the number one thing we need in order to meet retention goals, because we do have—unlike the Marine Corps, some retention problems. Some of the critical skills, such as nuclear power fuel techs, fire control tech, and some engineering rates, are causing us great concern for the future.

    But in terms of quality of life, we need to hold on to what we have got and incrementally add to it. That is my personal position.
 Page 276       PREV PAGE       TOP OF DOC    Segment 2 Of 3  

    Sergeant BENKEN. Sir, when it comes to retention, we really have the caution lights on when it comes to our second termers. They are the ones that do our training. They are the ones that are the five levels, as we call them. They are the ones that are front-line supervisors. And right now they have been going out at a higher rate than they had, and this has been going on since about 1992.

    The desert rotations have a big impact on us. If you take some of the core career fields like the avionics folks or the F–16 aircraft, the F–16 crew chiefs, if you take the folks at the bomb loaders, et cetera, those are some of the core areas, security police, some of the core areas where we are starting to have some problem. A lot of it has to do with the desert, desert rotation. They see no end in sight. They keep going continuously on the rotations. In the meantime, they wind up going to the deserts or to Kunsan, Korea, for instance, on a remote tour. They come back, and then they stare at desert rotations continuing. Again, no end in sight to that.

    We have training issues. You go to the flight line today, and the middle of the force, the second termers leaving, that leaves you with a high-ranking person and a lot of very inexperienced people in some cases. We have some issues there.

    The perception that—erosion of benefits, the retirement issue is a big issue now because now—in 1986, those that were grandfathered, it didn't affect that. But folks that are being affected now, as the Sergeant Major says, are hitting that 10-, 11-year point, and they are starting to say, wait a minute, my retirement is different. You know, for a Master Sergeant, it is about $2,500 a year less than the person who has a 50 percent retirement. So they look at that.
 Page 277       PREV PAGE       TOP OF DOC    Segment 2 Of 3  

    They look at their medical care, and they feel that Tri-Care, which is going through its bumps and bruises, if they are deployed and their spouse has a problem with that, that becomes an issue for them.

    Lucrative employment on the outside. Our people are very technically skilled, very technically qualified. And they are very disciplined, so they bring something very good to the market on the outside. So there is a lure for those people.

    So it is a combination of a lot of factors, and I will tell you, in response to your question, though, that the Quality-of-Life issues and the MILCON issues are certainly a big factor in the equation, the overall equation of all of that.

    Mr. HEFNER. Just one other comment. I believe it was Mr. Hagan who mentioned his reservations about the experiment with the public and private ventures. I have some reservations about that also. We have been involved in 801 housing and this sort of thing. Of course, I think that the whole thing has to do with the budget and where you can get housing much quicker and it doesn't appear on the budget as much. You know because most of us are not going to be there down the road and you won't be down the road, but we are giving up an awful lot, in my view. Of course, I am not going to make an opinion—I think the jury is still out on the overall operations. But I think we ought to be very cautious about doing this, because I think we could wind up giving away the store and not getting anything back.

    We appreciate you gentlemen coming and your dedication to your troops, and we always welcome you to this committee.
 Page 278       PREV PAGE       TOP OF DOC    Segment 2 Of 3  

    Thank you, Mr. Chairman.

    Mr. PACKARD. Thank you, Mr. Hefner.

    Let me just make a comment, and then we will go to Mr. Wamp for his questions and any comments he would like to make.

    Listening to each of you, it sounds like that we have not done a bad job of addressing Quality-of-Life issues. I am reminded that the first quality-of-life hearing that this subcommittee ever held I think was somewhere in 1981. So it isn't an issue that we have not been addressing for quite some time.

    It sounds to me like overall we have made some significant progress in that direction. I feel that we have. I think the quality of life, as I have been around visiting different bases—and I have tried to do that rather extensively the last couple of years—we are seeing a much better quality of housing, a much better quality of day-care centers. In fact, day-care centers were almost non-existent 10 years ago or 15 years ago. And our hospital facilities generally are better than what I think they have been in the past.

    I served in the services back when many of you were just starting your careers in the services, and I compare what I see today with what I saw then. Camp Pendleton is where I served. So I think we have made great progress. And it may be that now the priorities are changing a little bit. That is what I tend to be hearing from you.

 Page 279       PREV PAGE       TOP OF DOC    Segment 2 Of 3  
    Interestingly enough, most of the concerns that you are expressing that you are hearing from the men and women that you talk to are issues that this committee doesn't address: pay, benefits, deployments overseas and on board ship, et cetera. Those are perhaps a higher priority concern now than maybe the quality-of-life issues.

    Perhaps maybe those spill over into quality-of-work issues as much or more than quality-of-home and -life issues, and maybe we need to evaluate again where our priorities are and ought to be. But right now our jurisdiction and our funding responsibility is primarily over housing and quality-of-life facilities.

    And so we will not neglect that, and we certainly won't be reversing the effort that we have made in the last almost two decades, but the fact is I think we are hearing this morning from you that there are priorities that, again, don't fall into the jurisdiction of this committee that are perhaps even more important right now.

    Sergeant BENKEN. Sir, I would say that the MILCON is extremely important as well. If we did a reversal on that, it would just add to the other issues that we talked about. The dormitories, for instance, we need to continue on the privatization——

    Mr. PACKARD. We are not going to reverse that. There is no question about that. That is our primary responsibility.

    Mr. Wamp?

TRAINING FACILITIES
 Page 280       PREV PAGE       TOP OF DOC    Segment 2 Of 3  

    Mr. WAMP. Thank you, Mr. Chairman.

    I too, from last year to this year, did spend a lot of time focusing on particularly the pay and benefits issue because since my election in 1994 I think we have really focused on trying to move enlisted personnel off of qualification for food stamps and move it up, but, again, we got off on this last year, and it is really not our keeping our eye on the ball with respect to investment in construction, primarily. So I have got two questions.

    One, the Navy talked about training facilities, and I wonder if the other three service areas have priorities on training facilities that aren't mentioned that we need to focus on. If it is that high priority at the Navy, I wonder why there would be a disparity in the other three service areas in training facilities, if it is just a Navy problem or if it is also a problem.

    The second thing is if any of you have any input from your enlisted personnel perspective on the privatization of housing at our bases where a lot of our enlisted personnel actually live now developer-run, leased facilities. I know that is just kind of starting out, but I have a lot of interest, frankly, in that, not personally, but I hear more and more interest in privatization of barracks where we are on a long-term lease basis allowing the free enterprise to compete for this.

    What is your perspective from your enlisted personnel on this whole notion of privatized housing? So the training and the privatized housing. I just thought I would jump off here. The Navy has already talked about training facilities. Any of the other three services?
 Page 281       PREV PAGE       TOP OF DOC    Segment 2 Of 3  

    Sergeant LEE. I will take that question, sir. To my knowledge, although we received legislative authority in 1997 to pursue PPV for barracks, we are not actively pursuing any sort of public/private ventures to replace our single marine housing. We have every intention to keep that under our control. And as I pointed out in my opening statement, we too are very, very cautious about any other program we pursue in PPV, 801-type housing before we get into it. We are very, very cautious about it.

    We do have a couple success stories with some set-aside housing, and we do have some success stories small-scale with some 801-type housing. But those we watch very carefully.

    On the training, for a long time I think—we are trainers. We are operators. So probably in retrospect, for many, many years, we maintained our training facilities maybe at a higher level than we did the facilities that we have been trying very hard now for some time to bring up to standard. So our training facilities are in need of support. Yes, they are in need of support. But reality is how we are housing enlisted, single and family, still remains our greatest requirement, and, again, that is probably because we pay more attention to our training throughout the years than we did to that aspect of quality of life.

    Sergeant Major HALL. Sir, let me take training facilities first. I think we are okay on ranges, our multi-purpose range complex, our fire and maneuver ranges, and our combat training center type events. I think we are okay. The Kassebaum-Baker report, of course, which is going through the review process now and addresses gender-integrated training, may change things. We may see a big bill for new barracks because one of the recommendations is that men and women be billeted in separate buildings. I agree with safe, secure housing. But if the decision is for separate buildings, we are going to be hurting. We may have to build new ones at every training facility. We would have to relook that whole thing.
 Page 282       PREV PAGE       TOP OF DOC    Segment 2 Of 3  

    That is a little bit down the road, but it is coming. Within the next couple of months, we should know.

    Regarding housing, let me talk barracks just a second. We are okay. We are fully funded in barracks to get to the standard. If we look out to 2008 to 2012, we are going to be okay. Now, soldiers chuckle when I tell them we are going to get well, 14 years down the road because they hope to be other places in 14 years.

HOUSING PRIVATIZATION

    With regard to privatization of housing, let me just tell you, we can't get there without doing that. We are underfunded half a billion dollars a year on Army family housing. We will do privatization at Fort Carson, Colorado, which is the first one we will privatize. We were hoping that we would be able to sign a contract this month. My understanding this morning is there has been another protest, and the judge has delayed it until about May 1998 at this time.

    What privatization will do is renovate almost every set of quarters on Fort Carson. That is about 1,800 sets of family quarters, and it will build 840 new sets of family quarters on Fort Carson.

    Now, our whole system of family housing at Fort Carson today is 1,900 sets of family housing, and there are 2,000 people on the waiting list for those 1,900 sets. With our budget, we are going to be short about $3 billion by the turn of the century just in Army family housing. And that assumes that we will be able to revitalize those current facilities that we have now every 130 years. That is unacceptable.
 Page 283       PREV PAGE       TOP OF DOC    Segment 2 Of 3  

    So we just can't get there without privatization, and, sir, I understand and I worry about our culture, as the Master Chief Petty Officer of the Navy says, I understand and I worry about our traditions and our culture. But there is no way we can get there without doing it.

TRAINING FACILITIES

    Sergeant BENKEN. Sir, on the training facilities, I will have to take that for the record. I know that we are making progress in the dormitories or the housing, the RHTs, as we call them, for the recruits, recruit housing. I know we are making some progress on getting those well. But I am not sure exactly what the status is.

    On the single housing, the dormitories, I believe that those are off the books. We are not going to privatize single housing at all. I don't think that is going to happen.

FAMILY HOUSING

    As the Sergeant Major says, on privatizing family housing, I think it is a tool that we can use to leverage MILCON. I think it is—you know, we had one on track down in San Antonio. The frustration was how long it was taking to make it happen. We have to be very careful in how we write the contracts, what the expectations are of the contractors and those kinds of things. But I believe that it is something that we have to give a try and we are going to have to make work. I don't think the MILCON is going to buy us out.

 Page 284       PREV PAGE       TOP OF DOC    Segment 2 Of 3  
    Chief HAGAN. Let me just clarify part of my statement. First of all, let me address the barracks issue by telling you I think shipboard berthing makes our issues, Navy issues, unique. Quite honestly, I believe it makes them a priority issue because we have huge numbers of Sailors living in 50- to 100-person berthing compartments, some even larger than that, living in conditions that are—were they not unavoidable, they would be unacceptable.

    Those Sailors live there until they are an E–5 in the career force before they are eligible for single BAH only recently passed by the Congress. Prior to the last 2 years, a single E–6 was required to forfeit housing allowances to live aboard ship.

    We are transitioning to the 1 plus 1 standard over a lengthy period of time via a 2 plus 0 standard. I find that acceptable so long as our POA&M each year is visited by all the right people, including members of this subcommittee.

    In terms of PPV, I will restate my concerns. I don't disagree with either statement that the Sergeant Major or the Chief Master Sergeant have said. However, I have serious concerns that we are not using PPV to supplement but that because it is attractive in the way it fits into the budget, that we will begin to replace the infrastructure that we now own.

    I would be much more comfortable if there was an in-writing commitment—perhaps it is the business of this committee to look into this—an in-writing commitment to preserve the existing infrastructure or some portion of it beyond debate. For example, Navy has fewer family housing units, traditionally Navy-owned, than any other service does compared to the number of married Sailors. And so both our situation and our need is greater.

 Page 285       PREV PAGE       TOP OF DOC    Segment 2 Of 3  
    We can't get there from here either without PPVs as a supplement, but PPVs cost Sailors considerably more than BAH, and in a certain period of time, we don't own them.

    Mr. HOBSON. Excuse me. Could you explain that again? Why do they cost more?

    Chief HAGAN. Well, sir, when you move into family housing, traditional family housing, Navy-owned, you forfeit BAQ and BHA or BAH, and you have no other expenses other than those utilities that are considered a luxury—your telephone and your cable TV, for example.

    Mr. PACKARD. In other words, the base housing allowance covers the cost of housing.

    Chief HAGAN. The public/private venture varies according to the unique agreement with the contractor. The statutory goal of BAH, is 85 percent of your total housing costs. In the tiered hierarchy structure by rank, the PPVs can cost a great deal more than that, especially for junior personnel, even mid-grade career personnel.

    So, my concern is that in order to provide the number of family housing units in fleet concentration areas that we build new and more efficient ways of managing that Navy-owned housing, while simultaneously exercising the PPV option.

    In this testimony I have gone on the record as being not opposed to metering family housing units and charging for utilities. This may be necessary in order to maintain the Navy-owned infrastructure of family housing, because there is an efficiency that I see no other way to gain. We currently have no incentivation for conserving utilities in government-owned family housing.
 Page 286       PREV PAGE       TOP OF DOC    Segment 2 Of 3  

    But my concern is not that we should never use PPV. We certainly have several places where we have more housing options than we would have had. I think we may be leading the other services in the use of PPVs. We have it in place in Everett, in Corpus Christi, and this is not the 801, 802, but something beyond that.

    I am not opposed to PPVs. I am just opposed to the fact that it might slowly and almost surreptitiously displace the traditional family housing owned and controlled by us.

    Mr. HEFNER. Would the gentleman yield?

    Mr. WAMP. Sure.

    Mr. HEFNER. I hope it is going to work. You know, we met with the Secretary a while ago. This has been a while developing. At Fort Bragg, we did 300 801 housing units. It is a nice little city there. If I was in the private sector and I was the contractor, I would do everything I could to get in to build these ventures or build 801 housing. I don't know of any other business where you could be guaranteed full occupancy and after 20 years to say this is mine. You have to renegotiate, which in most cases it would be applicable to the services.

    But it is something that I think we have got to be very careful about because we could wind up, long after you gentlemen are retired and I am gone—of course, the chairman will probably still be here, but we could wind up at the mercy of—with nothing that we own, like the old Cape Hart building. I understand why we are doing it. It is because of the budget.
 Page 287       PREV PAGE       TOP OF DOC    Segment 2 Of 3  

    There is an answer to it, and it is called money. If we would fund it, but then the budget would be completely out of sight. We couldn't get enough allocation.

    I just think we ought to be very careful. As you said, as we line up these contracts, make sure that the Government is going to be responsible to keep it for the occupancy and for the money. So we want to make sure that people that are doing the venture are going to be responsible for the upkeep and all the things that go along with it. I have been on this committee and on the Defense Subcommittee long enough that when we get in a situation where we are going to have lawsuits, we never win, whether it is with architects, defense contractors, or anyone else. The taxpayer never really wins in a lawsuit.

    So, Mr. Chairman I thank the gentleman for yielding and I just think we ought to be real careful about doing this. Proceed with caution. Let the buyer beware.

    Mr. WAMP. Just real quickly, sir, Mr. Hagan, moving up the live fire range for one year, what is the cost of that?

    Chief HAGAN. That is currently budgeted at $7.1 million. I understand it would be cheaper if it is moved up a year.

    Mr. HOBSON. So it would be less than seven?

    Chief HAGAN. I understand it would be less than $7 million if it is built sooner. That is the projected FY00 projection, sir.
 Page 288       PREV PAGE       TOP OF DOC    Segment 2 Of 3  

    Mr. PACKARD. Mr. Hobson, we will go to you.

    Mr. HOBSON. Well, first of all, sir, I think we need to know, the committee needs to know, how much savings there would be in doing that, because money is very scarce in this committee.

    Chief HAGAN. I can get that for the record quickly, sir, the projection.

    [The information follows:]

    The Navy does not anticipate any savings due to accelerating the Small Arms Range at NTC Great Lakes.

    Mr. HOBSON. Because we want to help you but we have a lot of priorities, and I think that is a priority.

    I am concerned about the housing also. I have Wright-Patterson Air Force Base in my district, and housing is, I think, still a problem. I am also concerned, have some of Mr. Hefner's concerns, and I happen to come from the real estate business and a builder, but I am concerned about winding up with the problem that we have got today with Section 8 housing where we have all these contracts out there which we have done, and we can't afford now to continue the payments. And that is a problem I am worried about us getting into if we do too much of this, that we won't have the money to pay the bill when it comes due.
 Page 289       PREV PAGE       TOP OF DOC    Segment 2 Of 3  

    On the other side of that, I am concerned about the fact that the base housing doesn't take in all the costs of that when we do it because there are costs in there, like the metering and things of this sort, that don't encourage people to do the things that other people do in trying to meet their utility costs.

    But do most—let me say this, if I can say this right, because when I was in the service, I never wanted to live on base. But if you are overseas, you can't afford it. But even then I didn't want to live on base, but I was made to live on base. So do most—do you survey people? Do most people want to live on base if they can? Or are they forced to live on base by the pay? Or in some cases, do you want to control their lifestyle?

    Chief HAGAN. Let me begin answering that by telling you I think most people want the option. I personally strongly want the service to preserve that option for the career force as a priority. There is an internal debate occasionally about the housing being the first priority for the career force, for senior enlisted or for junior enlisted, where the need can be quantified as greater. But the simple answer I would give you is most people want the option, and it is an individual, specific, situational preference.

    Sergeant LEE. Sir, your question is kind of—in the Marine Corps it is kind of a combination of all the above. Absolutely we want to control the lifestyles of the single marine, especially the first-term single marine. We want to control that person's lifestyle. We intend to keep that person billeted on base to the extent we possibly can and look after them in that manner.

 Page 290       PREV PAGE       TOP OF DOC    Segment 2 Of 3  
    What we would like to continue to have is, as the Master Chief said, the option for those who can't afford to live off base—if they had the luxury of making that decision. Finally—you are right, those who live in high-cost areas want very much to live on base. We have long waiting lists. Camp Lejeune, North Carolina, if we have a waiting list, it is purely because of the size of the house or the location because the cost of living at Camp Lejeune, North Carolina, is not that high.

    So that is where the option comes in. We have most of our marines under 24 years old, single, serving on one contract, our priority is—a good term you used, ''control of their lifestyle.'' We are going to continue to do that. That is what our emphasis is on.

    Mr. HOBSON. Having been an enlisted man, I know about controlling lifestyle. [Laughter.]

    Mr. PACKARD. They call it discipline, Mr. Hobson.

    Sergeant Major HALL. Sir, from the Army's perspective, Army families prefer to live on base. They want the close proximity to medical treatment, the exchange system, the commissaries, the convenience. All the quality-of-life benefits are convenient for those who live on base. So most soldiers with families would prefer to live on base.

    Mr. HOBSON. Do you survey that? Or how do you handle that? Does that come like his thing where—I mean, let me put it this way: There are subtleties that happen in the service that—and maybe sometimes they aren't to subtle—where you don't have any choice but to say, yes, I want to live on base.
 Page 291       PREV PAGE       TOP OF DOC    Segment 2 Of 3  

    Mr. HEFNER. According to how they explain it to you.

    Mr. HOBSON. Yes, things have been explained.

    Chief HAGAN. Our actual waiting lists are better than any survey we could conduct. In places where we have waiting lists, people are clamoring and calling almost twice a month to see if they have moved up. So we have some analytical proof, as the Sergeant Major said about soldiers, in many locations Sailors not only want to live on base, the only way they can envision themselves living in that area is to live in Navy family housing. Pearl Harbor is one such example.

    Mr. HOBSON. I have been there. I have toured that facility.

    Sergeant LEE. Sir, we do have housing locations, family housing locations, where we require them to live on base. Otherwise, we would have a great emptiness. But we do mandate sometimes that they live on base, even if they don't want to.

    Mr. HOBSON. I have seen that, not just in your service but I think it has happened in the Air Force.

    Mr. HEFNER. Would the gentleman yield?

    Mr. HOBSON. Yes.

 Page 292       PREV PAGE       TOP OF DOC    Segment 2 Of 3  
    Mr. HEFNER. The chairman mentioned day-care, and I remember years ago we began to really concentrate on day-care at Fort Bragg. It seems that the people with small children seem to have sort of a family atmosphere where they know that if you have got a problem with kids or what have you, you got a good volunteer group there, and they know these folks. It just seems to work out better if the housing is adequate. That is the whole thing. Just knowing that they can call somebody and they are going to be protected and helped out if they get into a problem. I think our improvements with daycare that we have done over the past few years, especially with the type of services we have now, has been a real boom to the military, in my opinion.

    Sergeant BENKEN. I believe the number of people on waiting lists for family housing in the Air Force is roughly 40,000, which is a significant number. Given the high deployment rates that we have nowadays, it provides a security for them, their families are back home, and we have people that are deployed more than 120 days a year. It gives them a great warm feeling that their family is on base, secure, and has access to the facilities and things like that.

    On the single side, thanks to your help, we are buying out the central latrines, the gang latrines, you know, which is no way to live, and for that reason, our troops are finding much more satisfaction with living on base on the single side of the house, too. As a matter of fact, we have some E–5s now that would like to live on base in the dormitories because of security and——

    Mr. PACKARD. If the gentleman would yield, there is no question that as we move toward the 1 plus 1 or the 1 plus 2 facilities, the desire to live within those facilities will become, I think, even greater.
 Page 293       PREV PAGE       TOP OF DOC    Segment 2 Of 3  

    I can give you a personal observation. My daughter is married to an Air Force physician, active duty, and they just were able to get on base. They had been on the waiting list. Every base they have served in—and he has served now for 10 or 12 years. Every base they have served on, they have put themselves quickly on the waiting list to get on base housing. They can afford off-base housing. As a physician in the service, he is probably one that could afford to live off base in most any economy. But they still like to go on base. That is just a personal observation.

    Mr. HOBSON. What percentage of your first-termers are married?

    Chief HAGAN. It is now on the order of 23 percent of our first-term sailors are married for the entire force, it now approaches 60 percent now.

    Mr. HOBSON. I am going to go through that now because I want to ask the question. I sit on two committees. I sit on Defense Appropriations Committee as well as this one, so I am dipping into some knowledge from that, too, if you don't mind.

    Sergeant Major HALL. Sir, I can only say 62 percent of the Army is married. I would have to state for the record the percentage of first-termers.

    [The information follows:]

PERCENTAGE MARRIED AMONG FIRST-TERM SOLDIERS

 Page 294       PREV PAGE       TOP OF DOC    Segment 2 Of 3  
    As of September 1997, 59.4 percent of the active-duty enlisted force was married. Because the data on marital status do not provide information on term of service, we have attempted to get at this by looking at the percentage of married E–1 through E–3 with two, three, and four years of service. These percentages came to 21.2 percent, 23.6 percent and 23.9 percent, respectively, though some unknown percentage of them are prior service. Among non-prior service enlisted recruits who accessed in fiscal year 1997, 14.9 percent were married.

    Mr. HOBSON. I would like to know because it is important for the next question I am going to ask, if you don't mind.

    Sergeant LEE. Sir, about 39 percent of my total enlisted force is married, and about 43 percent of my enlisted force had dependents. Only 17 percent of my large first-term force, E–3 and below, 78,000, only 17—only about 16,000 are married. To give you the exact numbers, too, I have to do the same thing. I would say of my first-term force, my 110,000 marines serving on one contract, my youngest marines.

    Mr. HOBSON. That is a 4-year——

    Sergeant LEE. Yes, sir, on average. I would say roughly maybe 25 percent of them are married.

    Sergeant BENKEN. Roughly 35 percent of the Air Force are single. I will have to give you the first-term single.

    [The information follows:]
 Page 295       PREV PAGE       TOP OF DOC    Segment 2 Of 3  

    Air Force 1st Term Enlisted Martial status is as follows: Married: 30,331 (32.1%), Single: 62,366 (65.9%).

    Mr. HOBSON. The reason I ask that, I think it is very difficult for a first-termer—I have never heard ''first-termer'' but that is an interesting term—for the first-time enlisted person to be married and get the kind of salary that is necessary to have that lifestyle. I think that is very hard to do, even with the extra stuff that you give them, for a young family like that.

    Especially using this first term or first enlistments, you are having a lot of, I guess, stress within the services. You are beginning to learn about the service. So it makes it a double shot. But that is enough on that.

MEDICAL CARE

    I also want to talk about one other area, if I could, Mr. Chairman, and that is health care. As you may know, or may not know, I have been doing some stuff in the National Security Appropriations subcommittee on the quality of health care, and there have been some recent discussions on that. Mr. Hefner is over there too, so he knows some of the stuff that we have been doing on that.

    But the facilities that we have for health care I think is something that needs to be looked at and continued to look at. I know we have done some things at the hospital at Wright-Patterson. But I would like to know how you think the quality of your facilities are. And then secondly, how that is affects your retentions. I do not run into any retiree today who said, the only reason they re-upped was not for the retirement today, it was for the health care. I do not know whether that is just because it is changing, but that is what they all say. I do not think that that is particularly true, but that is what they say.
 Page 296       PREV PAGE       TOP OF DOC    Segment 2 Of 3  

    I would like you to discuss, if you could briefly, one, your facilities; two, what the changes in health care and how important all this is to you in your retention of people.

    Sergeant Major HALL. Sir, I will start off. I think the facilities that we have for health care today are adequate to support the Active duty force. When you take the retiree population into consideration, however, I think we are going to find some challenges. Under TriCare, we are going to cap the number of people that will be enrolled in one of the programs, in TriCare Prime. That will be a limiting factor. The facilities will not support everybody who was promised health care.

    With regard to the health care system today, we have problems with TriCare. They are being worked, but we have an education problem on educating the force how to make TriCare work for them. It is a different TriCare almost everywhere. I found in every region that I go into, it works a little bit different. It is not easily transportable from region to region. There are some nuances there that soldiers have not figured out yet, but they are being worked.

    When I talk to the medical folks, they tell me that we are now working on how to handle our soldiers who are stationed in remote locations away from a military installation. We have big numbers of them out there supporting the Reserve components and recruiters. They are not near a military installation. They do not have access. So they are working on how to handle them under TriCare Remote.

 Page 297       PREV PAGE       TOP OF DOC    Segment 2 Of 3  
    We have problems with access and the time limits that it takes to receive care. That is being worked. We have problems with how claims are processed and how reimbursements are paid. That is being worked. What I say to my soldiers is, ''It is working, working, working''. That is not a good answer for them, but it is the only answer I can give them.

    Mr. HOBSON. Anybody else want to take that on?

    Sergeant BENKEN. I will tell you that we have the horror stories that the Sergeant Major was talking about, which gets compounded by the person who is in the desert, for instance, and calls home and the spouse is wrestling with some kind of a TriCare payment that has not been made, and the creditors are calling because the doctor has not been paid and all those kinds of things. But those things are being worked very, very aggressively. And where we find those, I think we are able to work them.

    There is no doubt that we are going to have to transition to another medical care system to support the existing one because of the declining infrastructure.

    Mr. HOBSON. How about the facilities?

    Sergeant BENKEN. The facilities, I think for the most part are good. They will not be able to accommodate the number of retirees that we have, obviously.

    Chief HAGAN. Even before we began to downsize hospitals, we could not accommodate all the retirees who desired medical care, although in some locations we did very well, for instance, Orlando. When we BRAC-ed Orlando there was quite a bit of retiree trauma over access to that treatment facility going away because they had had good access.
 Page 298       PREV PAGE       TOP OF DOC    Segment 2 Of 3  

    If you leave the retirees out of the equation, our facility infrastructure is still not sufficient to see dependents. TriCare Prime continues to improve, and I would concur with the Sergeant Major of the Army as I heard him state, that it improves too slowly. I know it is a large bureaucracy and it is a huge monster.

    My own personal concern is that we will not be able in the outyears to preserve the TriCare core benefit. I think the core benefit is good. The $6 copay for E–4 and below, the $12 copay for E–5 and above for outpatient, the $12 a day inpatient cap, the very reasonable catastrophic cap, and the no annual deductible is a good plan. I am pleased with that.

    My long term worry is not that we will not work out the bugs and finally make it region to region consistent, et cetera, but that we will not be able to keep the cost that low. And of course, I think the retiree will gradually get accustomed to the fact that the $230 per person, $460 per family enrollment is reasonable if there is a TriCare network. As one who will be retired before you interview my peers again, I say if there is a TriCare network for me to enroll in, $460 a year seems to be a very reasonable figure.

    Mr. PACKARD. If the gentleman would yield. Let me ask, are you satisfied with the retention of your providers, your physicians, nurses, and so forth?

    Chief HAGAN. No, sir. That continues to be a problem in some areas where providers are upset with slow payment, with interpretation of procedures that are eligible and those that are not. That is one of the issues that we are told regularly from the OSD health affairs and other places that we are working on.
 Page 299       PREV PAGE       TOP OF DOC    Segment 2 Of 3  

    Sergeant BENKEN. Region by region TriCare is very successful where it has had time to mature. In other areas we have significant problems with it. It is a huge system and it is going to take a little bit of time to bring it in place.

    Sergeant LEE. Sir, the Marine Corps owns no medical people or medical facilities, per se. We depend upon the other services.

    Chief HAGAN. You depend upon the Navy.

    Sergeant LEE. Well, the other services. I want to say, where the health care——

    Mr. HOBSON. It is hard for him to say Navy.

    Sergeant LEE [continuing]. First of all, where the facilities are available, the facilities are very good. Where the care is readily available, the care is outstanding. I want to lay that out.

    But where it is CHAMPUS-ed out or TriCare-ed out, the growing pains, the disconnection, the happiness with the providers, all of those things roll into play and they are considerable. I deal with them all the time.

    Is our retention affected? I do not know. I would have to say in some ways for an individual, yes, it affects retention. But again, overall I do not have a retention problem so I cannot measure the one thing.
 Page 300       PREV PAGE       TOP OF DOC    Segment 2 Of 3  

    Mr. HOBSON. Just one other thing I want to mention. It is one thing I think is going to happen from talking to the Surgeon General. One of the problems that troops have when they move around is records following them. I think with the computerization of records project that you are all going to have problems that people have and delays in some payments get better. We need to get this little computer chip of records that everybody is going to have, which is moving forward pretty rapidly.

    The last thing I want to mention is what is being done in some places including Wright-Patterson. In pediatrics, they have actually gone to, as I understand this is happening, they have gone to the children's hospital in the area. They have got a contract now with—the hospital at Wright-Patterson does. Rather than continue on with pediatrics, the Air Force is going to refer those people to the children's hospital in the area. I think may be a thing that a lot of services need to look at.

    Rather than building new facilities you may want to look if you can outsource some of those facilities. They are still going to have the same personnel, but you are going to get some additional help.

    So I know it is a tough problem and I appreciate the indulgence of the chairman, but it is very helpful to have you talk about these things.

    Chief HAGAN. Mr. Chairman, could I respond quickly to the live fire, what you said in the beginning? I should tell you that I extrapolated the perception of savings in acceleration based on the data that had we programmed this in 1993 when BRAC predicted the consolidation of all recruit training at Great Lakes, it would have cost 5.1. We chose to depend upon the old range lasting longer than it environmentally did. The cost is now projected at 7.1.
 Page 301       PREV PAGE       TOP OF DOC    Segment 2 Of 3  

    But regardless of savings, I would just like to underscore again, it is a considerable deficiency to send Sailors to their first duty station without live fire. As the security alert force, the backup alert team, the quarterdeck watch and all sorts of roving watches aboard war ships, Sailors are required to be armed and qualified to at least a familiarity degree. Thank you, sir.

    Mr. HOBSON. Thank you very much, Mr. Chairman. Appreciate your indulgence.

    Mr. PACKARD. Mr. Edwards?

    Mr. EDWARDS. Thank you, Mr. Chairman. I think the challenge of our committee and the other defense committees that Mr. Hefner and Mr. Hobson sit on, is to take a limited amount of money and find a way to best use it to keep the best and brightest in the military. One of the frustrations I have is trying to get objective information on where should we really put our resources. Is it in health care, is it in retirement programs being enhanced, is it in housing, is it in child daycare?

QUALITY OF LIFE SURVEY

    We are either going to make those decisions totally subjectively—and our judgment is not always bad. Between the people we talk to and certainly the people you talk to, you get good anecdotal feedback. But is there any sort of objective system in all the services when enlisted personnel leave, they fill out a form that could then be computerized and available to us to show us? You could do it by installation, you could do it by all sorts of different categories, and rate the primary reasons people are getting out of the military.
 Page 302       PREV PAGE       TOP OF DOC    Segment 2 Of 3  

    Is that system available? Could this committee get that kind of information next week if we wanted it?

    Chief HAGAN. Yes, sir. I can give you the Navy's short answer. We have exit polls that are fairly detailed, and I can provide for the record very quickly a summary of, and any specific data you would like. That is the poll that I quoted when I said the primary reason given by officers and enlisted for leaving the service after the first or second, and even subsequent enlistments because we have a third and fourth term retention problem in the Navy, is family separation.

    That poll is as detailed as 10 or 12 blocks or checks in a priority. Again, I can provide that for the record, sir.

    [The information follows:]

    The annual Navy-wide Personnel Survey and Quality of Life Survey were designed to collect opinion data on a systematic basis and to provide timely information on issues of importance to policy makers. The samplings representative of the entire Navy population, allow the identification and analysis of trends in opinions and attitudes toward plans, programs, and policies that materially affect the performance and morale of a Navy Personnel. Both surveys are administered and analyzed by the Navy Personnel Research and Development Center in San Diego, California. Copies of the 1997 Navy-wide Personnel Survey and 1997 Quality of Life Survey are provided for the record.

 Page 303       PREV PAGE       TOP OF DOC    Segment 2 Of 3  
    We also collect information on Sailors' attitudes through our Retention/Separation Questionnaire, given to enlisted Sailors when they reenlist, extend or separate. Officers are asked to complete the questionnaire when they leave active duty or execute a permanent change of station move. Response to the questionnaire is voluntary. Sailors are asked to rate their satisfaction with 45 aspects of Navy life and to identify the most important reason for leaving or thinking of leaving the Navy. A copy of the Navy Retention/Separation Questionnaire and OPNAV Instruction are provided for the record. Officer and Enlisted ''Reasons for Leaving the Navy'' data for the fourth quarter of FY97 are provided below:
    Offset folios 817 to 818 insert here

    Mr. EDWARDS. How long has that poll been in existence?

    Chief HAGAN. Many years, sir.

    Mr. EDWARDS. So we could go back and look at the trends to see if the general trends are changing, to see what their concerns are.

    Chief HAGAN. The trend is available. Long term trends are interesting to analyze, and I could also provide them.

    Mr. EDWARDS. That would be interesting to see, and for the Defense Appropriation Subcommittee to see. Do the other services——

    Sergeant Major HALL. Sir, to the best of my knowledge the Army does not have an Army-wide survey. We do it installation by installation. I have seen some of those and it runs the full gamut of reasons. And of course, those reasons for getting out are coming from the 30 percent who would probably have gotten out anyway. They had already made a decision to get out.
 Page 304       PREV PAGE       TOP OF DOC    Segment 2 Of 3  

    Mr. EDWARDS. I realize we would have to filter that out. And ideally you would also take a poll of those who are reenlisting and ask them why they are reenlisting. I think the ones leaving might be a little more freer to be totally honest in what they say. But it seems to me at least there ought to be a system in place of trying to collect that data, and we could use our judgment and you could use yours to filter through it and determine what it says.

    Otherwise, we are just making these multi-billion dollar decisions of resource allocation based on totally subjective experiences in our own particular districts based on who we just happened to see when we went to an installation in our district.

    I will follow up. I am having a meeting with General Reimer today even. And it seems like the Army ought to have some sort of way to collect that information on a system-wide basis.

    Sergeant Major HALL. There may be one. I am unaware of it, if there is one. I will certainly provide a definitive answer one way or the other to you.

    [The information follows:]

SURVEYS

    The Army's Sample Survey of Military Personnel (SSMP) is conducted twice yearly on a sample of 10 percent of the active-duty officers and 2–3 percent of the active-duty enlisted members. Respondents who indicate that they are thinking about leaving the Army are also asked to indicate the reasons that first made them think about leaving the Army, as well as the most important reason. These questions have been asked since 1992, so we do have trend data on the topic of why soldiers leave the Army. While some reasons are commonly mentioned over the years by both officers and enlisted members (e.g., amount of time separated from family and overall quality of Army life), issues which are increasingly mentioned as the most important reason for leaving the Army are retirement benefits and the amount of basic pay.
 Page 305       PREV PAGE       TOP OF DOC    Segment 2 Of 3  

    Mr. EDWARDS. Thank you very much, Sergeant Major.

    Sergeant Major HALL. From an Army perspective, and from an all-service perspective, sir, I think it is taking a budget and having to balance modernization, readiness, and quality of life. It is definitely a balancing act.

    Mr. EDWARDS. It will always, even with this information, require subjective decisions by members of Congress themselves. But at least those subjective decisions could be made based on some sort of objective data.

    How about the Marine Corps and the Air Force?

    Sergeant LEE. Sir, on the Marine Corps, no, we do not have one, and I do not think you would benefit from it if you concentrated on those who leave the service. My example is very simple. Again, the way we operate, I have got 28,000 very good first-term marines getting out this year; 28,000. I can only let 4,500 of them stay in. So to ask the other 24,000 why they are leaving, many of them are leaving because they cannot stay.

    You said something that is very positive. In the case of the Marine Corps we ask, why are you staying and what would you like to see help you in the next tour you are going to sign on for. We would better off——

    Mr. EDWARDS. You do that system-wide?

 Page 306       PREV PAGE       TOP OF DOC    Segment 2 Of 3  
    Sergeant LEE. No, sir, we do not.

    Mr. EDWARDS. I do not understand why the services would not do that system-wide for those reenrolling as well as those leaving.

    Sergeant BENKEN. Sir, the Air Force does that. We do a quality of life survey. We talk to not only individuals—periodically we do a survey of individuals. But we also do first sergeants and commanders. Based on that we develop a quality of life strategy where we have seven primary points which are fair and equitable compensation; balancing the high OPTEMPO that we have, especially with the desert rotations; quality health care; safe and affordable housing; preserve the retirement system benefits; community programs for our teenagers and children; and expanding educational opportunities. Those are the things that are on the list.

    The only thing that changes or has changed from year to year when we have done has been the order somewhat of those items. But we have a Quality of Life Strategy and we would be glad to provide it to you.

    [The information follows:]
    "The Official Committee record contains additional material here."

    Mr. EDWARDS. Yes, in fact those of you that do, either for those leaving or those re-upping, whatever you have maybe for the last two or three years, just to look at that, I would be very grateful.

    Sergeant BENKEN. Be glad to do that.
 Page 307       PREV PAGE       TOP OF DOC    Segment 2 Of 3  

    Mr. EDWARDS. Thank you, Mr. Chairman.

    Mr. PACKARD. Thank you.

    We welcome Mr. Kingston from Georgia here, and if you have any questions?

BINGO

    Mr. KINGSTON. Yes, Mr. Chairman, thank you.

    Let me apologize to you gentlemen for being late. First question, bingo, particularly the Department of Army. Are you familiar with the bingo situation right now?

    Sergeant Major HALL. Sir, I was just down at Fort Stewart about two weeks ago, so I am vaguely familiar with the situation there now.

    Mr. KINGSTON. As I see it, you are damned if you do and damned if you do not. You have a private contractor who is running the bingo, and because of that he has been able to generate a lot more money, but it has put a hurt on the local veterans organizations and non-veteran organizations and whatever charity groups are running bingo, and they are trying to work out something about it.

    What is your recommendation? I see it both ways. I will say this, Colonel Fife—I assume you have met with him?
 Page 308       PREV PAGE       TOP OF DOC    Segment 2 Of 3  

    Sergeant Major HALL. Yes, sir, I have.

    Mr. KINGSTON. He is a top-notch guy. Have nothing but the highest things to say about him. He has done extremely good work with citizen groups in the middle of deploying 3,000 troops to Kuwait. So it is a smaller problem on the scale of things. But still, the locals just will not let it go because, you know, they can raise $10,000 a year or more in bingo. It is huge money to them. They are mostly retirees. They love bingo. Just let me hear some sage advice on it or a perspective.

    Sergeant Major HALL. Sir, I do not want to tell you I do not have an opinion on it because I do. I think the problem is that the veterans' groups and other people outside the gates in Hinesville are not using the private organizations. They are waiting until they come on post to play bingo. So I do not want to tell you I do not have an opinion on it.

    The division sergeant major down there owes me some information. Could I get that back to you when they put their work together?

    Mr. KINGSTON. If you will. I have mixed emotions about this. I see it both ways. I am not doing any kind of grandstanding at all. It is an issue that I wish could quietly fade away. We have been wrestling with it I think for six months?

    Sergeant Major HALL. Yes, sir.

 Page 309       PREV PAGE       TOP OF DOC    Segment 2 Of 3  
    Mr. KINGSTON. And we cannot get rid of it as an issue. Colonel Fife has been absolutely a prince about the whole issue.

    But you do have some legitimate concerns out there, and one of the concerns is that a U.S. military post is exempt from the Georgia State laws regarding bingo, which in the perception of the charity groups, gives them an unfair advantage. Whatever you can do—I do not know exactly what I want you to do except get the locals happy.

    Sergeant Major HALL. I do not either, sir. I would suggest probably that this not come to this subcommittee, but to come directly to you. If I could come to you and talk to you about that one, would that be appropriate?

    Mr. KINGSTON. No sweat whatsoever. Let us just agree to continue talking.

    Sergeant Major HALL. We will do that.

    Mr. KINGSTON. Again, I really appreciate the way you all have handled it.

CHILD CARE

    Now let me switch over, particularly to Air Force, because in your testimony you said that daycare was one of your best—to give the quote, one of the most important benefits we give our people, particularly young families is base child care facilities. Now interestingly enough, the Navy has told me that to some degree daycare is a problem because DOD is not in the child care business and should not be in the child care business, and it takes—it is a diversion from the mission statement of readiness to have officers who are involved in daycare.
 Page 310       PREV PAGE       TOP OF DOC    Segment 2 Of 3  

    It would appear that we should look at privatizing daycare rather than going into daycare. You may know at King's Bay we pushed for a daycare facility. This committee was successful in funding it, and this committee took a leadership role in that. But now we have got daycare and it takes away from nuclear submarines.

    Chief HAGAN. When you say the Navy has told you, I am not sure if you are referring to specifically my statement.

    Mr. KINGSTON. No.

    Chief HAGAN. I did make a specific statement about child care because I have very serious concerns that we do not try to quantify 100 percent of the need. I occasionally attend a meeting at the OSD level and at the Navy level where the goal of some entity is to establish 100 percent of the need. I always want to define the need before we establish 100 percent or 85 percent of it and build an infrastructure to support it. I believe that as some people define the need, that it is a bottomless pit.

    I will go very quickly to my bottom line, and that is that some child care needs, some parental obligations, no small amount of them are mutually exclusive with continued military service. I am not interested in building an infrastructure that can meet every child care need but simply one that perhaps prioritizes member-member marriages, single parents, and never takes into account 24-hour child care or deployment guardianship. Believe it or not, that is occasionally raised as an issue by those who would throw in with us for a career if we could meet their every need.
 Page 311       PREV PAGE       TOP OF DOC    Segment 2 Of 3  

    That is the basis for my reservations. And as I said in my testimony, we owe the single Sailor more prior to plussing up or doing any more for child development centers. The single Sailor who lives on a ship has been long neglected. In our quality of life master plan that I would be glad to submit along with that survey which Dr. Roscoe, our ASN at the secretary level has devised, is a leading part of that single Sailor need right now. But those two subjects are intertwined in my mind, single Sailors' needs and child development center.

    Mr. KINGSTON. Is this with deployability as part of this discussion?

    Chief HAGAN. Deployability, I am convinced, cannot be a part of child development center planning.

    Mr. KINGSTON. I agree with you.

    Chief HAGAN. We deploy for six months, or down in King's Bay they go for 90 days. That arrangement for child care for that period of time has simply got to be the parental responsibility. And in some cases it does conflict in an unresolvable way with continued service, and I respect the judgment of those who leave our organization because of that obligation.

    Mr. PACKARD. If the gentleman would yield for a minute on that. Is that particularly true with single parent deployments?

 Page 312       PREV PAGE       TOP OF DOC    Segment 2 Of 3  
    Chief HAGAN. Particularly true, sir. I respect the judgments of many single parents when they choose to leave our service. Some ratings serve five years of sea duty followed by two years of shore duty. I respect the judgment of a single parent who says, my situation is not temporary. It is semi-permanent, and I am forced to separate because there are obligations in this all-volunteer force that are more important.

    Sergeant BENKEN. Sir, if I could address the child care issue from the Air Force standpoint? We tie it to readiness. Close to 70 percent of our force is married. It is a fact of life that we have children, and we have to deal with that. With the deployment rates that we have today, it is necessary for our troops that are overseas to have great faith that back at home those children are being cared for properly. We have a very high accreditation rate. As a matter of fact, the President said that our child care system is a model for America. That is what we have done with it and that is how far we have taken it.

    Mr. KINGSTON. Is it a seven-day-a-week availability, or is it five?

    Sergeant BENKEN. No, sir, normally it is five days. It is based on about 50 hours a week.

    Mr. KINGSTON. Is there a push to go to seven days a week?

    Sergeant BENKEN. We have always told local senior enlisted advisors that they can look at the child care system. If you have the demand and you can meet it and pay for it without losing money, which is what we are chartered to do, then they can certainly look at it. But I will tell you, in most cases we cannot provide 24-hour a day child care seven days a week, no.
 Page 313       PREV PAGE       TOP OF DOC    Segment 2 Of 3  

    Mr. KINGSTON. Is it possible that this is going to evolve into deployability and a decision of reenlistment, the availability of child care?

    Sergeant BENKEN. I think it will evolve into a reenlistment issue if we degrade Child Development Centers or child care for our armed forces. We are meeting about 60 percent of the need right now, I believe is the number.

    Mr. KINGSTON. Is this something, Mr. Chairman, that we should be concerned about looking into in the future? Because I am concerned that it would become the linchpin of reenlistment. Then you are in the child care business in a major way, far more than we intended, and it actually becomes one of the huge benefit considerations, to where we are just taking the money away from airplanes and ships and tanks and so forth.

    Maybe we should acknowledge that we are in the business, but it should be privatized as much as possible. Because I would rather have your manhours spent training people how to fly planes than what the proper conduct is in a child care center.

    Sergeant Major HALL. Sir, if I could add a little bit to that. Within the Army we are currently, in the aggregate, meeting about 65 percent of the child care needs, and that is the DOD goal. That is on base. There is no move for guardianship type care. Neither is there a move to put soldiers in the business of running those. Those are primarily non-appropriated funded facilities.

    We do supplement that some, and we are trying to get away from that. But the cost of bearing payment for child care, minus our supplements, is done by the soldier. They would, quite frankly, pay for it whether it is on base or off base. So on base enhances readiness because it is child development as opposed to what I call, in some cases, warehousing of kids in some places off the installation.
 Page 314       PREV PAGE       TOP OF DOC    Segment 2 Of 3  

    Mr. PACKARD. If the gentleman would yield? I visited several child care centers on bases and I think that the comments are correct. That is, my observation is that they are not run by the military. They are not a military installation, and there are certainly no active duty personnel involved. They are usually either volunteers, or the amount that they receive from the parents with the children at the center will cover the cost of the personnel required to operate them on a daily basis. And really they are quite well done.

    I think, frankly, our assignment and our role is to provide the facilities. But as far as the operational activities, I have not sensed that that has detracted in any way from readiness or from mission assignments. They are run rather independent of the military operations and military supervision.

    Chief HAGAN. Sir, I would like to provide this material for the record. I cannot give you the exact data now, but there is a substantial subsidy in the operation of the child development center network independent of the MILCON. If they were a break-even facility, my stance would be very different.

    The potential for privatization and for in-home family caregiver, which is the portion of child development that I am the most excited about and supportive of, the potential for those two issues, privatization and in-home family caregiver to meet an increasing portion of the need I am excited about.

    But the actual cost of running the child development center, from utilities and other overhead, to salaries, is far greater than we take in in our tiered-fee structure. We charge junior enlisted less than senior officers, and it does not come anywhere close to breaking even. But I will have to provide for the record the actual percentage.
 Page 315       PREV PAGE       TOP OF DOC    Segment 2 Of 3  

    [The information follows:]

    The Navy's Child Development Center (CDC) program is supported with 65 percent appropriated funds (APF) and 35 percent non-appropriated funds (parent fees). It should be noted that Navy cares for more infants and toddlers (50 percent of the center program) than the DOD model projected (40 percent). Infant/toddler care is much more expensive and labor intensive than center-based care for 3–5 year old children (i.e., an infant space costs approximately $7,000). The overall direct APF cost per space in the center is approximately $3,900, but Navy is currently working with Commands and Claimants to reduce staffing and operating costs in the center. Navy is also attempting to shift infants and toddlers from center-based care into the more cost effective subsidized Family Child Care program. This will allow Navy to realign potential savings to expand child care and improve the School Age Care program for children ages 6–12. In addition, Navy is looking at other more cost effective methods of delivery such as off-base Family Child Care and the potential of outsourcing on a regional basis.

    Mr. KINGSTON. I think that would be interesting, because I know the money comes out of somewhere. That is the concern I have, and the interest to see that these things are run privately as much as possible.

    Chief HAGAN. I will point out, the money that is budgeted is fenced. There is no discretion by the commanding officer to shift that budget to some other need. That is the law. So those inflexible data points are an important part of my personal stance on child development centers.

 Page 316       PREV PAGE       TOP OF DOC    Segment 2 Of 3  
    Mr. HEFNER. Would the gentleman yield? That does not surprise me that the fees charged for daycare does not meet the expenses. That does not surprise you, does it?

    Chief HAGAN. No, sir.

    Mr. HEFNER. That is what some folks would call a perk around here. I do not imagine all the things that we do such as food and things that are cost effective are perks. One of my real hang-ups is child care. I remember years ago I went out to Fort Hood and these ladies that were spouses had taken an old mess hall to use for a daycare facility. They were trying to redecorate it and fix it up to make a daycare. They were going to run the daycare. It was going to be operated with volunteers.

    I do not have any problem with privatizing things, but you get a different atmosphere when you have got the spouses involved in daycare. This may sound chauvinistic, but the guys, they do not spend a whole lot of time involved in daycare on these bases. It is mostly the spouses, the stay-at-home spouses. Even though some of them work, they go, they drop their children off at the daycare and pick them up as they come home from work.

    I think you had better be careful just like with housing, when you start privatizing child care, in my view.

    Mr. PACKARD. If the gentleman would yield?

    Regulations almost prevent volunteerism. There are very strict and specific regulations that require qualified people, people that have credentials and training and so forth. So it is not something that you can just decide you want to have a volunteer program and put just one of the mothers in there for that two-hour block or whatever. It just does not work that way any more.
 Page 317       PREV PAGE       TOP OF DOC    Segment 2 Of 3  

    Sergeant BENKEN. Sir, let me—the Air Force Council looked at privatization and agreed not to pursue it, and here is the reason why.

    We said it would increase the cost to parents by 20 percent. The law requires matching appropriations. Taxpayers would be affected. Contractor liability, insurance costs add to the cost per space. Some of the other difficulties that you have is contractor's inability to pay and attract Air Force GS–2 to GS–3 equivalents, military spouse employment opportunities diminish, employees are 75 percent military spouses. It is a good PCS employment prospect. Civilian centers, 5 percent accredited, Air Force centers are all accredited. Contractors not available overseas and in remote locations.

    I think all of those are good reasons to keep child development centers. As I would again state, the Air Force sees that as part of the readiness package.

    Mr. KINGSTON. Let me just comment on that. I think those are excuses, not really reasons. I say that respectfully. I think if you wanted to privatize, none of that is insurmountable—none. The suggestion that maybe the military is going to care or love more than the private sector, and that the private sector is going to be less sensitive to children is—and you are not suggesting that, I know.

    I am not taking exception at all to anything but the report. I think that is a cop-out. If you really wanted to privatize—I mean, the cost of daycare is not cheaper. It is just that it is subsidized differently.

 Page 318       PREV PAGE       TOP OF DOC    Segment 2 Of 3  
    Sergeant BENKEN. If you ask any military member, sir, what they would prefer, to have their children on an on-base Child Development Center or downtown, I can almost guarantee you, to the family, they will tell you on base.

    Mr. KINGSTON. I would say on base you still could have some privatization in there, and not necessarily move it out. For example, at King's Bay you have, some of the security is privatized. You have hybrids here and there, but I think in terms of a $5.4 trillion debt, military spending that has been basically flat, missions everywhere. And as you know, you have the strain of your folks going all over the world any time. We have got to look at what is the best, most effective way to handle whatever service, be it food or whatever that can be privatized, we should look into it.

    Chief HAGAN. We use the word privatization here, sir. I think innovation might be a better word, because in-home family caregiver which employs, almost exclusively to the present, spouses of active duty personnel. Currently we are expanding in the Navy that off base to the civilian community where we certify and train and supplement in a modest way the spouse that stays at home and cares for the children up to a certain very reasonable number—I think a maximum of five children. We supplement that to a savings of 90 percent.

    In other words, it costs us 10 percent per child of what it costs to supplement the child development center on base. The figure I have is $4,300 per year per child out of the Navy MWR budget. That is exclusive of MILCON and is paid by the Navy. It costs $430 a year per child when the child is in the in-home family caregiver program.

    I think there are lots of ways we can expand. I am certainly not against child development centers and child care. But I do share the concern I perceive here for it to become a fixed portion of the budget that drives other more important things.
 Page 319       PREV PAGE       TOP OF DOC    Segment 2 Of 3  

    Mr. HEFNER. I do not want to be argumentative here, but I get the feeling that the children are sort of a byproduct of the military. I have four grandkids and am very concerned about children. If we are going to have the type of service that we have got, you are going to have married folks and you are going to have children. You are going to have to look at daycare. Corporations are beginning to look after their employees and all of them have daycare service or this sort of thing or Head Start, and they are involved in these type of programs.

    I like to see the private sector do as much as it can, but this, to me is something that should be a priority. We do not build chapels any more and at King's Bay we built craft centers and car washes and all this stuff. I have been dealing with King's Bay for years. But to me, it seems we have got to focus on child care.

    Mr. KINGSTON. If the gentleman will yield? I think if you look at, say, who cuts the grass, who maintains the trees and so forth, you do not have to have U.S. Navy personnel to cut the grass.

    Mr. HEFNER. We do not do that.

    Mr. KINGSTON. Exactly. You do not have to have U.S. naval personnel to actually be running and changing the diapers—not that they are. But as much as possible, whatever can be privatized is not synonymous with denying the benefit.

    Mr. HEFNER. I am not getting in an argument with the gentleman, but I will just tell you what. I am not in favor of warehousing children or old people; and I happen to be an old person. We have had problems with nursing homes and this sort of thing. When you start privatizing, especially the children, you potentially get into all sorts of problems.
 Page 320       PREV PAGE       TOP OF DOC    Segment 2 Of 3  

    I just have a disagreement. You can disagree without being disagreeable, which I am doing. You can have the last word. It is okay.

    Mr. KINGSTON. This is not about warehousing children. This is not about denying a benefit. This is about, can we do it better? Can we do it less expensively? And are we going to think innovatively, and are we going to continue to look at different ways of spending the dollars?

    The thing that I think is also important on anything like this is what may work on a base might not work on a post, might not work in Georgia but it might work in Alaska. We have to have the flexibility to provide the best quality benefit to that personnel and at the same time be accountable as possible to the taxpayers and aware of the strained resources that DOD already is operating with.

    Sergeant Major HALL. Sir, if I may? My view is that I am all for doing anything if we can do it cheaper and also do it better. I am for doing that.

    I do worry about the family unit though. We spend a lot of time with family units because we are already fighting a perception that our benefit package is eroding very quickly. That is one reason that non-commissioned officers tell me they are getting out of the Army. They are not sure where it is going. We have to be very careful, in my view, with things like that because they will see it as one more benefit that is gone.

    You know, I also have two grandchildren. I have a daughter that is married to a sergeant, an E–5, who just came back from his second tour in Bosnia and leaves in June for his third, and they are doing okay. My daughter has worked in those child care facilities. There is a tremendous training and accreditation process that she had to go through before she could do that. And they pay very well. So not only is she bringing in income, she is taking care of my grandkids and also helping to take care of her Army family at the same time.
 Page 321       PREV PAGE       TOP OF DOC    Segment 2 Of 3  

    But I think it is the benefit package that worries me more than anything else.

    Sergeant BENKEN. If we already have the best, how can we make it better through privatization?

    Mr. KINGSTON. As long as you are satisfied with what your budget allocation is. This is about not your dollar, not my dollar. We have got to continuously look for ways to provide the best quality, the best benefits at the cheapest price.

    Sergeant BENKEN. Yes, sir, and I would say that the American taxpayer needs to recognize the fact that you cannot go cheap on defense. We cannot do that. We have got to step up to a lot of things, and that is tough for the American taxpayer, I realize that. But that is a reality. There are things that are expensive about defense. Modernization is expensive. And the care of feeding of the people that operate and defend this country, there is a price tag to it. I think the American taxpayer has to step up to that. And I am a taxpayer, too.

    Mr. PACKARD. This hearing has gone a little longer than I anticipated and I have not got to my questions. I am only going to ask a couple three small ones. Are you through?

    Mr. KINGSTON. I have got to go meet a school group. Thank you for the time.

 Page 322       PREV PAGE       TOP OF DOC    Segment 2 Of 3  
MORALE AND WELFARE

    Mr. PACKARD. Thank you very much, Mr. Kingston.

    Morale, how would you evaluate the morale generally? Very quick answer from each of you.

    Sergeant Major HALL. Sir, I would say morale is very well. It is fragile, but it is okay right now.

    Mr. PACKARD. Is it improving or going the other way?

    Sergeant Major HALL. Well, the Army has had a hard time over the past year, so I would have to tell you right now it is improving. We have learned a lot about ourselves as a service because of the situation at Aberdeen and other places. Morale took a big dip. It is on the upward slope, but fragile.

    Mr. PACKARD. Sergeant Major?

    Sergeant LEE. Very good, sir. Very good.

    Mr. PACKARD. I would expect that in the Marines.

    Chief HAGAN. Acceptable and steady. I do not have a morale concern about the state of morale. Good leadership is meeting that need, yes, sir.
 Page 323       PREV PAGE       TOP OF DOC    Segment 2 Of 3  

    Sergeant BENKEN. General Ryan and I just went through the desert. We went over there right before the diplomatic conclusion came about. I want to tell you that morale was peaked at that time. I have concerns about morale associated with the desert and the continued rotations over there and not seeing an end in sight. That definitely concerns me.

    Mr. PACKARD. I think our general observation is, we have gone on the bases, is very similar. I think that it is not a huge concern of ours right now. We think that it is good. We recognize that even though budgets are trimmed down significantly that it has not destroyed morale like it was 15 years ago.

OVERALL FUNDING

    That brings up one other point. We are having a decreasing budget. My notes show that, and I mentioned it, that our first quality of life hearing was in 1981, February, and since then—the budget that year, incidentally for the 1982 budget year was $7.3 billion. We moved up in that 17-year period to about $11.3 billion, and we are now down to where the President submitted at $7.8 billion. So it is almost back to the level of 1982.

    We really have not had any significant growth in our budget. Does that give you concern, and do you think that is going to continue to be eroded?

    Sergeant BENKEN. Sir, I think from an American perspective—and I am not an economist and I am not a policymaker, but I would just say that we have to take care of the people that defend this Nation. We have to take care of veterans, and we have to modernize our force. We have to do all those things. To have us shifting things around and trying to figure out how we are going to do that, in my mind is just not the way to do business.
 Page 324       PREV PAGE       TOP OF DOC    Segment 2 Of 3  

    But I understand living within budget constraints and things like that, but there is a price to be paid for that.

    Chief HAGAN. The shrinking budget concerns me the most from the standpoint of readiness, apart from the important issues we have been talking about today. But perhaps more important is that we build more than one ship a year to replace the fleet. Long term as a taxpayer, and I hope an honest patriot, I am concerned about what could be called modernization or recapitalization.

    But whatever size Navy we are going to have, within the window that I believe we are shooting for, we are not replacing it fast enough under current shipbuilding. So a flat budget gives me pause all across the spectrum from the quality of life issues and holding what we have got, to having an adequate force for the needs of the future.

    Sergeant LEE. Sir, we are flat out hurting. You know that. In fact, we need money for improvement, we need money for modernization, we need money for O&M, we need money for more MILCON. Probably $1 billion is what we need this year to give us a reasonable expectation for the year 2000.

    Sergeant Major HALL. Sir, I would tell you—and I am not a mathematician nor a budgeteer—that if you do the math, we are probably $3 billion under-funded in the Army this year. It is going to be a tremendous balancing act to do what we have to do and pay the bills that we must with regard to modernization, readiness, and quality of life.
 Page 325       PREV PAGE       TOP OF DOC    Segment 2 Of 3  

    We just spent a few days at an off-site with all the senior commanders, and just the level of detail that those senior commanders have as they run their major commands and to make just the base operations piece work is extraordinary. It is going to be tough.

FAMILY SEPARATION

    Mr. PACKARD. Let me direct this to the Navy. Are families choosing family separation because of shortages of affordable housing rather than mission requirements? I think it is referred to as geographical bachelors. How widespread is this problem in the Navy?

    Chief HAGAN. I hate the phrase ''geographic bachelor,'' but it is the one we have adopted. It describes a married individual who is separated from their family because of their permanent change of station status. They have chosen not to move their family with them to the new location. It is driven by a variety of factors.

    The lack of adequate family housing on the receiving end where the member is not the highest one on the list is one significant factor. But higher than that is cultural and societal pressures: to keep children in the same schools, for spouse employment, because they have made an investment in a home that they do not want to sell or rent. Those seem to be the leading factors.

    This is a phenomenon that is causing us, in the Navy a great concern because of our sea-shore rotations. I appreciate the fact, sir, that you have read it and considered it an issue worthy of your attention.
 Page 326       PREV PAGE       TOP OF DOC    Segment 2 Of 3  

    Mr. PACKARD. Thank you very much. I would like to just—unless you have some further questions?

    Mr. HEFNER. I just want to make an observation, Mr. Chairman, if it is in order. I have been on this committee for a long time. My dad was in World War I and this reminded me of a little story about this fellow campaigning down in the Ozarks who used to be a member here.

    He went into this store and this old fellow was sitting there with his coveralls on and a cap, and he was chewing tobacco and had a pot-bellied stove. This old man was sitting over at the bar, and he walked over and said, uncle, how old are you? He said, if I live to January 2nd I will be 96-years-old. He said, I guess you have seen a lot of change in your life. And he said, yep, and I have been against every damn one of them. [Laughter.]

    Over the period of time that I have been here there have been some tremendous changes in the mood in the country. Around here we do what the taxpayers will let us know. Occasionally we will put one over on them or something, but there was a mood in the country back in the Reagan years where if you said it was for defense, it did not matter what it was, the public was behind it.

    Now the average guy out there on the street, he does not see that there is any potential threat. The Soviet Union is no longer a superpower. He sees it as just brushfire wars and what have you, and there is really no demand to spend a lot more for defense. So we have to struggle to try to stay where we are.
 Page 327       PREV PAGE       TOP OF DOC    Segment 2 Of 3  

    Yesterday and today we are having hearings on the Defense Subcommittee and we are talking about some systems that we need to do that are under-funded and we are not going to be able to accomplish. So, I just said that little story to just show how things change. God forbid that we would ever have to go to another full-scale war, but I think we have to be prepared.

    In my view you have got to have that rock bottom, what do you call it, grass roots support, and it starts with the families. Especially with the kind of force that we have got now, if you do not keep the family happy, the guy is not going to enjoy going out there and fighting. But it all ties together and we just have to cope with it as best we can. I really sympathize with you. It has been our focus to try to do our best to keep you guys happy on the bases, and also keep the wives and everybody else happy.

    That is a tough challenge, and the chairman has done a tremendous job of fighting for more allocations with the full committee. But it is a tough job. It is one of the reasons I am quitting this year.

    Mr. PACKARD. Certainly one of the men that has done probably more than any single person in trying to bring about these positive changes that he has referred to is the gentleman himself. Mr. Hefner has certainly been a long supporter of the programs that you have fought for, and he has fought for them here. He has made a remarkable contribution and he will be sorely missed. I do not know whether he initiated these hearings with you gentlemen, but certainly he has been through most of them for many, many years, and has listened carefully and made a major contribution.
 Page 328       PREV PAGE       TOP OF DOC    Segment 2 Of 3  

PRIVATIZATION

    I have one comment that I would like to make and then we will close. On the privatization issue, there is not anyone that has pushed for it more than I have and has been a supporter of making that a supplement to the existing—in an effort to close the huge gap that exists on the quality of life facilities. But I think that I can express a concern, as I have heard particularly from the Navy and others on the committee, that we may be making this transition faster and going further than what even we intended.

    I particularly was interested in the Navy's testimony day before yesterday where we are now not looking—and my initial feeling or attitude on PPVs was that it would be a way of building new facilities or refurbishing existing facilities. But now we see where we are literally exchanging and making almost wholesale proposals to exchange current and existing base facilities and transferring it into the private sector, which means that we could very well be depleting existing Government operated and owned facilities much more rapidly than what we anticipated.

    It is an area that I think we are going to have—the committee is going to have to look into to see if that is a direction we want to go. In San Diego alone, the Navy's proposal is to transfer some 9,000 units to the private sector. Those are units that now exist, that this committee has built in the last umpteen years.

    That is something that I had not thought about in terms of privatizing. My concept was that when we need 200 new units at Camp Pendleton or in San Diego or at Norfolk or at Fort Bragg, my concept was that maybe the private sector can help us build those 200 units and retain ownership. But that would not affect the other 5,000 units that we already have there, or whatever.
 Page 329       PREV PAGE       TOP OF DOC    Segment 2 Of 3  

    So we have to look into that and see if we are moving in a direction that is literally transferring existing facilities that we have worked hard to catch up on, to the private sector and leaving ourselves literally without the current existing stock. That is an interesting thing. But I see the Navy moving more aggressively than perhaps the other branch of the services, or at least if I have heard the testimony of the last few days accurately.

    That may be proceeding—we have asked the same question to every one of the Secretaries. That is, are we replacing the current process of MILCON quality of life facilities with privatization? The answer generally has been no, it is a supplement. But I am concerned that maybe in fact it is a replacement process. We will have to look into that carefully and make sure that your concerns are not going to come to fruition, and it is now our concern as well.

    There is a place for privatization; a very important place for privatization. But I think there is also a significant place for what we have been doing for years and years in trying to close the gap. We in some way have to come on this committee to a blend of those, and other innovative ideas that will allow us to do what has to be done to close the gap and still provide the right quality and character of housing and facilities for our men and women.

    You have enhanced that concern, at least in my mind, and I think in the committee's mind. Your testimony has been extremely valuable from a variety of points of view, but that is certainly one of them.

    There will be some questions that we will submit for the record. These are often more detailed questions, or more specific questions on specific bases or specific issues. We would appreciate your responding to that. But if you have no further question or comment?
 Page 330       PREV PAGE       TOP OF DOC    Segment 2 Of 3  

    Mr. HEFNER. Good to see you, gentlemen again.

    Mr. PACKARD. Good luck to each of you. Thank you very much. The hearing is adjourned.

    [CLERK'S NOTE.—Questions for the record submitted by Congressman Porter]

NAVAL TRAINING COMMAND GREAT LAKES

    Question. As you know, Great Lakes was forced to close its only live fire range this past year due to extreme deterioration of that facility. Could you please comment on the effect this has had on the training of recruits?

    Answer. The live fire range at Naval Training Command Great Lakes had to be closed due to environmental concerns centered on high lead levels throughout the facility. Continued weapons training for recruits has been limited to high tech, laser weapons simulations. This technology enables limited training and scoring of weapons accuracy for the qualifications as ''marksmen,'' ''sharpshooter,'' and ''expert.'' However, this does not enable recruits to even have a familiarization firing of an actual weapon. Current budget planning is in place for a new live fire range in two years. In the interim, the Commander of NTC Great Lakes estimates 200,000 Sailors will complete their basic training without any live fire experience and report to the fleet where they must be able to qualify on weapons to stand security watches. I am concerned that we are leaving too much of the basic training to be done after Sailors reach their initial duty station. While training at NTC Great Lakes only sets the foundation for Sailors, it must include the basic training necessary for them to meet the needs of the fleet. Completion of the new live fire range is critical and will enable us to better meet this goal and to make NTC Great Lakes a first class facility.
 Page 331       PREV PAGE       TOP OF DOC    Segment 2 Of 3  

    Question. In order to ensure that our recruits continue to meet the standards set for physical fitness in the Navy, it is vital that Great Lakes have an adequate number of facilities to accommodate all of its recruits. Do you perceive that these standards can be met with existing facilities for physical training?

    Answer. In my studied opinion, the fitness facilities currently in place at Naval Training Center Great Lakes are inadequate to meet the minimum requirements to train our recruits. Many of the facilities date back to World War II and are in great need of updating. Due to the often harsh winter conditions at Great Lakes, much of the physical fitness training must be accomplished indoors. These drill halls are very old and require maintenance that is fast approaching the cost effectiveness point. Swimming is also an important survival skill and fitness goal for Sailors. We currently have one training pool in place with another programmed but not yet completed. Building a state of the art swimming facility offers the opportunity to incorporate increased fitness programs with training that simulates the conditions Sailors will face at sea. Our Navy is making every effort to increase the fitness level, from the most junior recruit to the most senior Admiral. To set the foundation for this effort, we must start with a solid fitness program from the very first day recruits enter Great Lakes. This is all a very fundamental part of making Great Lakes a first class facility.

    Question. My understanding is that, at most, there is only one confidence course available for use by trainees at Great Lakes. Could you tell us why it is important to have trainee access to confidence courses, and the proper number that should be available at Great Lakes given the number of trainees?

 Page 332       PREV PAGE       TOP OF DOC    Segment 2 Of 3  
    Answer. A single ''confidence'' or obstacle course was built as a self help project. This course incorporates Navy specific evolutions into a challenge/obstacle course that is meant to be used for fitness, team building, and to build confidence. The current course has a very limited capacity and only offers the opportunity for recruits to complete this course once or, at most, twice during their nine weeks of basic training. Building a network of confidence/challenge courses would enable us to make this a regular part of recruits training. A total five to seven such courses would allow recruits to train on the confidence course three or more times weekly. This network would allow us to utilize the courses for their intended fitness, confidence and team building objectives and would constitute a significant step forward in preparing recruits to meet the rigors of service in the fleet.

    Question. I submitted questions for the record during Under Secretary of Defense William Lynn's testimony before this subcommittee regarding Impact Aid. I would be very interested in having your perspective as well on how quality of life in the military is affected by the Department of Defense's repeated neglect of this program so vital to maintaining the schools attended by children of military personnel.

    Answer. The local public school system is an important consideration for many Sailors in the selection of a new duty station and the selection of where they live in a current duty station. Sailors even choose to leave their families at a previous duty station and serve in a separated from family (geographic bachelor) status rather than taking their family to a new duty station where the schools are deemed to be inadequate. Others are commuting great distances to ensure their children are enrolled in quality schools. We must take any necessary measure to ensure a minimum quality standard is maintained for the education of the children of our Sailors. In regions where children of Sailors make up a large percentage of the enrollment of schools, financial support is provided to ensure the quality of the education is up to that minimum standard. We owe Sailors a reasonable, consistent quality in the education system which serves them across their career. This is an important retention issue of great importance to the growing number of Sailors, who like parents in all walks of life, are greatly concerned about their children's education.
 Page 333       PREV PAGE       TOP OF DOC    Segment 2 Of 3  

    [CLERK'S NOTE.—end of questions for the record submitted by Congressman Porter.]
Wednesday, February 25, 1998.

DEPARTMENT OF THE ARMY

WITNESSES

ALMA B. MOORE, ACTING ASSISTANT SECRETARY OF THE ARMY

PAUL JOHNSON, DEPUTY ASSISTANT SECRETARY, ARMY INSTALLATIONS AND HOUSING

KATHERINE CONDON, ASSISTANT SECRETARY FOR RESOURCES AND MILITARY SUPPORT

MAJOR GENERAL DAVID WHALEY, ASSISTANT CHIEF OF STAFF, INSTALLATION MANAGEMENT

BRIGADIER GENERAL JAMES HELMLY, DEPUTY CHIEF, ARMY RESERVE

COLONEL PROMOTABLE MICHAEL SQUIER, DEPUTY DIRECTOR, ARMY NATIONAL GUARD

Statement of the Chairman

    Mr. PACKARD. It is 9:30 and Mr. Hefner, I understand, is on his way. And we will go ahead. We have Mr. Edwards with us representing the other side, and as soon as Mr. Hefner arrives we will hear from him.
 Page 334       PREV PAGE       TOP OF DOC    Segment 2 Of 3  

    We are extremely grateful to welcome Secretary Alma Moore with us today. She will be our principal witness. We welcome all of those who accompany you. We appreciate you being here. We are aware that this is your first time before this committee as a witness, but it is certainly not your first time before these kinds of committees. You have spent a lifetime on this side of the aisle, this side of the table. We appreciate your wonderful experience over the years and are looking forward to your testimony.

    We want to congratulate you personally for staying the course in barracks construction, particularly on foreign locations. There is no constituency that supports foreign construction. This subcommittee has to stand for that, as well as, of course, the leadership in our military. But there are a lot of people that support our bases here domestically here in the Congress, but there is no one overseas that votes for us to speak of and so we tend to neglect that. And I appreciate you staying the course there.

    I have read all of your testimony. It kept me up late last night. And I don't want you to read it to me again.

    Ms. MOORE. Thank you, sir.

    Mr. PACKARD. We would love to have you highlight as much as you would like, and you can read any portion you would like. It was very organized and very well done. I want to say that. You have good experience, and certainly that came through in your testimony. I did read it all carefully. I have underlined some of it. I may have some questions, direct questions relating to your testimony.
 Page 335       PREV PAGE       TOP OF DOC    Segment 2 Of 3  

    I would like to clarify one thing. I know that your budget was submitted on the basis that the exercise of the line-item veto in our last year's bill would stand. And I am sure your budget reflects that. As you may know, the House has not approved the line-item vetoes of last year's bill. The Senate will take that up and vote on it probably today. We fully expect that they will pass, as the House did, sufficient numbers to override. So that will have to be taken into account as we reflect upon your budget.

    I know that it appears that there is an increase in the military construction budget, but with that consideration and other factors, we recognize it and see it as a decrease. I am very concerned, as I expressed in the last hearing, of the overall decrease in military construction budget. The Army has decreased it from our figuring approximately $48 million, which is a seven percent decrease when you take into account the line-item veto issue as well as adjusting for the chemical demilitarization program.

    But nevertheless, we appreciate the budget that you have submitted. We will do our best to make it right as far as what we feel is important for men and women in the services and their support facilities.

    We are very grateful, again, to have you here. We hope that you will be able to highlight the issues that are important in your testimony. I might announce to the members that are here in five minutes we have got a full house.

    Mr. EDWARDS. Let us vote.

 Page 336       PREV PAGE       TOP OF DOC    Segment 2 Of 3  
    Mr. PACKARD. I guess Chet Edwards would offer a motion for a new chairman and it would probably carry and it would be him.

    We are very pleased to—incidentally, I would like to announce, and I may do it later as others arrive, that our next hearing will be tomorrow at this same time, 9:30. The subject will be outside witnesses. That is always an interesting hearing, and we hope that we will have a good attendance by the subcommittee members. I am extremely grateful to have the members of the subcommittee here. Bill Hefner used to be the chairman of this subcommittee and certainly has served on this committee for a long time and is one of the experts on military construction issues. And I would like to hear from him before we hear from our secretary.

    Mr. HEFNER. Well, I will be very brief, and it is certainly good to see you again. You have been a fixture around this part of the world for a long, long time, and probably know more about how the military works than everybody combined on this panel. And I would just like to pay you a compliment for all those years you gave tremendous help to myself, our staff and our constituents. You serve the people of this country very, very well, and we certainly think that they couldn't have picked a better person. It would have been all right with us if you would have been the Secretary, not the Assistant Secretary.

    Be that as it may, we are happy to have you here this morning. And I have some questions, just a couple things, but I will put them in for the record. We are glad to have you this morning. I am looking forward to you proceeding.

    Mr. PACKARD. Thank you, Mr. Hefner. At the conclusion of her testimony we will have, of course, time for questions. And so you may wish to ask them at that time. Again, we welcome you here, Ms. Moore, and look forward to your testimony. You may proceed as you wish.
 Page 337       PREV PAGE       TOP OF DOC    Segment 2 Of 3  

Opening Statement of Acting Assistant Secretary Alma B. Moore

    Ms. MOORE. Thank you, sir. Mr. Chairman, Mr. Hefner, distinguished members of the subcommittee, it truly is an honor to be here today to discuss the Army's military construction budget requests for Fiscal Year '99. I have to confess that it feels a little strange to be on this side of the table. I think it is easier to write those questions than it is to answer them.

    Joining me today representing the total Army is Mr. Paul Johnson, our Deputy Assistant Secretary of the Army for Installations and Housing; Katherine Condon, Deputy Assistant Secretary for Resources and Military Support; Major General David Whaley, Assistant Chief of Staff for Installation Management; Brigadier General James Helmly, Deputy Chief of the Army Reserve; and Colonel Promotable Michael Squier, Deputy Director, Army National Guard. Colonel Squier is going to be promoted on Friday.

    Mr. PACKARD. Congratulations.

    Colonel SQUIER. Thank you.

    Ms. MOORE. Our combined written statement provides in-depth details of the '99 construction budget. With your permission, sir, I would like to submit it for the record.

    Mr. PACKARD. Of course, without objection.
 Page 338       PREV PAGE       TOP OF DOC    Segment 2 Of 3  

    [The prepared statement of Acting Assistant Secretary Alma B. Moore follows:]
    "The Official Committee record contains additional material here."

    Ms. MOORE. And just highlight some of the areas, as you suggested. The FY '99 military construction budget's main focus is the whole barracks renewal program for which this committee has been so supportive over the years. It represents our efforts to provide our single soldiers with a quality living environment, not just a place to live. The one plus one program includes such things as personal privacy, larger rooms, closets, upgraded day rooms and additional parking.

    The budget before you includes 12 projects totaling $307 million to keep us on our 2012 goal for completing modernization of all barracks in the United States and overseas. We hope to complete the barracks program in the United States by the year 2008, in Europe by 2010 and in Korea by 2012.

    Next I want to briefly discuss the Army's strategy to meet family housing needs through maintenance, divestiture and privatization. In FY '99 we will renovate 514 existing units and replace 560 units. This budget request also reflects the demolition of 350 houses that do not meet standards and would be too costly to repair. But these efforts do not come close to fixing the Army's housing problems.

    The Capital Venture Initiative, also known as the Military Housing Privatization Initiative, will allow the Army to leverage housing appropriated funds and Army-owned assets, such as land, to encourage the private sector to own, operate, manage, repair, improve and construct family housing.
 Page 339       PREV PAGE       TOP OF DOC    Segment 2 Of 3  

    The Army's first CVI project is, as I am sure you know, Fort Carson, Colorado. We have notified Congress and we are now in the 30-day notification period. If Congress has no objection, we would probably award that contract sometime about mid-March. Our plan is to use these authorities wherever feasible and economical in the United States. We will apply Fort Carson lessons learned to develop 26 additional projects if all goes well.

    Your support of this effort is essential to permit us to leverage housing resources and provide quality housing to all of our soldiers and their families in a much shorter period of time.

    This year's military construction budget was built on the Army's one team, one fight, one future concept. We made a conscientious effort to balance our resources among all components, the active Army, the National Guard and the Army Reserve. I have already talked about whole barracks renewal and housing, but I want you to know that our focus is also in Reserve centers and National Guard readiness centers. The Guard and Reserves were at the table throughout the budget building process. And I think they will tell you they believe they received a fair share of constrained resources.

    Readiness remains the number one priority for the Army, and I would like to highlight those projects that enhance our soldiers' readiness capabilities. We continue to provide funding for the Strategic Mobility Program to ensure our soldiers can employ their equipment as expeditiously as possible. This budget also completes the construction of close combat training facilities.

 Page 340       PREV PAGE       TOP OF DOC    Segment 2 Of 3  
    The greatest challenge, Mr. Chairman, facing the total Army in this budget, and as we move toward the future, is balancing today's readiness and quality of life with tomorrow's modernization requirements within available resources. This budget request continues our goal to provide better living conditions for the families and the single soldier while also providing the total Army with valuable facilities to enhance readiness.

    Mr. Chairman, this concludes my statement. All the people behind me are experts in their field, and I hope all of us together can answer your questions.

    Mr. PACKARD. Thank you very much, Madam Secretary. I didn't intend for you to be so brief, but again I am hoping that the members of the committee have read your testimony, and it is very complete and thorough. I appreciate very much your comments.

    I want to welcome Mr. Hoyer back to the subcommittee. Even though he has now moved on, he has shown an interest in coming back. He is welcome at any time.

    Mr. HOYER. Thank you, sir. I am not Mike Parker.

    Mr. PACKARD. No comment on that.

    Mr. HOYER. I want to particularly be here with Alma Moore. As you know, when Secretary Moore was here, she did an incredible job with the House of Representatives.

    Mr. PACKARD. That she did. That she did.
 Page 341       PREV PAGE       TOP OF DOC    Segment 2 Of 3  

    Mr. HOYER. She is one of the best people I think we have in the public service. Thank you, sir.

    Mr. PACKARD. Thank you very much.

    Mr. HEFNER. We should have to sit there once.

    Mr. PACKARD. Before you arrived, I thought he did. We will follow the rules that we set up initially for this subcommittee in terms of questions. We will hear from the ranking member first on questions. Then as they have arrived on a bipartisan basis we will recognize members of the subcommittee. I will probably defer my questions until later, and so I will just ask Mr. Hefner if he would like to start with the questions of our witness.

    Mr. HEFNER. Thank you, Chairman. I am going to defer. I may have some questions, but I am interested to hear the concerns of the other members, because they have deep concerns about things in their own district. I will wait till the end.

    Mr. PACKARD. That will be just fine.

    Mr. HEFNER. I would just like to mention once again it is such a pleasure to have you here. I know that this is going to really make it much easier for us to have someone in your position who knows how both sides work, both the legislative and the proposed and the proposee. So at this time, Mr. Chairman, I will just yield back my time and reserve my time for some time later.
 Page 342       PREV PAGE       TOP OF DOC    Segment 2 Of 3  

FAMILY HOUSING PRIVATIZATION

    Mr. PACKARD. That would be fine. That will be just fine. And Mr. Edwards will be the first, but let me first before Mr. Edwards is recognized just ask if you would submit for the record the 26 projects that were mentioned that will follow Fort Carson in terms of your privatization efforts. We are very interested in that. I think you are aware that the subcommittee has been very supportive of that effort. And I think we would like to know what those 26 follow-up programs are. We think that Fort Carson very well could be a prototype, and it is an opportunity, as you have stated, for you to work out the bugs and to learn from that experience. We hope that it will work over and over again, if it is successful.

    Mr. Edwards.

    Mr. EDWARDS. Thank you, Mr. Chairman. Just a couple points. I want to begin by commending you, Mr. Chairman and Mr. Hefner. I never thought I would see a vote on a spending proposal in the House get more votes than the highway spending, and the vote on the MilCon line-item veto override, I think, is a real compliment to you putting together a bipartisan program. Despite some critics' talk of Congressional add-ons being pork barrel, it showed that this Congress is committed to seeing that we have a better quality of life for our military families.

    I just can't do any better than what Mr. Hefner and Mr. Hoyer said about Alma. What a real pleasure to see you back after so many years of service. And I think all of us would like to leave this institution knowing we made a difference. I don't know about the rest of us. I know in your case you have made a difference for military families. And I want to say thank you.
 Page 343       PREV PAGE       TOP OF DOC    Segment 2 Of 3  

    Ms. MOORE. Thank you, sir.

MILCON BUDGET REDUCTIONS

    Mr. EDWARDS. Just a couple of questions if I could, Alma. Is the Army reduction of seven percent in the MilCon budget equivalent to what the Navy and Air Force will face or will the Army take a bigger or smaller dip than the other services?

    Ms. MOORE. General Whaley, can you respond?

    General WHALEY. In terms of MCA, sir?

    Mr. EDWARDS. Yes.

    General WHALEY. I think my understanding is that—first, I have not done a comparative analysis. The Air Force may have. Our entire budget is constrained. I think we have taken a balanced approach to the MCA, and in fact on the barracks side, you will find that we are getting better in terms of lowering price for the same quality, as it is indeed a standard design that has been beautifully supported by you. And again, let me add I think the total Army thanks you for your support in the House.

    Mr. EDWARDS. I appreciate that answer. And if someone could follow up just with a written answer, I would just like to see how the Army compares relative to other services.
 Page 344       PREV PAGE       TOP OF DOC    Segment 2 Of 3  

    Ms. MOORE. It might be a little confusing because the Army has acquired the budget for the chemical demil program. We also have several—well, two projects that were on the veto list that are in this year's budget also. The Chairman mentioned that. So——

    Mr. EDWARDS. You would have to subtract from that end.

    Ms. MOORE. Yes, we will try and figure that out for you and provide a detailed answer for the record.

    [The information follows:]

SERVICE MILITARY CONSTRUCTION BUDGET COMPARISONS ACTIVE COMPONENTS

    The following information compares the fiscal year 1999 (FY99) President's Budget Request with Congress's FY98 Military Construction Appropriation, by Service. Fiscal year 1998 includes the Congressional Reductions, but is not reduced by the amount of the President's Line Item Veto. Also, for comparison purposes, the $125 million for chemical demilitarization is shown separately.

Table 39



FAMILY HOUSING PRIVATIZATION—FORT CARSON

    Mr. EDWARDS. Okay. Secondly, I want to commend you on your innovative efforts in privatization. Fort Carson has been working on a privatization program for housing. Fort Hood, which you know I represent, is now in the midst of trying to do that. Brief thoughts, a little bit more detail as to what the status of the privatization efforts—what the benefits are. It is my understanding you shortened the cycle for upgrading the family housing from, I don't know——
 Page 345       PREV PAGE       TOP OF DOC    Segment 2 Of 3  

    Ms. MOORE. 130 years.

    Mr. EDWARDS [continuing]. Way out in the future. Yes, 130 years to the real world.

    Ms. MOORE. That is the most important benefit. Our soldiers and their families would get quality housing much sooner. I think at Fort Carson privatization will provide housing within 5 years. We could never afford to do that in the Defense budget. As you know, the military construction budget does not provide enough funds to reduce the timeline. That is the main benefit from CVI. We have taken a lot of time to work with Fort Carson because the program is new to all of us and new to the private sector as well. We notified the Congress, as I said, on February 10, and there is a 30-day wait. At the end of that 30 days, we intend—if there are no objections from the Congress to award the contract. And soon after that, we have a conference scheduled to go over lessons learned throughout this process up until now. We intend to use those lessons learned as we go forward to Fort Hood and other locations.

    Mr. EDWARDS. With the limited budget that we have to deal with, what an innovative way to really expand and more rapidly upgrade our housing.

    Final question on the issue of privatization. Have we gotten over the hurdle that OMB constantly threw at us in terms of scoring between a lease payment over 20 or 30 years and the first year?

    Ms. MOORE. They approved it, obviously. General Whaley?
 Page 346       PREV PAGE       TOP OF DOC    Segment 2 Of 3  

    General WHALEY. We have a working agreement and understanding of how it can be scored. Sir, we are—typically we are, I say, joined at the hip, and as we go through each installation—as you know, each installation will be a different equation because it will be based on the total calculation, its own economic conditions, numbers of houses to be revitalized and number to be built and what economic indicators are in the area. I think we have cleared the path using Fort Carson as the spearhead to know what we need to do. The only piece, and it will vary from each installation, is exactly how much will be required. And that will vary from installation to installation.

    Mr. EDWARDS. Very good. Thank you.

SCORING

    Mr. PACKARD. Would the gentleman yield. I want to make sure we are clear on that now. Are you saying, then that the scoring will be different than what was initially anticipated?

    General WHALEY. Sir, it will vary—my understanding is that it will vary from installation to installation depending upon the economics of each installation and the requirement of each installation.

    Mr. PACKARD. How will you manage budgeting on that basis, then? When and how will you know actually how it is going to be scored?

 Page 347       PREV PAGE       TOP OF DOC    Segment 2 Of 3  
    General WHALEY. Sir, we will not know until we present that to them. Now again, sir, as we go through this, for instance we are working Fort Hood as we speak, not to award a contract but the preliminaries of going through that. When we know the number of quarters to be revitalized, the number of quarters to be built, which we do at Fort Hood, then we can start bringing that equation to OMB through the OMB Secretary. And that is why we have 26 that are working, so we can get out ahead of the scoring piece and get at least a very good estimate of what that scoring is going to be.

    Mr. PACKARD. Thank you. You can have your time back.

    Mr. EDWARDS. Thank you, Mr. Chairman. Madam Secretary, it is a pleasure to call you Madam Secretary. Welcome back.

    Ms. MOORE. Thank you, sir.

    Mr. PACKARD. We are going to go back and forth. Mr. Kingston, on the right.

GUARD CONSTRUCTION

    Mr. KINGSTON. Thank you, Mr. Chairman. Ms. Moore, I need a little lesson here. It is my understanding on Guard construction that some is an add-on and some comes through the Pentagon. Which is the difference?

    Ms. MOORE. In the add-ons?
 Page 348       PREV PAGE       TOP OF DOC    Segment 2 Of 3  

    Mr. KINGSTON. Yes.

    Ms. MOORE. The Guard projects that come through the Pentagon in the budget request, of course, you know it is obvious what that is. And add-ons are what you gentlemen help us with from time to time.

    Mr. KINGSTON. No, I know that, but there is—I understand from our General that there are certain things that have to be added on that don't ever come from the Pentagon budget. Is that not correct? So the Pentagon can request any kind of Guard funding in all categories?

    Ms. MOORE. Yes. Any requirement for the Guard——

    Mr. KINGSTON. Okay, so if our——

    Ms. MOORE [continuing]. Comes through the construction budget.

    Mr. KINGSTON. Our General has come to me with items which he has told me will not be requested by the Pentagon because they cannot be requested by the Pentagon.

GUARD CONSTRUCTION PRIORITIES

    Ms. MOORE. What he probably means is that the priority is not high enough for certain projects to make it up into the budget request in the year, near-term year.
 Page 349       PREV PAGE       TOP OF DOC    Segment 2 Of 3  

    Mr. KINGSTON. That is not the case as I understand it. It was that there were certain categories——

    Ms. MOORE. Colonel Squier, can you help here?

    Colonel SQUIER. Let me attempt to address your question. We have a process that we have instituted and developed a six-year intensive program that is based on legitimacy of requirements, based on priority, accuracy of facilities and the priorities of the state. That is what we use as a budgeting process. What has happened in the past is that we have had Congressional members put add-ons to that which we obviously report, but it has to fall under the categories of our six-year defense plan in order for us to move it up in the process. You are right, your General is probably telling you that because of this prioritization system we can't get it moved up. Well, that is because we have a large number of those that we need to deal with that are—the requirements that we have, about $400 million a year, it is pretty hard to do that. We would consider all of his requirements and attempt to move them up in priority based on those three factors.

    Mr. KINGSTON. I can see, you know, moving something up in line and, you know, just sort of having an honest disagreement with the Pentagon what should be priority and what should not be.

    Colonel SQUIER. Sure.

    Mr. KINGSTON. But I was under the impression that there were certain things that just did not get requested by the Pentagon as a matter of policy.
 Page 350       PREV PAGE       TOP OF DOC    Segment 2 Of 3  

    Colonel SQUIER. We have a long-term construction program that is a part of that. It is all valued in those three factors, readiness support requirements and providing for adequacy of the facilities within the states and then priorities of the facilities in the state. We have a factor applying those and we mathematically figure priority for all of our——

    Mr. KINGSTON. Is the Army Guard different than the Army in the ranking of that priority or the way these things are categorized?

    Colonel SQUIER. No, we are working with the Army in establishing those rules. And it is based on readiness requirements, what the force requires. That is priority one. But it is not the only factor. The state does have input with that. With the adequacy of facility, one and two, their priority is also of value.

    Mr. KINGSTON. We have some Army Guard projects that we want to talk to you about that aren't on your list. We are concerned in terms of why. That is why I am asking the question.

    Colonel SQUIER. Sure.

    Mr. KINGSTON. Mr. Chairman, I yield back.

    Mr. PACKARD. Thank you. Thank you very much, Mr. Kingston. Mr. Olver.
 Page 351       PREV PAGE       TOP OF DOC    Segment 2 Of 3  

PRIORITIZATION

    Mr. OLVER. Thank you very much, Mr. Chairman. I would like to add on—to dwell for awhile on the same topic that my colleague from Georgia has just talked about. And let me just start by saying this program that you have of prioritization, I think you first laid that out for us, if I remember—maybe you can confirm that, Madam Secretary or General Squier, from our 1997 March hearing data, the executive summary of the hearing on that date, laying out this program. Is this the first year that this has been applied thereby, the assessment summary that you have described, General Squier?

    Colonel SQUIER. This is actually beginning the second year. We started the process in March of '97 based on your guidance to us, the Congress' guidance to develop a legitimate process where we can manage long-term requirements of the Army National Guard. This is really our second generation of that.

    Mr. OLVER. But the FYDP for last year would not have been written with that in—based upon that program.

    Colonel SQUIER. That is correct.

    Ms. MOORE. That is correct.

    Mr. OLVER. This is the first FYDP that would be true based upon that.
 Page 352       PREV PAGE       TOP OF DOC    Segment 2 Of 3  

    Colonel SQUIER. It would be the true result of that.

    Mr. OLVER. So the criteria that you have laid out, you say readiness, my sense of that one, if I may put my own words to it, is which of the units of the Guard, the Army Guard, would be first to be called up to support active units in some sort of a national security situation. So the order of priority would be most likely to be called up versus less likely.

    Colonel SQUIER. Yes, sir.

    Mr. OLVER. So readiness is roughly that factor. And the adequacy is fairly clearly whether the facilities that people are working with are adequate for the job that needs to be done. And then the state priority is also reasonably clear. I think the readiness one is the one that might have some uncertainty. Do you think that that provides you with an objective assessment of the program as it ought to be?

    Colonel SQUIER. Sir, I believe that it does provide an objective assessment based on wartime requirements of our service. It does have some limitations, obviously, because we can't meet all of the expectations of the states out there, but we have to have our number one priority as our federal mission. And priorities to meet their requirements has to drive our prioritization system. So I think we have made some real progress in getting—it probably needs some further work, and we are working at that, trying to work with the states. The bottom line is the requirements the service's put on us to be able to be more ready has to be our priority of effort.
 Page 353       PREV PAGE       TOP OF DOC    Segment 2 Of 3  

    Mr. OLVER. Well, I think one would expect any new system to require some fine tuning, perhaps, over a period of time in it, but I am glad to hear that you view it as a reasonably good hit of the target of objectivity here.

    Mr. KINGSTON. Would the gentleman yield?

    Mr. OLVER. Sure.

    Mr. KINGSTON. I wanted to put his question in a specific example. If a Guard unit is on a post with an infantry division that has been deployed, then obviously they are going to be a high priority for readiness, correct, if they are particularly maintaining equipment that that infantry division may be using in Kuwait?

    Colonel SQUIER. Not necessarily. The missioning that we are talking about is the missions that are assigned to us by the Army. That unit in all probability is—by what we just defined is probably part of our force pool, high priority support package that we provide to the Army. I might have to get some specifics on what unit you are talking about.

    Mr. KINGSTON. I would—and I am on Mr. Olver's time, but I wanted to—I would like to follow this up.

    Colonel SQUIER. Sure.

 Page 354       PREV PAGE       TOP OF DOC    Segment 2 Of 3  
    Mr. KINGSTON. Because there is a project that is not on your priority list which I would argue should be on the priority list.

    Colonel SQUIER. I would welcome the opportunity to look at that.

    [The information follows:]

ARMY NATIONAL GUARD UNITS ON ACTIVE DUTY POSTS

    The Army National Guard (ARNG) force structure is assigned by the Deputy Chief of Staff of Operations of the Army. While ARNG units may be located on an Active Duty installation, nearly all are located in communities throughout the States and Territories. The missions assigned to ARNG units do not reflect where these units reside. ARNG units on an Active Duty installation are only tenants of the post, and do not necessarily have a warfighting alignment with the host units on the installation. ARNG unit priorities for funding are established on the basis of their mission, not on their location.

    Mr. KINGSTON. Because of the Middle East situation right now, it has changed. What you probably formulated two months ago, you know, dramatically could have changed in the last couple of weeks.

    Colonel SQUIER. Well, I can answer that question without telling you—for the record I can give you a good answer. I will tell you that we are using units to meet the needs in Bosnia right now which are not priority forces, because we have kept priority forces for the full-scale war. That is the problem. I would be happy to define that issue.
 Page 355       PREV PAGE       TOP OF DOC    Segment 2 Of 3  

    [The information follows:]

UNITS IN SUPPORT OF BOSNIA

    Most of the units that we have deployed to Bosnia have been our lower tier units that are considered part of the nation's strategic reserve. For the most part, the units that are our first and second tier units have not been used for the Presidential Selected Reserve Callup in Bosnia. It is our lower tier units that have the required skills necessary in that theater. Currently, we have a company of the 29th Infantry Division that is in Bosnia. Because it is a divisional unit, it is low on the tiering. However, this unit has the skills required for the mission it has been assigned to carry out.

    Mr. KINGSTON. Okay, thank you.

    Mr. PACKARD. Would the gentleman yield for just a moment?

    Mr. OLVER. Yes, sir.

FYDP PRIORITIES

    Mr. PACKARD. You have budgeted about $50 million for Guard activities in this submission. That obviously will not fund all of your FYDP priorities. Have you selected for funding those that are at the top of the priority list within the FYDP down to where the $50 million would be used or are there—are you dipped down over, passing over some that are a higher priority or have you prioritized them within the FYDP?
 Page 356       PREV PAGE       TOP OF DOC    Segment 2 Of 3  

    Colonel SQUIER. We have tried—we have stayed within the FYDP established. We are getting on that FYDP within the limitations of the resources that we have, so the $50 million, approximately $48 million that we have asked for this year will then take care of those in priority the best we can. Certainly it will not cover all of the requirements.

    Mr. PACKARD. I think the subcommittee is aware that last year—virtually all—I think maybe all of the add-ons by this committee were in the FYDP as well. We did not go outside of the FYDP to fund add-ons.

    Colonel SQUIER. Right.

    Mr. OLVER. Are you going to give me some more time since everybody is taking my time?

    Mr. PACKARD. I sure will. You have as much as you like.

CRITERIA ON READINESS

    Mr. OLVER. I was going to say that virtually every member of this committee would like to have an objective set of criteria on which we could cite the readiness according to units and the adequacy of the facilities that are there and some relationship to state priority ought to be honored. With your indulgence, Mr. Chairman, I would like to follow up and look at the FYDP with the actual scoring as it turns out at the moment. And I spent an hour one night while I was watching a basketball game and scored everything that was on the FYDP, what is a very clear numerical arrangement. And everything that is in the FYDP, including what was in the FYDP for this past year, if I may pass it out—we have a number of copies here that we can give to at least the front row and Madam Secretary and other members. If there are others that want them, I will be happy to make them available. I would like to just draw a couple of rather interesting inferences.
 Page 357       PREV PAGE       TOP OF DOC    Segment 2 Of 3  

    [The information follows:]
    "The Official Committee record contains additional material here."

    If you look at the 1998, the first page there, what was on the '98 FYDP and those scores and then what was added by the committee and where we took them from different FYDP years, except for the last three, which were actually taken not from the FYDP at all, if you look at the averages they are roughly the same. I was interested to note that the averages of those that were added by the committee actually were essentially the same. They are certainly statistically the same, I think, as the ones that had been on the original 1998 FYDP.

    Well, then the rest of this is year by year of the FYDP score of the remaining five years, '99 through the year 2003. And interesting, if you note the projects and what year they were last year on the FYDP as opposed to this year, you can see the left column is where they appeared on the 1998 FYDP and now it is the 1999 FYDP, the ones at the bottom of the page, of each group in turn. The average score ends up dropping very dramatically for the year 2000, the year 2001, and then it begins to come up also dramatically in the year 2003 on the basis of your objective scoring mechanism.

    Now I would like to just call to attention, because this is something that we could spend a lot of time on, I don't think you will find any numerical errors in my scoring because they are very clearly objective numerical scorings and very easily done. But I would like to call your attention to just a couple of things. In the year 2003, for instance, there are several, six items that are brought forward from the old infrastructure list, which is a long list of hundreds of projects. And those that are brought forward from the infrastructure list are very high scoring. They are in the highest—they tend to be in the highest priority of readiness and unsuitability of the facility and in terms of state's priority.
 Page 358       PREV PAGE       TOP OF DOC    Segment 2 Of 3  

    If you will note that the highest score on any of these, and it is anywhere in the whole system of any one of the projects that is out there, is a 943. The 943s appear in 2003. And interestingly enough, several of them were in what we did last year. There is not a single 943 in the years '99, 2000, 2001 and 2002, not a single one.

    There are several projects on the—further projects on the infrastructure list that are at a score of 943 that are way down in the out years that don't even appear on the infrastructure list—excuse me, on the FYDP. There are at least the first dozen projects on an infrastructure list, continuing infrastructure list, first dozen on that list have higher scores than any one of the items that is on the Fiscal Year 2000 or the Fiscal Year 2001 proposals here, the ongoing Fiscal '99 programs. There are three projects—maybe I just said this. There are three projects with 943s, the same highest score that appears, that don't appear on any of the FYDP years at all.

    And there is one other interesting thing. Remember, I come from New England and I was interested in checking the geographical base here. And I note that in this list of FYDP projects through the year 2003, forgetting 1998, because they are already done and settled, but there are 33 projects from '99 through the year 2003. Not one of them appears in Pennsylvania, New York or any one of the New England states, which is an area of the country of nearly 20 percent of the total population, which has not a single project appearing on this FYDP. And actually, there are at least five projects in those eight states that have higher scores than most of the projects on these '99, 2000, 2001 and 2002, but not by any means higher than the 2003 projects.

 Page 359       PREV PAGE       TOP OF DOC    Segment 2 Of 3  
    So I am curious. It seems to me that, yes, this looks like a good objective kind of scoring mechanism. You say that it is an objective scoring mechanism and a good tool and so forth, but I think you have stepped on it in the way it has been used.

    Ms. MOORE. Sir, I don't know what the rule is. I will let Colonel Squier address that in a minute, but I can think of two or three reasons why those projects might be in 2003. For one thing, Congress may have procured equipment in another subcommittee perhaps, that won't come out of procurement until those years. There are a number of similar situations that could impact this list that don't show readily from the criteria that are being used.

    Colonel SQUIER. Sir, if I may, I think the first thing I would like to say is that you have asked a very complex question, and I think we owe you a good answer. I would like to take it for the record and give you an answer, but I will attempt to give you some thoughts on it.

    Number one, this program is new. It is a good step in the right direction. It needs to be fine tuned. We certainly understand that. There were some projects that were already on what was an established FYDP that we had to leave on there because of what Ms. Moore just talked about. We had already programmed for them. We already had equipment for them and we needed to keep them in the process. And third, I will say, we are looking for ideas. There are some other factors that we need to consider like geographical distribution throughout the state. Maybe that needs to be valued.

    I will tell you that what you just told me is probably a circumstance that the priority force is not very well evenly distributed throughout the states. If that becomes a driving factor, which it is as far as you are concerned, of readiness, then possibly the Northeast is not very well postured with high priority forces.
 Page 360       PREV PAGE       TOP OF DOC    Segment 2 Of 3  

    Mr. OLVER. No, that is not—that is clearly not true. High priority forces depend upon—will show up in the readiness factor. And if you have got very high positions in the readiness factor and very high positions on the adequacy factor and high positions on the state priority factors, then you end up with very high scores. That is how you get 943s. You must be in the highest category of all of those.

    Colonel SQUIER. Right.

    Mr. OLVER. Since there is no number one on the readiness factor anywhere in any of these systems, the highest factor is number two in readiness factor. So that one should not be an issue. I mean, I think that—I am sure there are other factors, but if you created an objective assessment system, it seems to me that in large measure it ought to be producing the products that are up higher on the system.

    Colonel SQUIER. And we agree with your assessment. Let us take it for the record and give you a good concrete answer. We are working with the states and territories in developing this product, so we are looking for them to give us ideas on how we can streamline the process of being fair and equitable. If we get it before states and territories that will be a flare in this process. Any insights on how we can improve it, we certainly are taking your ideas.

    Mr. EDWARDS. Mr. Chairman.

    Mr. PACKARD. Yes, sir.
 Page 361       PREV PAGE       TOP OF DOC    Segment 2 Of 3  

    Mr. EDWARDS. Could I just observe that, John, I think is a very impressive work and you ask good questions. Could I also ask the gentleman if that basketball game went into overtime.

    Mr. OLVER. Well, it is actually very simple, because I had the list. All I had to do was do the scoring. When the paperwork is all done, it doesn't take very long. As you are watching basketball scores you just score other things as well.

PRIORITIZATION

    Mr. PACKARD. The time has expired. We will go on and then come back to you, Mr. Olver. But one comment. Out of the $48 million budget for Guard facilities and about five—in excess of $5 billion of FYDP projects, it is—there is no way we are going to be able to really—I mean, the prioritization becomes even more complicated because you are dealing with such a small amount where so far behind in terms of the total amount of projects, 48 million out of 5 billion, is almost—it is difficult.

    Mr. OLVER. Sure, the long infrastructure list gets all the way up to your 5 billion, but many, many of those are very far down on the list. All I am urging is that we end up with ones that are up there already high on that list in a scoring mechanism that we all agree is at least close, maybe needs some fine tuning. But fine tuning usually isn't more than a couple of percent on the total assessment in fine tuning. But those ought to be moving up rather quicker, I think, than they are.

 Page 362       PREV PAGE       TOP OF DOC    Segment 2 Of 3  
STATE PRIORITIES

    Mr. PACKARD. How do you calculate or bring into your evaluation of a project in terms of priorities with state priorities? State priorities are different, of course, than Pentagon priorities. How do you use the state priorities or how do they fit in?

    Colonel SQUIER. We have percentages that we apply to those three factors, the highest one being the readiness requirement. That is rated at 40 percent. The adequacy of facilities is a number that you apply to that equation. The adequacy of facilities is 35 percent. The state priorities is 25 percent. You mathematically compute all that and you arrive at a figure. The list of requirements that we have in our major construction years is growing because we are an old institution, as you are aware of, and we have a lot of old facilities. And therefore $50 million is certainly moving in the right direction, but it doesn't even begin to scratch the surface of our requirements.

    Mr. PACKARD. Thank you.

    Colonel SQUIER. Those are completely included in the scores that I have given to the projects, the scores by state priority according to their system.

    Mr. PACKARD. Okay. Mr. Wamp.

    Mr. WAMP. Mr. Chairman, I don't have any specific questions. Without belaboring the point, if you want me to yield some time to you, Mr. Olver, to continue this, I will be happy to.
 Page 363       PREV PAGE       TOP OF DOC    Segment 2 Of 3  

    Mr. OLVER. No, I think the question—I think that they would like to answer in detail. I mean, there are obviously cases where because there is a change—Mr. Kingston made the comment that there may be a change in some project because a new force need occurs rather late in the process. That has to be possible to put in. That would be directed by the Office of the Secretary of Defense. And there are a couple of them that appear here as directed projects that went outside of whatever had been the regular programming.

    And if you were one of the fine tunings, obviously, but it also appears as a subjective factor in essence, namely if you are doing a several-phase upgrading of a particular facility. The first two phases might be very high on the score and the last phase by itself, taken by itself, might not look so high on the scoring. But on the other hand, to finish the job, you might do it even though its scoring was only 500 on this 0 to 1000 scale just to complete the job. And that is perfectly reasonable that that should happen, too. There are a number of those factors. But I would be very interested in the other fine tunings that might be there.

    I think we need time to maybe review it a little bit later.

    Mr. WAMP. Mr. Chairman, I will just yield back.

    Mr. PACKARD. Thank you, Mr. Wamp. I was going to yield time to Mr. Hefner, but let me ask a question and then we will come back to you, Mr. Hefner, if we may.

CHEMICAL DEMILITARIZATION

 Page 364       PREV PAGE       TOP OF DOC    Segment 2 Of 3  
    What is the reason for transferring the chemical demilitarization to be funded under the Army? What are the benefits of changing that? Does it make a difference whether the chemical demilitarization is funded under the Army or under the defense-wide program?

    Ms. MOORE. It really doesn't. We are the executive agent, and we can execute the program either way. This was part of the devolvement from the Department of Defense under a review recently completed where recommendations were made to reduce overhead in the Office of the Secretary of Defense. And we—it came down to us through what we call a PBD or program budget decision from the Secretary of Defense, devolving the dollars and the people to the Department of the Army. That program is managed, except for the military construction, out of SARDA through the acquisition process.

    Mr. PACKARD. Has the Army's program been reduced to make room for the chemical demilitarization program either this year or in the future?

    Ms. MOORE. No, sir. The dollars were added to the top line.

    Mr. PACKARD. They were. And the cost is figured to be about $950 million? Is that what I——

    Ms. MOORE. Yes.

    Mr. PACKARD [continuing]. Understand from your testimony?

    Ms. MOORE. That is right, $942.8 million.
 Page 365       PREV PAGE       TOP OF DOC    Segment 2 Of 3  

    Mr. PACKARD. Thank you.

    Mr. HEFNER. Ms. Moore, what percentage of our budget is there for the total MilCon budget? Ten percent, 15 percent?

    Ms. MOORE. It is $125 million this year.

    General WHALEY. It is about 1/7, sir. It is 100 of a 700, roughly 125 million of 800 million, actually.

    Ms. MOORE. Where we get concerned, if there were ever cost overruns, we would have to find dollars in the Army budget.

PRIVATIZATION

    Mr. HEFNER. While you are doing your lessons learned, once you award the full contract, what are your plans for the long-term review of the effects of privatization? I think we are about to rush to judgment on privatization and what have you. What are your long-term plans?

    Ms. MOORE. If the program works, and that is a big if, we intend to watch Fort Carson very carefully. We probably would not award the second one for two years because it takes that long. It takes a long time to get these things up and running. But we intend to watch the operation of it very carefully. And it will be, again, based on the individual installation and the factors that exist at that installation. One of the criteria, the number one criteria, is that it has to be community supported before we would even consider it. And do you want to add something to that, General Whaley?
 Page 366       PREV PAGE       TOP OF DOC    Segment 2 Of 3  

    General WHALEY. Yes, ma'am. In terms of direct review of quality and cost, long term we will start quarterly reviews with Fort Carson internal to the Army to pick up lessons learned as quickly as we can. Our intent is to accrue better contracts and better understanding. It is to improve quality and to improve cost effectiveness as each contract goes by. There will be a review of that internal to the Army to ensure that we are achieving the best quality that we can for a reasonable cost.

    Mr. HEFNER. If you wait two years, what is that going to do with the housing projects? Will they be put on hold or what?

    Ms. MOORE. We have one at Fort Bragg now. We would not do that for an extended period of time. If that happened, we would go ahead and build, award contracts and build them with the military construction budget, but it is just a temporary hold right now. And that money would be transferred over into the CVI program as—I hate to use this word, but seed money, if you will, to get that project, the CVI project, off the ground. If that doesn't work out or it takes too long, we will go ahead and award those contracts. So you will get housing one way or the other.

NON-EXECUTION OF FAMILY HOUSING PROJECTS

    Mr. HEFNER. I have no further questions, Mr. Chair.

    Mr. PACKARD. Let me follow up on a question on the point that Mr. Hefner was following. So have you indicated, Ms. Moore, that projects are not being held up, at least not being executed on family housing projects waiting for or pending privatization approvals and processing?
 Page 367       PREV PAGE       TOP OF DOC    Segment 2 Of 3  

    Ms. MOORE. The only one I am aware of is the one at Fort Bragg.

    General WHALEY. There are several that are.

    Mr. PACKARD. There are four or five as I understand it.

    General WHALEY. Correct, sir.

    Mr. PACKARD. What is the total amount of dollars that is being held up?

    General WHALEY. $64, $65 million.

    Mr. PACKARD. $64 or $65 million dollars. And is this primarily because OMB and their scoring costs?

    General WHALEY. Yes, sir. In anticipation of that, the other piece, sir, is, as we mentioned, two years may seem like a long time. We will balance lessons learned from Carson when we go to do the Fort Hood contract or Fort Bragg follow-on or any of the others that we follow on. As we learn the lessons, the time between contracts should go down. We are indeed confident that we are going to—Carson will not be the model time line. It may be somewhat less than two years, but we also have to balance that with some operational experience rather than just contract award experience before we go on. As we gain additional experience and knowledge, the rapidity in which we can do this should increase.
 Page 368       PREV PAGE       TOP OF DOC    Segment 2 Of 3  

    Mr. PACKARD. So you feel that any delays are temporary, that there will not——

    General WHALEY. Yes, sir.

    Mr. PACKARD. They will be corrected?

    Ms. MOORE. Yes, sir.

    General WHALEY. And as I said, we are learning as we go through this. Part of that is the problematics that you have pointed out. We will act accordingly not to misuse the authority that you give us.

    Ms. MOORE. We did not, sir, want to stop programming family housing because this is an uncertain program at this point. And we continue to do that at Fort Hood and Fort Bragg, our larger installations where we know we are going to be there forever.

PRIVATIZATION—SAVINGS

    Mr. PACKARD. In answer to a previous question I think it was indicated that the primary benefit of the privatization is that it shortens the backlog. Are there actual savings in that? Have we been able to determine if we are saving money as we move into the privatization programs?

 Page 369       PREV PAGE       TOP OF DOC    Segment 2 Of 3  
    Ms. MOORE. We don't think it will save money, but the major benefit would be to get the housing faster, sooner.

    Mr. PACKARD. You don't anticipate it saving money?

    Ms. MOORE. It would just move it from the family housing over into the——

    Mr. PACKARD. Well, is that——

    Ms. MOORE [continuing]. Allowances account and they pay their monthly allowances and we don't have the capital cost.

    Mr. PACKARD. So you feel it is a shift from this bill to the housing allowance, is that what you have said? Is this a zero-sum game, then, that is——

    Ms. MOORE. Yes.

    Mr. PACKARD. There won't be any savings that you see? It is simply a shift of cost?

    Ms. MOORE. Yes.

    Mr. PACKARD. Our assumptions about the future funding levels for housing allowances—is there—how does it relate to that? Is there any assumption about the future funding levels for housing allowances?
 Page 370       PREV PAGE       TOP OF DOC    Segment 2 Of 3  

    Ms. MOORE. Simply we expect allowances to go up as the cost of living goes under the military personnel guidelines as far as BHA is concerned.

    Mr. PACKARD. Then if there is not really a savings anticipated, how do you look on the privatization? Do you feel that it is a direction where we want to move everything in or is virtually every project looking at privatization? Are we going to continue to do some under the standard program?

    Ms. MOORE. Just some. We don't do it now as a total program. We would analyze each installation, sir, as we come to it.

    Mr. PACKARD. So privatization is not——

    Ms. MOORE. Some places it might work. Other places it might not work.

    Mr. PACKARD. I see.

    Ms. MOORE. Where it does, it gives us housing much faster.

    Mr. PACKARD. Mr. Tiahrt.

BASE REALIGNMENT AND CLOSURE

 Page 371       PREV PAGE       TOP OF DOC    Segment 2 Of 3  
    Mr. TIAHRT. Thank you, Mr. Chairman. I was reading your background here and it says in 1990 you developed a six-year process for the closure and realignment of U.S. military installations.

    Ms. MOORE. My past has caught up with me.

    Mr. TIAHRT. It expired December of '95, but it directed the closure and realignment of 386 military installations. Is that process complete or is that continuing?

    Ms. MOORE. We are still closing bases, the '93 and '95 BRACs are still ongoing.

    Mr. TIAHRT. Thank you.

    Ms. MOORE. There was an '88 closed, a '91, and they have been completed. Is that right, is it '93 and '95 that are still out there? We expect those to be completed—there was a six-year time frame associated with each closure list.

    Mr. TIAHRT. Are the closures continuing?

    Ms. MOORE. For six years, yes.

    Mr. TIAHRT. Six years, so six years from '95 the last should be closed?
 Page 372       PREV PAGE       TOP OF DOC    Segment 2 Of 3  

    Ms. MOORE. Right.

    Mr. TIAHRT. About 2001?

    Ms. MOORE. Yes.

    Mr. TIAHRT. Do you think it is time now to look at another base realignment?

    Ms. MOORE. We really need to do that, all the services do. We didn't get all the installations that we needed to close. We still have that same capacity in a number of places. And of course, you know, that costs us money. We need another BRAC.

    Mr. TIAHRT. Does that start in 2001 as far as completion?

    Ms. MOORE. That may be.

    Mr. TIAHRT. So you would continue the process by continuing in 2001?

    Ms. MOORE. Yes, sir.

    Mr. TIAHRT. I think there has been concern that we are not completing what we started out to do in the previous rounds of BRAC. And before we do another one, we want to make sure that we complete the task we started out and not let the process become politicized.
 Page 373       PREV PAGE       TOP OF DOC    Segment 2 Of 3  

    Ms. MOORE. Yes, sir. That was the beauty of that process. It was not supposed to be politicized, removed the politics from the list.

PRIVATIZATION—UTILITIES

    Mr. TIAHRT. I have a specific question, and this doesn't have to be answered today, but at Fort Leavenworth with the disciplinary barracks phase II, it was my understanding that the Army was trying to get out of the utility business, and yet there is a gas-fired boiler in the project to provide some power. I just wonder if there has been a study done of whether it is cost effective or not.

    Ms. MOORE. We could——

    General WHALEY. Sir, I don't know personally, but the guidance is that before we buy any utility there would be an economic analysis done to make sure that the right economic decision is made, privatization versus buying. Heating plants are one of those that are on—that is really dependent on where you are, what structure you already have and where the economics figure in each facility.

    Mr. TIAHRT. We will give you an opportunity to answer that in writing. At the state level they are undergoing retail wheeling considerations which would allow utilities to bid for the right to power, and that may make it cheaper than what it is now.

    Ms. MOORE. We are very interested in those programs. We are pursuing them everywhere.
 Page 374       PREV PAGE       TOP OF DOC    Segment 2 Of 3  

HOMEOWNER'S ASSISTANCE PROGRAM

    Mr. TIAHRT. One other thing that I don't know if you have done any work or studies or not, but in the private sector when people are required to transfer, there company will quite often pick up their privately owned housing and market it for them or provide some guarantee that they can move from one location to another and not fear the loss of equity in their home.

    And one of the drawbacks of being in today's military is that you don't have location security in your job security. Thus, you may have job security, but you don't have location security. I wonder if there has ever been a feasibility study so that enlisted as well as officers have a guarantee that if they buy a home they will have some kind of assistance for down payment. In addition to that if they are transferred to another base that they have the ability to have some company manage the sale of their home without fear of loss of their equity.

    Ms. MOORE. We have a homeowner's assistance program that is designed to help those people who have suffered losses from their real estate property because of a base closure or some other action that is beyond their control. And we buy those houses and help them at the beginning. And we will try to resell——

    Mr. TIAHRT. Is there a policy on that?

    Ms. MOORE. We can——

 Page 375       PREV PAGE       TOP OF DOC    Segment 2 Of 3  
    Mr. TIAHRT. Would you send that to my office.

    Ms. MOORE. Yes, sir.

    Mr. TIAHRT. Thank you very much. Thank you, Mr. Chairman.

    Mr. HEFNER. To follow up on what Mr. Tiahrt stated, is there a length of time that a person has to be at an installation? Say he goes in and he buys a place and has it for six months. Then all of a sudden he has to relocate. Does he have to be there for a certain length of time before he is eligible for a buy out or whatever?

    Mr. JOHNSON. If he buys the house and then he gets ordered to move for some reason or another, if the installation has a drawdown on or an A–76 program. Then he is eligible for us to pick up his house. And if he sells it, we will guarantee him 95 percent of his loss. If we have to buy it, we give him 75 percent or pay off the mortgage.

    Mr. PACKARD. Okay, we will start the second round of questions. Mr. Edwards.

QUALITY OF LIFE

    Mr. EDWARDS. I really don't have a lot. I wanted to ask, perhaps, one question that affects my thinking about the importance of the quality of life issues. On page 3 of your report, Secretary, you said that on an average 197 days are spent away from one's family when one has been either deployed in an contingency operation or off post training. Is that the same basis that—a year or so ago I was using 135 days as the average time spent away from home. Is this a different calculation or are we that dramatically increasing?
 Page 376       PREV PAGE       TOP OF DOC    Segment 2 Of 3  

    Ms. MOORE. I think it has currently increased because of all the contingency operations we have on right now.

    Mr. EDWARDS. And, Mr. Chairman, I just make a note on that. In our discussions with our colleagues about the importance of supporting quality education and housing and health care for these families, I don't know, you know, how many people if they had to do that year after year would stay in the military no matter how patriotic they were. My wife would probably push me over a cliff or divorce me if I spent that many days away every year, and that is a tremendous strain on our family.

    Ms. MOORE. That is exactly why all these quality of life programs are so important, sir. Retention is a very important result of a good quality of life, because families have a large say in whether someone stays in the service or not. We need to take care of our soliders.

BASE REALIGNMENT AND CLOSURE—SAVINGS

    Mr. EDWARDS. One other final question. I think this was addressed earlier. You did touch on BRAC, but you have the expenditures in the budget for BRAC, but is there a number, appropriate number for what you estimate the true savings from base closings to be? Would that be in your budget or somewhere else? There is a lot of discussion, obviously, around the capitol about BRAC, and I know the savings have been slow in coming, but it is my impression that in the last year or so we were about to turn the corner where there would be a definite savings.
 Page 377       PREV PAGE       TOP OF DOC    Segment 2 Of 3  

    Ms. MOORE. We did last year. We are saving more now than we are spending.

    Mr. EDWARDS. We are saving more than we are spending?

    Ms. MOORE. That is right.

    Mr. EDWARDS. Any number on that? Are we about to break even?

    General WHALEY. Sir, we have already broken the juncture where we are now saving more than we are spending. The savings will continue to grow as we go out in the out-years.

    Mr. PACKARD. And at the same time, the spending will decrease?

    General WHALEY. Will decrease, yes.

    Mr. PACKARD. I think that is the long-term picture. The savings will be in perpetuity almost. The spending will be very, very short term.

    Mr. EDWARDS. The spending comes out of our military construction budget, though. The savings may apply to other areas of the budget, is that correct?

    Ms. MOORE. We will be saving about a billion dollars come 2001.
 Page 378       PREV PAGE       TOP OF DOC    Segment 2 Of 3  

    Mr. EDWARDS. Okay, a billion dollars by 2001. Thank you, Mr. Chairman.

RETENTION

    Mr. PACKARD. In view of some of the comments just made and the answers, and I know that Mr. Hefner has and every member of the committee has been saying that retention is being affected by the deplorable conditions of our quality of life facilities, housing and barracks, daycare centers, et cetera. We have been saying that, and I think it is true, but we have really no statistical background to bear it out. Would you provide for the committee a comparison chart showing the percentage of Army reenlistment during the last ten years? I would like to see if in fact we are seeing a decrease in reenlistments. We don't—naturally we can make assumptions that much of that could be because of housing deficiencies and problems. It could be a multitude of other reasons, of course. But I would like to see if in fact we are seeing a deterioration of reenlistments. Would that be a difficult thing for you to produce us with that, a comparative chart of the last ten years?

    Mr. HEFNER. I think there are a lot of other things that enter into the equation other than the housing.

    Mr. PACKARD. Oh, sure.

    Mr. HEFNER. Because I remember in years past we had some big incentives for people to reup, and it was fine to be in the military even back in the Regan years when patriotism was on the rampant. We found that the majority of people were joining the services basically for the education benefit that they got. Of course, they were patriotic, too, but now for people to reup there is not the real incentives that there were in the past.
 Page 379       PREV PAGE       TOP OF DOC    Segment 2 Of 3  

    Ms. MOORE. The economy is good.

    Mr. HEFNER. The economy is good and you can almost make an argument to have a draft. You know, I have always said that a draft, to me that was a logical way where nobody is exempt from military service. The argument was made that under the volunteer army we got so many people to pick from. Under the draft, you got everybody to pick from. So I think we have got a problem down the road being able to fund quality of life programs, the retirement system and the whole thing. We have got a real tiger down the road that we are going to have to address some way. This committee has fought so hard for so many years for quality of life. We have povided a major share of the burden for these programs.

    Mr. PACKARD. Only a man retiring would recommend the draft come back.

    Mr. HEFNER. I recommended that 24 years ago.

FACILITIES REDUCTION

    Mr. PACKARD. On page four of your testimony, Ms. Moore, you mention that you show the figures of the decrease of square footage of facilities overseas of reductions. I figure—I calculated about 34 percent reduction in physical facilities on an overseas basis that is projected by the year 2003. Has the manpower reduction exceeded that or about the same or is it much less?

 Page 380       PREV PAGE       TOP OF DOC    Segment 2 Of 3  
    Ms. MOORE. We are down, I think, to about 100,000 overseas now.

    Mr. PACKARD. I saw that in your testimony, 100,000. From how many?

    Ms. MOORE. We are coming down to 65. 300,000.

    Mr. PACKARD. It was 300, so it appears that the manpower may exceed the actual physical facility reduction.

    Ms. MOORE. Yes, sir.

U.S. MILITARY ACADEMY, WEST POINT: PHYSICAL DEVELOPMENT CENTER

    Mr. PACKARD. Thank you. Also a couple other questions I had specific to your testimony. On your Army Military Academy thing, and I think there is a question or two I had, but I will probably overlook that for a time. I simply want to emphasize that in your testimony you mention $12 million of revitalization for physical development center at West Point. That really is an $85 million project, is it not?

    Ms. MOORE. Yes.

CHEMICAL DEMILITARIZATION

    Mr. PACKARD. And then down on that same page, on page 8, you speak of destroying chemical weapons of war. What are those chemical weapons?
 Page 381       PREV PAGE       TOP OF DOC    Segment 2 Of 3  

    Ms. MOORE. That is the chem demil process. They are chemical weapons.

    Mr. PACKARD. Would you provide for the record the different kinds of chemical weapon systems that you—okay, I would just like to know what they are so that I can be virtually familiar.

    [CLERK'S NOTE.—The Army did not respond to the chairman's request.]

    Ms. MOORE. Yes.

    Mr. PACKARD. Now let us go back to Mr. Olver for his second round of questions.

    Mr. HEFNER. New ball game.

    Mr. OLVER. Thank you very much. I will be much shorter this time, because I want to agree with the quality of life comments that have been made here. That is what this committee is about. I see it as a problem of not just the enlisted and people, but in something that we get an opportunity to do in making our recommendations for appointments to the officer core and to go to the academy. And I must say that within the six years the number who are applying has gone down very substantially. And in part it is because of the good economy and the career development and so forth, but quality of life issues are going to be very important whether we are going to get high quality personnel at all levels.
 Page 382       PREV PAGE       TOP OF DOC    Segment 2 Of 3  

    I am not sure that I made a clear enough statement in the beginning about why I got so interested in this Guard issue. I would like to say to you, Madam Secretary, that coming from New England, where as I do, we really don't have much in the way of active bases anymore. We have basically Guard and Reserve bases at all of the—in all of the services.

    And so I am—if you look around this table, I am the only one that comes from up that part of the country. And I am—well, obviously that is not going to be the only thing that I take an interest in. It is one that I feel that I should be at least particularly sensitive to the needs of Reserve and Guard needs, both Reserve and Guard in the services where both of them apply. The Reserve where there is no Guard——

    Ms. MOORE. Well, we appreciate that.

    Mr. OLVER [continuing]. We will be watchful for and sensitive to the kinds of needs that are there. So I am trying to understand the process and see that I know it well over a period of time.

    Let me just—one last item here. I have been trying to figure out, because I had been given an early version, I guess, of the FYDP. There must be an additional version of the FYDP or something like that. And the original FYDP had Camp Dawson in Fiscal Year 2000. Of course, I think that was related—and I want to get this confirmed whether you would be able to or not.

CAMP DAWSON—UNBUDGETED PROJECT
 Page 383       PREV PAGE       TOP OF DOC    Segment 2 Of 3  

    We ended up doing an add-on on Camp Dawson whether it was Congressional or marking a defense, who knows. It was an add-on to last year's, the original '98 FYDP, but which if everything on the '98 FYDP was included, then some additional items were taken off several other years. And deeper down, every one of the years, in fact, '99, 00, 01, 02 and 03 were projects taken, as well as a couple that had not been on that FYDP at all, one of which was the Camp Dawson. At that point what was put in the budget for this year was $6.8 million. And I notice in the original FYDP here we had Camp Dawson down for $13 million in the year 2000.

    Now I see that in the year '99 in the Department of Defense release—and it does—when it is added, you come up with a 42.9 million project, $42 million plus, that you had given in your testimony. But with that last project, it was not in the '99 FYDP. It is in for 4.4. There is actually—either project of the two, what was done in previous fiscal year and what is done—what you are proposing for '99, does that cover? It is very close to, at least. It is not quite the same number as the one that originally appeared in the old FYDP under the year 2000.

    Ms. MOORE. The 4.4 represents 75 percent of the federal funding for a smaller facility.

    Mr. OLVER. Those are two different projects. So the 4.4 in '99 and the 6.8 that was done in '98, they do not—do they or do they not go together to substitute for the one which was on your last year's FYDP before any directions by the Congress? The two complete to Camp Dawson items.

 Page 384       PREV PAGE       TOP OF DOC    Segment 2 Of 3  
    Ms. MOORE. The budget, the 4.4, for the——

    Mr. OLVER. For the 6.8?

    Ms. MOORE. Yes.

    Mr. OLVER. Then indeed the 4.4 would complete that project?

    Ms. MOORE. That is right.

    Mr. OLVER. That is all. Thank you.

ADVANCED APPROPRIATION

    Mr. PACKARD. Thank you. Would you explain for me again—I read carefully your advanced appropriation section of your testimony. How is this done and how is it scored? I apologize for my ignorance on it, but I really would like to try to get a feel.

    Ms. MOORE. It is our understanding that OSD, after consultation with OMB, OSD, directed again by a program budget decision down to the Army that these projects be funded based on advanced appropriations. Now it does have something to do with only using funds that can be obligated in a given year but appropriated for the full project up front.

    Mr. PACKARD. That obligation future Congress knows.

 Page 385       PREV PAGE       TOP OF DOC    Segment 2 Of 3  
    Ms. MOORE. Right, it does, yes, sir.

    Mr. PACKARD. But it does not score all of it in one year, in the year that it is—it is scored on a year-by-year basis?

    Ms. MOORE. Our incremental funding has done the same thing. It allows us to include more projects, more priority projects, bipartisan priority projects in a given year than we would ever be able to do without the incremental funding. Now OMB has wanted to fund the entire project in any given year.

    Mr. PACKARD. And score it that way?

    Ms. MOORE. Yes, and score it that way. Which has made it very difficult for the services to fund.

    Mr. PACKARD. Has the current system not worked for you, the incremental funding?

    Ms. MOORE. Well, we are limited in how many projects we can get through the budget process to present to you because the top line doesn't allow a number of them. This is the first time they have ever allowed it in the budget process, if that makes sense at all to you.

    Mr. HEFNER. Have we dealt with this before, Mr. Chairman? We have dealt with phase funding. That is basically what you are talking about.
 Page 386       PREV PAGE       TOP OF DOC    Segment 2 Of 3  

    Ms. MOORE. Yes, sir, that is what it is.

    Mr. HEFNER. And I think the question is if this year we take the Army Hospital in Fort Bragg as an example. We phase funded that, and say for the sake of argument in this fiscal year, we are going to have, say, $60 million for that this phase. Now in the next phase it wouldn't be—only $60 million that would be carried in this budget that we are talking about, am I right?

    Ms. MOORE. That is right.

    Mr. HEFNER. So you don't get double counting, but something like he was talking about someone wanted to do the full funding. If you do that, you are not able to complete the project, so you have got that money counted there and see there is money you have got to borrow or count interest against it some way. You have got to account for it some way. Phase funding, in my view, is to assure people or the services or whoever, that once you start a project with phase funding you have got to commit that you are going to complete it.

    Ms. MOORE. That is fair.

    Mr. HEFNER. So you may have to stretch it out further. I know in the seventies we had to do this and at Fort Bragg we had to do it with the medical center there. We do it all the time with weapon systems.

    Ms. MOORE. Yes.
 Page 387       PREV PAGE       TOP OF DOC    Segment 2 Of 3  

    Mr. HEFNER. We don't pay the whole bill for the B–2 in a year. I mean, if you did that we would have a lot of pauses along the way. Is that what we are talking about?

    Ms. MOORE. I think in a way this committee pioneered the phase funding, and it has worked very well. It has been done on the Hill. This is the first time OMB ever bought off on it, but they are asking for essentially advanced appropriations out through the life of a contract. We didn't do that with incremental funding. We just knew we were going to do it every year.

    Mr. PACKARD. I think this committee will review that very carefully. The thing we don't want to do is lose control of those annual funding mechanisms for any savings that might come about through the process. The committee, I think, would want to retain control of those. But I want to understand it fully and I have still yet to do my own research on it and then we will evaluate whether this is the direction we want to go.

TOTAL ARMY FORCE

    Did you have any other questions that you had? I wanted to ask a couple of questions, if I may. We have been briefed here recently on the Hill on the total Army force. And they were good briefings, incidentally. And I was somewhat pleased with what I heard in that it appeared that there was going to be a greater effort for the active Army to integrate the Guard and the Reserve into their total planning and total force concept. Will we see that affecting the way you budget? Are we going to have a total force budget submitted as we proceed in this direction? Will we see planning done with an integration of the Guard or the Reserve better than we have seen in the past where they become an integral part, sit at the table, determine where your plan is going to take you——
 Page 388       PREV PAGE       TOP OF DOC    Segment 2 Of 3  

    Ms. MOORE. Absolutely.

    Mr. PACKARD [continuing]. And what the budgeting priorities are going to be?

    Ms. MOORE. Absolutely.

    Mr. PACKARD. You see that?

    Ms. MOORE. That happened in this process.

    Mr. PACKARD. It did this year?

    Ms. MOORE. Yes, sir.

    Mr. PACKARD. For the first time?

    Ms. MOORE. Yes, sir.

    Mr. PACKARD. Let me ask you, the Reserve or the Guard, are you satisfied that it is moving into this total force concept, is it drawing the Guard and the Reserve into the planning process better than ever before?

    Colonel SQUIER. Yes, sir, I feel very confident in the process that we are working with the Army now to recognize every part given the very limited budget. I think we are pointed in the right direction.
 Page 389       PREV PAGE       TOP OF DOC    Segment 2 Of 3  

    Mr. PACKARD. I think it goes right back to Mr. Kingston's, and of course Mr. Olver's interests, and that is in the past this committee and this Congress have plus upped due to fund programs in the Guard and the Reserve that were not submitted in the budget. And the general feeling was that the Guard and the Reserve were almost, as it were, a stepchild of the active duty. And the active duty knew that we would plus up and provide funding, that they didn't have to include it in their budget. And I think we expressed last year in this committee that we would like to see that reversed and that their budget would reflect the priorities of the Guard and the Reserve and that we would not be required to do nearly as much plussing up as what has been done in the past. Is that what you anticipate will be the effect of this total Army force concept?

    Ms. MOORE. Sir, when we started the integrating, that is true, and under constrained resources, their packages, they believe, is fair this year. But it is enough dollars for the Guard, the Reserve or the entire MilCon budget or anywhere else, for that matter. So we don't have enough money to satisfy all the requirements, but we are sharing the lack of money.

    Mr. HEFNER. Sharing the pain.

    Ms. MOORE. Yes, and it is painful.

    General HELMLY. Congressman, if I may, this is actually the second budget submitted by the Department in an integrated form. And that is to say that in the mechanical process of building the Army budget we collapsed the separate program evaluation group for the National Guard and for the Army Reserve. And that whole process is integrated across six program groups. In the case of this committee in particular, all the installations where we deal with MilCon budget we have already made a related budgetary department. So that has worked out well. I would say that as a mark of that Guard and Reserve proponents saw a real growth despite the fact that the Army's total obligation authority was down. So I think that that very fact—there is lots of money top line, but both Reserve and Guard saw uncertain counts. It shows the integration and prioritization that occurs regardless of the proponent.
 Page 390       PREV PAGE       TOP OF DOC    Segment 2 Of 3  

    I would add one thing. In testimony a few weeks ago, and I stated it again this morning, I personally believe that the Army for its perhaps past sentence has been unfairly branded as the least integrated of the services. I would only note that in my own view I believe the Army's faced extraordinary efforts over the last several years to integrate its Reserve and Guard at virtually every operational frame, resource, policy making and management procedure that occurs. And as Ms. Moore said, at a time when we are all constrained and placing a whole effort to new challenges of how to reduce, how to respond in terms of demographic challenges, et cetera. We all share in those challenges.

    I sort of say it this way. For a long time we asked put me in the game. You happened to put me in the game and the game is really tough, but we are in the game.

    Mr. PACKARD. Well, I hope you are in the game and I hope that it is not just rhetoric, that it is not just platitudes to the Guard and the Reserve when it comes to budgeting. The fact is, and that is why I am still a little concerned, that it is not happening quite the way that it sounds. Last year the request was about the same as this year for Guard and Reserve, about $50 million, 45 last year. We plus up to about 120 last year. I would like to see the plus ups come down and the submissions coming from the Administration going up in that area to better reflect what the realities are.

    That is something I will watch rather closely as long as I am on the subcommittee, because I really believe that if you really do intend to integrate into a full Army force or total Army force, then that has to be reflected in the budgeting. And if we feel a need to plus up, then either we are wrong or you are wrong in terms of the funding. And we don't feel that we ought to be micro-managing your budget. We think that that ought to come from you, that you are the ones that ought to determine those priorities and not the political process as it has been done in the past.
 Page 391       PREV PAGE       TOP OF DOC    Segment 2 Of 3  

    We may not get there. I am not sure we will ever get there because the politics is going to enter into this probably forever. But I would like to see that your budget proposal will come up in those areas as they are integrated into the planning, into the budgeting, in the prioritizing of projects and that our plus ups will come down. The need for those plus ups would come down, I would hope. Comments, sir?

    General WHALEY. Madam Secretary, Mr. Chairman, if I might, since my two brothers over here have already given their piece, two issues. I believe in everything you say. I think we have made some progress. If you take out the barracks programs, strategical ability and chem demil, set those aside away from the act of force, the requirement for revitalization has been satisfied 12 percent for the Guard, 38 percent for the U.S.A.R. and 13 percent for the active of the requirement. So I think you are already seeing that adjustment amongst the three components. As Madam Secretary has already pointed out, if we had more we would spend more. It is not there. It is a balanced risk program amongst all the other appropriations.

    The other piece is as far as integration. I have an active Reserve and National Guard MilCon action officer in my office that fills the FYOP and the budget itself, because I think before there were agreements made that may not have gotten down to where the work is actually done. The work is now actually done by those three component officers that if it starts to get out of kilter what was agreed to, right back to the components.

    At the other extreme, I know the Chief of Staff of the Army, General Boratz and General Navas, review personally the level that they got out of the budget. I have heard that from General Navas's mouth just yesterday. He is pleased personally with the process.
 Page 392       PREV PAGE       TOP OF DOC    Segment 2 Of 3  

    Mr. PACKARD. Mr. Hefner.

RECRUITMENT

    Mr. HEFNER. Well, I just want to ask a couple questions coming to us off of the reservation. Is our recruitment sufficient now to basically meet our quota?

    General WHALEY. We are. Fortunately I heard it yesterday, so it is still fresh in my mind. We are making our reenlistment and recruitment objectives. It is a delicate balance. Back to a ten-year how are we doing statistic, we have to put—we downsized in that ten years, as well. So there is a mix of messages being sent to American cohorts.

    Mr. PACKARD. If you would yield, please. I understand also that when reenlistments do not take place or they muster out, that there are reasons given and those are tabulated. I am not asking that you give us statistical information on it, but for members, there are reasons given why they go out. And it would be very interesting to be able to review those, basically, but I think the numbers would probably be overwhelming for us to do so.

    General WHALEY. Sir, to get to the heart of the matter, that is your efforts in enhancing quality of life. One of the reasons that we are being successful in retention is that families and service members are confident that, one, their families will be taken care of, and two, the families are confident that they will be taken care of. And their quality of life is adequate. So when that service member is not at home, then the family is being taken care of. They trust you and they trust us to insure that that quality of life is there. It is not just housing. It is the family action programs, the Army family building. It is childcare. It is the hospital.
 Page 393       PREV PAGE       TOP OF DOC    Segment 2 Of 3  

    Across the spectrum if they see improvement as a result of your efforts in their quality of life and a promise that it will not only stay the same but indeed will get better in time, they trust us to do that. I think they are not walking. They are staying. And there is pockets where folks—where we have little some stay and some don't by MOS, by skill. We had a little problem with infrequency, but I think that has been fixed.

    The report yesterday that was asked for of the Army yesterday was retention is fine and recruiting is up.

    Mr. HEFNER. One other question. As we continue to draw down are we going to have a problem with our reserves? The guys that are coming out that want to stay in the reserve, are we going to get into a situation where we have to make a choice for the guy that is already in the reserve? Are we going to have a problem with that?

    General HELMLY. Sir, for our part there is—the recruitment effort is very heavy, and I think the same holds true of the National Guard for what we call prior service soldiers, because that young man or woman is being released from the active part of the Army with a very strong skill level and a great amount of experience. So we both try to assess very heavily from that. And I don't believe there is really any competition between the prior service and non prior service. Non prior service on a daily basis is getting more difficult, that is recruiting from the civilian market, because of the many factors you have discussed here this morning. There is not a competition between prior service and non prior service.

    Numerically there can be an individual basis where you need a certain type of skill, a mechanic, and the soldier leaving active duty is a signal repair or something of that nature. And even there we budget it to provide him retraining to capture the soldiers prior experience and skill level. That soldier has already proven their ability to assimilate, live and function in our Army today, including its values, its missions and understands its traditions, the chain of command, leadership and everything. And they provide us an excellent product, prior service.
 Page 394       PREV PAGE       TOP OF DOC    Segment 2 Of 3  

    Mr. HEFNER. I guess that is the answer. What I am basically interested in is numbers, the ones that are coming out and the ones that are out. Are we going to be able to accommodate people that want to stay and people that are coming out and want to get in. Is that going to be a problem?

    General HELMLY. I do not believe it will be a problem. Our industry numbers are sufficient that for the number of soldiers leaving the active proponent there is room for them. In our case, this year U.S. Army Recruiting Command recruits centrally for the Army Reserve and the regular Army. And their destination target this year is 40,000 for the Army Reserve, and roughly 60 percent of that will be prior service market as a whole. And you might be able to find an individual case some place, but I don't know any instances where an active duty soldier leaving was denied entry to a National Guard or Army Reserve unit except on an exceptional kind of basis.

    Mr. HEFNER. I don't have anything further.

OVERSEAS HOUSING AUTHORITY

    Mr. PACKARD. Thank you. I would like to pursue just, again, my own edification. The overseas housing authority, outline for us the Army's legislative proposal for the establishment of the overseas housing authority as a non-appropriated fund instrument and provide some details. You might want to provide for the record more detail, but would you just briefly outline how it works and in which countries you are anticipating using it, et cetera.

 Page 395       PREV PAGE       TOP OF DOC    Segment 2 Of 3  
    Ms. MOORE. Early on, Korea and Germany.

    Mr. PACKARD. Korea and Germany?

    Ms. MOORE. General Whaley has been living with that intimately day by day, so perhaps he can give you the specifics.

    General WHALEY. Yes, ma'am. Sir, essentially since we don't own the quarters and it is in a foreign country, our objective is the same as it is in the United States, to revitalize our family housing to standard by 2010. Without massive dollars put against that, we will not make that. One way to do it is similar to CVI except the entity that would manage the dollars that come from a soldier's basic housing allowance would be a government non-appropriated fund entity, U.S. Government. Those dollars would be applied against, the revitalization of those quarters. So it is a steady flow of dollars to put against the existing sets of quarters. We would not build any more. We would just revitalize the ones that we have.

    There are a couple of issues, one, we need your approval to do it. Two, the benefits are that it provides a steady stream of revenue or cash that would allow an entity, a government entity, a non-appropriated fund to manage, operate and maintain those quarters with a steady flow of income. Right now the dollars that we give them is up and down. Did I say something wrong? No, okay. If I did, please say so.

    Mr. PACKARD. So it is primarily to be used for upgrading and maintaining?

 Page 396       PREV PAGE       TOP OF DOC    Segment 2 Of 3  
    General WHALEY. Yes, sir.

    Mr. PACKARD. Not to build new?

    General WHALEY. Not to build new.

    Ms. MOORE. Because of status of forces agreement.

    Mr. PACKARD. Do you anticipate a savings in this process?

    General WHALEY. No, sir.

    Ms. MOORE. No, sir.

    Mr. PACKARD. The Congress has never failed to give you what you have asked for in terms of foreign overseas family housing. Why do you wish to have this additional authority?

    Ms. MOORE. Well, sir, we probably should make clear it hasn't been approved by OMB yet, so it is not really before the Congress. It is still in the Department of Defense and OMB. They can't figure how to score it.

    Mr. PACKARD. Okay, well, we will probably hear more about this later. Is there any—Mr. Olver, do you have additional questions?

 Page 397       PREV PAGE       TOP OF DOC    Segment 2 Of 3  
    Mr. OLVER. No.

LOW BUDGET SUBMISSION

    Mr. PACKARD. There are other questions, but I am going to ask you just to respond to them for the record. I have some questions I had and then I have a whole list of rather technical questions that are strictly for the record. Usually on a project-by-project basis that we certainly don't wish to get into today.

    I think that I can conclude if there are no further comments or questions by committee members, that we are concerned about the low budget submission that was expressed in the last hearing. We would like to express it again that we are concerned that the President and the Administration has submitted to us a budget of significant reduction on military construction overall. The Army, perhaps not quite as great, we see an overall reduction of about 15 percent from last year's appropriated levels. To us that is unacceptable. We simply do not believe we can meet our commitment to our men and women in the services that we are going to address seriously their quality of life issues. And we think that a 15 percent cut where we already have cut ten percent in the last two years in a row, about a 30 to 35 percent cut over three years. We don't believe that that reflects a commitment to our men and women and the quality of life for them in the services.

    And so you can fully expect that we will probably be adding on projects to the submission, the President's submission. Your submission is about a seven percent cut the way we calculate, which is certainly about less—more than 50 percent less than what the overall budget level is in the request of the President, but it is still of concern to us. I want you to know that, Ms. Moore. We are concerned about the stated commitment to protect and provide adequate facilities for our men and women in the services, particularly in the State of the Union message, but the fact is that it doesn't reflect in the budget. And that is something we will have to grapple with as a committee.
 Page 398       PREV PAGE       TOP OF DOC    Segment 2 Of 3  

    However, we do appreciate very much your testimony, your explanations, and we will appreciate very much your written response to the questions that we are submitting for the record.

    Mr. PACKARD. With that, if you have no further comments——

    Ms. MOORE. I just have one. I want to thank the subcommittee for your support over the years to the Army and to all the services, for that matter. It would be very difficult without your support. And I also want to say to Mr. Hefner, who will be retiring, he was a tower of support over the years. When we had a problem, we could come to Hank and Liz and the committee and certainly you, sir, have followed in the same way. We depend on you. We thank you, and we will miss you, sir.

    Mr. HEFNER. Thank you.

    Mr. PACKARD. That is very true. We will miss Mr. Hefner. We work very closely, as your own experience reflects, with the authorizing subcommittee on the National Security Committee. Mr. Hefner and I have worked closely together, and I will certainly continue to do that. Your experience there, I think, will be very helpful in that response. If there is nothing further from members of the subcommittee, then, this hearing will be adjourned. And again, thank you.

    [CLERK'S NOTE.—Questions for the Record submitted by Chairman Packard.]
 Page 399       PREV PAGE       TOP OF DOC    Segment 2 Of 3  

Chemical Demilitarization

    Question. The Army Military Construction account appears to show a considerable increase over the amount appropriated for fiscal year 1998. However, after taking into account final action on line item veto, as well as $125 million for chemical demilitarization that was financed in the past in another account, the Army's construction is actually down 7%. What difference does it make whether chemical demilitarization is funded under the Army or under Defense-wide?

    Answer. There will be no change in the execution of the chemical demilitarization program. However, two changes in current law are required:

    (1) Chemical Demilitarization Legislation must be amended to permit the program to be funded in the Army Military Construction account rather than in the Military Construction, Defense account for chemical demilitarization projects.

    (2) Since the prior authorization was provided in the Military Construction, Defense account for the Umatilla AD and Pine Bluff Arsenal projects, new appropriation legislation is required to permit the Army to spend Military Construction, Army (MCA) funds for these projects.

    Question. Has the Army's program been reduced to make room for the chemical demilitarization programs, either this year or in the future?

 Page 400       PREV PAGE       TOP OF DOC    Segment 2 Of 3  
    Answer. No, the Army's program has not been reduced. Current and future chemical demilitarization construction program requirements were fully funded at the time funding responsibility devolved from the Department of Defense to the Army. In addition, due to future uncertainties and past history of the program, additional funding was programmed in the out-years to cover unanticipated requirements. It is not anticipated that there will be any impact on the future Army program as a result of the chemical demilitarization program. Currently, the military construction program is scheduled to be completed in fiscal year 2003. If cost overruns or program extensions occur that are not covered within the funding transferred by OSD, the Army will be responsible to provide the additional funding.

    Question. Is it correct that the chemical demilitarization program will cost over $950 million in military construction over the Future Years Defense Program?

    Answer. A total of $942 million for all chemical demilitarization construction projects is included in the Future Years Defense Program.

Total Army Force

    Question. The Army is briefing members of Congress about the integration of active, guard, and reserve forces in the total Army force structure. To what extent is there integration in the planning and budgeting process, especially or infrastructure requirements?

    Answer. In the installations funding arenas, this is a success story. All components have joined in the process on equal footing or parity, with each component determining requirements based on the same methodology; for construction requirements, each component uses a 57-year revitalization cycle plus buyout of the facilities deficit spread over 25 years. We believe the concept of parity is fully embodied in the development process of the Army's installations programs. Each component is represented on each of the main decision making bodies involved in the planning and budget process: Program Evaluation Group, Program Budget Committee, and the Army Resource Board. Nevertheless the Army has only been able to fund its highest priority programs—statutory requirements (such as environment and ChenDemil); Barracks; and Strategic Mobility. Beyond this, all components are funded equally, for example in revitalization military construction (Active—13%, Guard—12%, Reserve—38%).
 Page 401       PREV PAGE       TOP OF DOC    Segment 2 Of 3  

    Question. Is there any movement toward developing a single integrated projects list, coordinating the facilities requirements of the active duty Army, the Army National Guard, and the Army Reserve?

    Answer. No. We do not envision integrating the list of projects required by the Active, National Guard, and Reserve.

Advance Appropriations

    Question. The budget request includes a proposal to provide advance appropriations of $293,250,000 for fiscal year 2000, $189,500,000 for fiscal year 2001, and $72,300,000 for fiscal year 2002. What criteria were applied in determining which projects would receive advance appropriations?

    Answer. OSD directed that selected Army projects be funded based on advance appropriations. The basis for selecting projects was that all the funds required for the projects were not needed in the budget year for construction, and construction completion and delivery of the projects were not affected by this approach. All of the projects selected had construction periods of greater than two years. None of the projects can be split into smaller, complete and usable facilities.

    Question. What benefit is expected from advance appropriation?

    Answer. This approach permits us to finance more priority projects that otherwise could not have been funded if large sums of appropriations were tied up unnecessarily in single, large-scale projects. Advance appropriations also provides an assurance that the projects will be fully funded.
 Page 402       PREV PAGE       TOP OF DOC    Segment 2 Of 3  

    Question. What problems would be solved by this approach, and what problems would be created?

    Answer. OMB policy is that projects in the President's Budget will be fully funded up front. The request for advance appropriations permits the Army to comply with OMB policy and to pace appropriations with the expected placement of construction. Advance appropriations would ensure that all facilities would be complete and usable. We are not aware of any problems that this would create.

    Question. What was wrong with the approach taken in the past—that is, full authorization followed by incremental annual appropriations?

    Answer. OMB policy is that projects in the President's Budget will be fully funded up front.

Family Housing Privatization Through the Family Housing Improvement Fund

[''Capital Venture Initiative'' Program]

    Question. The Committee has received the notification for the family housing privatization project at Fort Carson, and it is under review. We will be going into detail about this project, and about the ongoing privatization program, when we have our hearing with Under Secretary Goodman on March 12. I have several questions that I will ask you to answer from the Army's perspective: Does the Army anticipate that there will be any financial savings from privatization as compared with the traditional program?
 Page 403       PREV PAGE       TOP OF DOC    Segment 2 Of 3  

    Answer. No savings are anticipated. The goal of Army privatization is to leverage appropriated funds and assets to revitalize and replace existing inadequate family housing that would not otherwise be revitalized or replaced using traditional MILCON funding. Based on current and projected funding levels, Army's current revitalization cycle is 130 years and the revitalization backlog continues to grow. By leveraging MILCON funds and other available assets (e.g., land and housing units) by at least three-to-one, the Army can revitalize and replace existing inadequate housing by the year 2010.

    Question. Is privatization becoming a substitute for the traditional family housing program, rather than the supplemental program that Congress originally intended?

    Answer. We see the privatization authorities as another tool for providing quality family housing. Our analyses show that we can leverage our scarce MILCON dollars to revitalize the inventory. Although we see promise for this approach, we realize not all housing will be privatized under these authorities and we will not always have some government family housing.

    Question. Is it correct that the cost of the family housing program is being shifted from this bill to the housing allowances subaccounts within the military personnel accounts in the National Security appropriations bill?

    Answer. For the housing units the Army retains, we will continue to require resources in the Army Family Housing appropriations. The Army will transfer family housing funds incrementally to the military personnel accounts as privatization projects are executed.
 Page 404       PREV PAGE       TOP OF DOC    Segment 2 Of 3  

    Question. Is this a ''zero-sum'' game, that is, will there be any savings or is it just a shift of costs?

    Answer. There are no savings, but by shifting costs and leveraging assets, the Army will be able to revitalize its inadequate inventory.

    Question. Is there any assumption about the future funding levels for housing allowances?

    Answer. The Army assumes that the funding levels for housing allowances will increase based on increases in housing costs tied to the geographic area.

    Question. Is it correct that the Army is holding up the execution of family housing projects for which funds have been appropriated, pending privatization efforts?

    Answer. Yes, we are holding several projects at locations where we are developing privatization packages. We expect to leverage the MILCON project funds to obtain an increase in the amount of revitalization at those specific locations by a ratio of at least 3 to 1.

    Question. Which projects are being held up, and what is the total dollar amount being withheld from obligation?

 Page 405       PREV PAGE       TOP OF DOC    Segment 2 Of 3  
    Answer.

Table 40


    The Army estimates that this $65.3 million may be leveraged to help revitalize all existing houses and eliminate the deficit at these five locations.

    Question. Are these funds being withheld to meet OMB scoring costs on mortgage guarantees to developers against base closure, downsizing, or extended deployments?

    Answer. Yes, and also to cover scoring costs on direct loans as applicable and to leverage enough private-sector capital to revitalize the existing inventories and buy out the deficits.

    Question. The Army is about to conduct a ''lessons learned'' procurement review of the Fort Carson proposal. Of course, that will be limited to events to date, and cannot offer any ''lessons learned'' from operational experience. The Army is proceeding very aggressively to execute additional ''whole installation'' type projects, involving Army contribution of land, facilities, infrastructure, and mortgage guarantees to developers and financiers. Wouldn't it be prudent to gain some experience with how well this program works, before making such a large commitment to turn over so many assets for a fifty year term?

    Answer. We believe we are moving forward in a very deliberate fashion, taking advantage of lessons learned at every step. These lessons are being applied to the development of projects at other participating installations but we are not rushing through these. We should start seeing some results from Fort Carson as well as from other Services' experiences before our next project.
 Page 406       PREV PAGE       TOP OF DOC    Segment 2 Of 3  

    Question. It took us many, many years to build up these family housing assets. Tell us about some of the steps the Army is taking to protect its investments under the Capital Venture Initiative program.

    Answer. While it took many years to build up the Army's family housing assets, inadequate and unpredictable appropriated funding streams caused these assets to severely deteriorate over the years. Under the Capital Venture Initiatives program, the Army anticipates that the existing inventory will be revitalized to meet the contemporary housing standards within a 10-year period. In addition, the contractor will be responsible for high quality maintenance and improvements to sustain the units over the life of the contract. Contract performance is being insured through a series of escrow accounts to be used in the event of contractor non-performance.

    Question. Provide for the record a list of the twenty-six locations that are under consideration for Capital Venture Initiative projects beyond the Fort Carson project, including the number of units in the current inventory at each location, as well as some indication of the value of the total government contribution at each location.

    Answer. The Army installations participating in the CVI Program are listed below. In the case of Fort Carson, the government contribution is $9.6 million as a guarantee to be held in a Treasury account for scoring purposes. There is no cash contribution to the contractor. Estimates for the other 26 locations have not yet been determined.
    "The Official Committee record contains additional material here."

Fort Carson, Colorado
 Page 407       PREV PAGE       TOP OF DOC    Segment 2 Of 3  

    [CLERK'S NOTE.—On April 22, 1998, the Department of the Army cancelled the proposed award of the whole-installation capitol venture initiative project at Fort Carson, Colorado. This contract would have been the first exercise of the authority sought by the Department of Defense and enacted in the National Defense Authorization Act for Fiscal Year 1996 on February 10, 1996 (section 2801 of Public Law 104–106, 10 USC 2871). The Army's decision was based upon litigation instituted in the U.S. Court of Federal Claims, and has resulted in a re-examination of the acquisition process. The Army is now studying corrective action alternatives,including a return to best and final offers and re-solicitation.]
    "The Official Committee record contains additional material here."

    Question. Outline for us how this proposal would work, and in which counties.

    Answer. The Overseas Housing Authority (OHA) would collect rents from the military occupants equal to their housing allowances. These rents would be deposited into a single non-appropriated fund (NAF) account that would be separate and distinct from any non-appropriated funds that support morale, welfare and recreation. The local installation would use funds from this account to pay normal operating and maintenance expenses that would otherwise be paid out of the family housing operations appropriation. With any excess available funds, the installation then can invest in major improvements and revitalization that would otherwise be paid out of the family housing construction appropriation. No new units will be purchased or constructed above those already authorized. There are several benefits to a NAF over the existing system: a stable, predictable income; no-year money; increased financial flexibility; no OMB scoring; and more simplified procurement procedures.
 Page 408       PREV PAGE       TOP OF DOC    Segment 2 Of 3  

    The legislation would provide the authority to carry out demonstration projects for up to five sites overseas (Germany and Korea). The pilot program will be in Mannheim, Germany. If it is successful, we plan to transition OHS to other sites.

    Question. Isn't this another effort to finance family housing under the allowances subaccounts in the National Security appropriations bill?

    Answer. This is an effort to get family housing operations into a more business-like environment. We will only transfer family housing program funds incrementally to the military personnel account as we implement this authority if the test legislation is approved and it proves to be successful.

    Question. I am not aware that Congress has ever reduced the amount that was requested for overseas family housing. Why is this additional authority necessary?

    Answer. The overseas housing program has suffered from insufficient funding at the same level as Army housing in the United States.

    Question. What savings are anticipated from this approach?

    Answer. There are no anticipated savings, however the authority provides a way to revitalize overseas housing at a faster rate than the 130-year revitalization cycle we are now experiencing.

 Page 409       PREV PAGE       TOP OF DOC    Segment 2 Of 3  
    Question. Is the problem in Europe and Korea a housing deficit, or is it the need to upgrade and maintain government owned or government leased units?

    Answer. The problem is not a deficit issue. It is the need to revitalize the existing inventory.

SOUTHCOM Headquarters

    Question. The Army's original plan was to relocate the headquarters of the Southern Command from Panama, with no requirement for military construction. In fact, part of the decision to go to Miami was based on the plan to lease space. What happened to change this plan, so that military construction funding is now requested?

    Answer. In the aftermath of the Khobar Towers bombing incident, force protection considerations required a security buffer zone around the building. The Army is committed to a 10-year firm-term lease on the headquarters building. However, when coupled with the proposed lease of a 19-acre security buffer are around the building, it was determined that approval by the Office of Management and Budget (OMB) and the Public Works Committees would be required. OMB felt that the combined leases for the building and land met its criteria and policy considerations requiring either purchase or a capital lease. Therefore, the Army included a request for $26,700,000 in its fiscal year 1999 budget for the purchase of both the headquarters and the land.

    Question. Is it correct that the Army currently occupies the SOUTHCOM headquarters building in Miami, under an interim lease?
 Page 410       PREV PAGE       TOP OF DOC    Segment 2 Of 3  

    Answer. Yes, that is correct.

    Question. When will the final lease be signed?

    Answer. The final lease is expected to be signed February 27, 1998.

    (NOTE: Subsequent to the hearing, the lease was signed on February 27, 1998)

    Question. Under the terms of the interim lease, what will happen if funds are not appropriated to purchase the building?

    Answer. If funds are not approved to purchase the building, the lease continues in force until a final lease is signed. Under the terms of the proposed final lease, we will be obligated to continue leasing for a 10-year period.

    Question. How is this possibility being addressed in the final lease?

    Answer. The proposed final lease is for a firm-term 10-year period and does not address purchase. We do not have authority to purchase and therefore would not be able to enter into negotiations or any written agreement related to purchasing the building.

    Question. Does the draft final lease address the need for a 200 foot security buffer, which requires approximately 19 acres of land?
 Page 411       PREV PAGE       TOP OF DOC    Segment 2 Of 3  

    Answer. No, it does not.

U.S. Military Academy, West Point, New York Cadet Physical Development Center

    Question. The budget request includes $12 million for the cadet physical development center at West Point. Outline for us the full scope and cost of this project.

    Answer. The total cost of the project is $85 million. It will be constructed in three phases. Phase I will be in fiscal year 1999 at $12 million. This will allow the construction and conversion of space for a temporary physical education facility to continue the physical education program during the construction period. It will also realign the utility services and enable the remainder of the Center to continue to function. After realigning the utilities, the central core of the facility will be demolished to include asbestos and lead paint mitigation.

    Phase 2 is scheduled in fiscal year 2000 at $29 million and it will allow us to construct various physical education areas in the footprint of the demolished section.

    Phase 3 is scheduled in fiscal year 2002 at $44 million and we will be able to demolish the central portion of the Center, continue construction of physical education areas, and expand Crandall pool along with the diving area.

Base Realignment and Closure: Cost of Base Closure

 Page 412       PREV PAGE       TOP OF DOC    Segment 2 Of 3  
    Question. When this Subcommittee took on the cost of base realignment and closure, we were assured that these resources would be returned to the military construction and family housing programs upon completion of BRAC. How has the Army benefited from this Program?

    Answer. The BRAC program has enabled the Army to dispose of excess infrastructure and consolidate functions resulting in increased efficiency. The savings generated by these actions will be applied to Army programs including military construction, housing, research, operations, and procurement. Units and activities that were relocated due to realignment or closure moved into modernized facilities at the gaining installations. Barracks and other facilities accounted for a substantial portion of the expenditures in the BRAC program.

Line Item Veto

    Question. The President's use of the line item veto on last year's bill took a total of fourteen projects for the active Army, the Guard, and the Reserve. The House overrode this veto on February 5, and the Senate will soon attempt override, perhaps today. Can you assure us that these projects will be executed promptly, if the Senate action is successful?

    Answer. Yes. Based on the Senate override, every effort is being made to award the projects this fiscal year.

Alabama-Anniston Army Depot Ammunition Containerization Complex ($3,550,000)

 Page 413       PREV PAGE       TOP OF DOC    Segment 2 Of 3  
    Question. The form 1390 for this project indicates that the fiscal year 2000 program will include a request for $7,000,000 for phase VII funding for the Anniston ammunition demilitarization facility. Why was this item not included in the request for advance appropriation?

    Answer. The fiscal year 2000 $7 million project is phase VII of the chemical demilitarization program at Anniston. The chemical demilitarization project at Anniston has already been fully authorized in a prior year. Since no appropriation is required for fiscal year 1999, it was not necessary to make an advance appropriations request this year.

    Question. Is it correct that the containerization complex is a current mission requirement and has nothing to do with the chemical demilitarization project?

    Answer. That is correct. The fiscal year 1999 $3.55 million ammunition containerization complex is part of the Army strategic mobility program to improve ammunition storage and shipping capability during deployment., This project is required for strategic mobility and has nothing to do with the chemical demilitarization project.

Alabama-Redstone Arsenal: Missile Software Engineering Annex Phase II ($13,600,000)

    Question. The existing building was built in 1986, but is now undersized and inadequate. How will this existing building be reutilized?

    Answer. The missile software engineering annex will be constructed directly adjacent to the existing software engineering facility. The existing facility will continue to be utilized to support the growing requirements for missile, aviation, and other weapon systems development. The facilities will be connected via enclosed walkway.
 Page 414       PREV PAGE       TOP OF DOC    Segment 2 Of 3  

Arkansas-Pine Bluff Arsenal: Ammunition Demilitarization Facility Phase III ($16,500,000)

    Question. What is the current status of the first two phases of work which were funded in fiscal year (FY) 1995 and 1997?

    Answer. Construction of the demilitarization facility has not begun. A construction contract for the full scope of the demilitarization facility (all phases) was awarded on July 25, 1997. The funds appropriated for Phase I in FY 1995 and Phase 2 in FY 1997 were obligated against the contract with the exception of $10.9 million. Subsequent to contract award, a protest was filed with the GAO by one of the unsuccessful offerors, and according to regulations, a stop work order was immediately issued on August 8, 1997. This effectively prevented the Contractor from starting work on the contract. GAO supported the unsuccessful offeror and recommended that the Army make a new source selection decision for the solicitation. This new source selection decision is expected shortly. Until the successful offeror is identified and, if necessary, the contract is adjusted, the stop work order prevention construction will remain in effect.

California-Fort Irwin: Heloport Phase II ($7,000,000)

    Question. What is the estimated initial operation capability date for the use of the Barstow-Daggett heliport?

    Answer. April 2000.

 Page 415       PREV PAGE       TOP OF DOC    Segment 2 Of 3  
    Question. Upon completion of this project, what will become of the on-post Bicycle Lake Airfield?

    Answer. This airfield will continue to be used to support fixed wing, air assault aircraft (C–30, C–17), and rotational attack helicopters. The airfield has no maintenance facilities available.

    Question. Currently, the helicopter repair function is being performed by Lockheed, a contract maintenance service provided at leased facilities. Upon completion of this project, what annual savings will be generated by avoiding such costs?

    Answer. Lockheed will continue performing maintenance in the new facilities. The cost to lease the existing facilities is being funded by the Army, no by the contractor. Negotiations between the Army and San Bernadino County are not yet finalized on the cost of the long-term lease for the land at Barstow-Daggett airfield (including charges for utilities and roads).

Florida-SOUTHCOM Headquarters: SOUTHCOM Headquarters and Land Acquisition ($26,700,000)

    Question. Submit for the record a copy of the current lease agreement, highlighting provisions regarding annual payments subject to the availability of funds and regarding lease termination provisions.

    Answer. A copy of the current lease is hereby provided as requested. Provision regarding annual payments subject to the availability of funds is highlighted. There is no provision for termination of the lease for the convenience of the Government. Other circumstances warranting termination of the lease are highlighted. The interim lease and the final lease are both provided below.
 Page 416       PREV PAGE       TOP OF DOC    Segment 2 Of 3  
    Offset folios 1015 to 1066 insert here

    Question. What is the annual payment under the existing ten year fixed term lease?

    Answer. The base lease on the headquarters building is currently $1,749,393 per year.

    Question. Does the current lease agreement include buyout provisions? If so please highlight the relevant portion for the record.

    Answer. The lease does not include a buyout provision.

    Question. What is the basis for the estimated $16,700,000 cost for purchase of the existing leased facility including nine acres of land, and for the estimated $10,000,000 cost of an additional 19 acres of land?

    Answer. The $26,700,000 estimate was developed in September/October 1997, from data which included the owner's cost of the original two tracks of land on which the building is sited (10.4 acres), the owner's capital investment in the building to date, the estimate for the remaining improvements and the estimate to acquire the additional land needed for a security buffer. The numbers were first calculated in May 1997, based on then current data, in coordination with the building owner and GSA Central Office for the purpose of preparing a Lease Prospectus to submit to Congress. The original estimate was approximately $25,300,000. This was updated to $26,700,000, based on general knowledge of the volatile Miami market and development in the local area of the headquarters.
 Page 417       PREV PAGE       TOP OF DOC    Segment 2 Of 3  

    Question. What are the results of the appraisal that was scheduled for completion this month?

    Answer. The appraisal has not been completed yet.

    Question. Wouldn't price escalation argue for continuing the current ten year fixed term?

    Answer. We have included the request for purchase in the Army's fiscal year 1999 budget based on direction from the Office of Management and Budget.

    Question. Why wasn't a force protection buffer zone included in the original lease?

    Answer. At the time the original commitment to lease was signed, the Khobar Towers and Oklahoma City incidents had not occurred and a requirement for a large security buffer was not envisioned. Subsequent to those two incidents, in November 1996, the Defense Special Weapons Agency conducted a force protection assessment of the headquarters building and surrounding area. It determined that various security enhancements were required, among them the establishment of a 200 foot security buffer around the building.

    Question. Is the additional land available, and are there willing sellers/lessors?

 Page 418       PREV PAGE       TOP OF DOC    Segment 2 Of 3  
    Answer. Yes, the additional land is available. To the best of our knowledge, the owners appear to be willing to consider selling.

    Question. What would it cost annually to lease this additional land as a separate action for a ten year term?

    Answer. The exact cost would be based on negotiations with the owners. However, we estimate that the cost would be approximately $847,100 annually.

    Question. When the Army entered into the existing lease, there was no apparent hesitation regarding dependence on a single contract provider for the facility space and operations/maintenance/repairs. Construction was completed and occupancy was taken in September 1997. What has changed in the last six months to cause this to be a concern.

    Answer. We do not see this as a concern.

    Question. If no funds are appropriated for purchase of either the currently leased building or for the nine acres of land that the building and appurtenances occupy, what impact would this have on the terms of the existing lease?

    Answer. It would have no impact on the current lease.

    Question. What is the current status of lease family housing in the Miami area for SOUTHCOM leadership personnel?

 Page 419       PREV PAGE       TOP OF DOC    Segment 2 Of 3  
    Answer. Eight houses have been leased for SOUTHCOM leadership personnel.

    Question. If the precedent is set for establishing a 200 foot security buffer for this facility, how big is the Army's backlog of such force protection requirements?

    Answer. This situation is a special case, since the headquarters building is not a military installation. It is not considered a model for security requirements for other locations. The Army has no other current requirements for similar security buffers. However, as new construction is proposed, the Army will review the concept design for anti-terrorism/force protection considerations.

Georgia-Fort Benning: Whole Barracks Complex Renewal ($28,600,000)

    Question. This project includes demolition of ten buildings totaling 27,313 square meters and new construction totaling 15,469 square meters. What is the total demolition cost included in this project?

    Answer. The total demolition cost is $1,101,000.

    Question. The form 1391 mentions required ''protection of historic landscape features.'' What does this involve?

    Answer. Protecting historic landscape means the contractor will need to take special precautions not to damage established features like stone or brick walks, trees, and other plantings.
 Page 420       PREV PAGE       TOP OF DOC    Segment 2 Of 3  

    Question. To what extent is this project made difficult by considerations within the Historic District of Fort Benning?

    Answer. To address the historic considerations, additional coordination with the State Historic Preservation Officer (SHPO) is required in addition to the protection of historic landscape features in the historic district. Coordinating with the SHPO is normal for any construction project on Fort Benning, given the age of the facilities. The installation does not foresee any peculiar problems with this effort.

Hawaii Schofield Barracks: Whole Barracks Complex Renewal ($47,500,000)

    Question. Describe the environmental remediation that is required at this construction site.

    Answer. A portion of this project is built on areas previously occupied by Directorate of Public Works (DPW) facilities. Environmental remediation, $250,000 is needed for site where underground storage tanks (UST) were previously removed. It is also needed for clean up of areas due to pesticide storage and PCB storage contamination of concrete slabs and surrounding soil.

    Question. Only about one-third of the cost of this project is for barracks, with the balance financing other facilities. Is it correct that this project will provide barracks for only 192 personnel?

 Page 421       PREV PAGE       TOP OF DOC    Segment 2 Of 3  
    Answer. Yes. This project is the last phase of the Infantry Brigade Complex. The entire complex will provide for a maximum utilization of 980 soldiers. Most of the barracks were built during early phases to provide upgraded housing for our soldiers quickly. Nine of the 13 required company operations facilities and all three battalion headquarters are built in this last phase.

    Question. Why were the soldier community building, the company operations facilities, and the battalion headquarters combined in this last phase of the Infantry Brigade Complex?

    Answer. Most of the barracks were constructed during earlier phases in the sequence of building facilities to get soldiers out of existing substandard barracks faster. Two other soldier community buildings were previously built in the earlier phases to support barracks built during those phases. The remaining support facilities are required to complete this Brigade Whole Barracks Complex.

Illinois-Rock Island Arsenal: Electrical Distribution System ($5,300,000)

    Question. The Committee encourages the submission of projects such as this, to address infrastructure requirements at existing facilities. This project will correct deficiencies in the electrical distribution system of the building that supplies forty percent of the Army's mainframe computer processing. How will operations continue while this work is being accomplished?

    Answer. The work will be done in phases. For each phase, after the new segment of the improved distribution is completed, a power outage will be scheduled. The switchover to the new system will occur during the outage. All switchovers will be scheduled for weekends, when less processing is done.
 Page 422       PREV PAGE       TOP OF DOC    Segment 2 Of 3  

    Question. Upon completion of this project, will this building be structurally sound and fully adequate to meet its requirements?

    Answer. Yes.

Indiana-Crane Army Ammunition Activity: Ammunition Containerization Complex Phase II ($7,100,000)

    Question. Why was this project divided into two phases?

    Answer. This project provides a second Ammunition Container Complex at Crane Army Ammunition Activity. After the programming of the initial phase, increased outloading requirements were identified due to U.S. Army Material Command streamlining efforts. A second phase was added to meet the increased outloading requirement.

    Question. Will the first phase provide complete and usable facilities, and is it on schedule?

    Answer. Yes, the first phase will provide a complete, separate and usable ammunition container complex. The second phase also will provide a complete, separate and usable ammunition container complex. Each project is separately sited on Crane Army Ammunition Activity. The first phase is on schedule, design is complete, and project award is anticipated for 1 August 1998.

Indiana—Newport Army Ammunition Plant: Ammunition Demilitarization Support ($2,000,000)
 Page 423       PREV PAGE       TOP OF DOC    Segment 2 Of 3  

    Question. Would this work be required if there were no chemical demilitarization program?

    Answer. No, this project would not be required.

    Question. If not, why isn't this project combined with the ''Phase I'' demil project?

    Answer. The support work is physically separate from the containment area of the demilitarization facility and can be accomplished more cost effectively as a separate contract to a small contractor. In addition, parts of the support facilities are required prior to construction commencing of the main facility; i.e., access roads and vehicle parking lot.

Indiana-Newport Army Ammunition Plant: Ammunition Demilitarization Facility Phase I ($27,500,000)

    Question. This project differs greatly from the earlier chemical demilitarization projects, in that it will provide a pilot test of an alternative to incineration. As such, your estimates are based upon the best available data and costs are adjusted for risk associated with design and construction of a first-of a kind plant. Describe for us the precautions you will take to protect the government from cost overruns in construction by cost-reimbursable design-build contract, beyond merely providing ten percent for contingency.

    Answer. The U.S. Army Corps of Engineers (USACE) Resident Engineer's Office will be staffed with qualified personnel and will carefully monitor and manage the design-build contract, utilizing ''Earned Value Management System'' procedures.
 Page 424       PREV PAGE       TOP OF DOC    Segment 2 Of 3  

    The contract requirements have been written to require the contractor to complete the design for each construction package prior to start of construction of the particular construction package. Part of the requirement is for the contractor to provide an intermediate design with an associated detailed cost estimate and a final design package with a detailed cost estimate. The Government will thus have two opportunities to review, evaluate and agree or disagree with the cost estimate. Once the Government is satisfied with the final design and cost estimate, the design package will be authorized for construction.

    As part of ''Earned Value Management,'' the USACE resident office will continuously evaluate the cost and schedule performance of the contractor and take appropriate management measures to require compliance and performance.

    The contract will be structured as cost plus award fee. The award fee criteria will include evaluation of cost and schedule performance.

    Question. The total cost for this construction project is estimated to be $189,500,000, plus an additional $545,500,000 in equipment to be procured from the R&D account. Will the contract include provisions for termination for the convenience of the government under any circumstances?

    Answer. Yes, the contract will include a clause, termination for convenience of the government.

Kansas—Fort Leavenworth: U.S. Disciplinary Barracks, Phase II ($29,000,000)
 Page 425       PREV PAGE       TOP OF DOC    Segment 2 Of 3  

    Question. Phase III construction to complete this project is programmed for fiscal year 2000. Describe how the second and third phase of this complex project have been divided.

    Answer. Based on full authorization in the fiscal year 1998 budget, the project will be awarded under a single contract. To minimize any impact to the contractor due to the incremental funding, the contract will not define what work is to be performed in each funding increment. Rather, the contractor is limited by the amount appropriated. The incremental funding clause of the Defense Federal Acquisition Regulation Supplement (DFARS) allows the government to identify funds availability and the contractor's responsibility regarding exceeding that amount prior to additional funds becoming available.

    Question. Will the existing disciplinary barracks continue to operate until all three phases of this replacement are complete.

    Answer. Yes, the existing barracks will be used until the entire project is complete.

    Question. The Army is working to get out of the utility business. Why does this project include a centralized utility plant with gas-fired boilers?

    Answer. The broiler plant associated with the new disciplinary barracks provides central heating and air conditioning support to the complex much the same way as a furnace operates in a house. A boiler plant is not part of the DOD-wide utilities privatization initiative since it's main mission is heating and cooling rather than power generation.
 Page 426       PREV PAGE       TOP OF DOC    Segment 2 Of 3  

    Question. Do the Army and the Department remain confident that a 512 person capacity for maximum security confinement will meet the requirement?

    Answer. Yes, we are confident that 512 person capacity will meet the requirement for maximum security confinement.

    Question. What is the estimated cost of laying away the existing confinement facility, and is that cost included in this replacement project?

    Answer. The Army plan is to demolish the existing facility pending historical and environmental concerns. Final costs for the disposition of the disciplinary barracks are not included in the project and will not be known until completion of the studies.

    Question. Is there any plan to reutilize the existing facility?

    Answer. Due to the age and condition of the existing disciplinary barracks, there is no plan to reuse the facilities. Because the National Park Service and the Advisory Council of Historic Preservation have designated the disciplinary barracks a significant historic structure, an adaptive reuse/demolition study is required by the National Historic Preservation Act of 1966 and the National Environmental Policy Act of 1994. These studies/plans are being developed to ensure all legal procedural requirements are met prior to the final disposition of the disciplinary barracks.

Kentucky—Blue Grass Army Depot: Ammunition Containerization Complex ($5,300,000)
 Page 427       PREV PAGE       TOP OF DOC    Segment 2 Of 3  

    Question. Would this work be required if there were no chemical demilitarization program?

    Answer. Yes. The ammunition containerization complex project is a strategic mobility requirement that provides facilities to outload munitions during a major contingency. It is a separate mission and has nothing to do with the chemical demilitarization mission.

    Question. Is there any relationship between this project and the ''Phase I'' chemical demilitarization project that is programmed for fiscal year 2000?

    Answer. No. This project supports the Army's strategic mobility program. There is no relationship between this project and the chemical demilitarization project.

Kentucky—Fort Campbell: Whole Barracks Complex Renewal ($41,000,000)

    Question. The form 1391 states that the total estimated design cost for this project will be $3,400,000. How much was saved on this project by using a standard or definitive design?

    Answer. We expect approximate savings of $450,000, due primarily to the use of a standard or definitive design.

    Question. The form 1391 states that this is the first of three phases, but the form 1390 shows a barracks project in the fiscal year 2000 program, and no further projects planned for the next three program years (''new mission only''). When is the third phase programmed, and at what cost?
 Page 428       PREV PAGE       TOP OF DOC    Segment 2 Of 3  

    Answer. Phase III of the project is currently programmed in fiscal year 2002 at $36,000,000, in the current Future Year Defense Program (FYDP). The form 1390 shows only new mission projects in the out years.

    Question. Will each of the three phases provide complete and usable facilities for an entire barracks area, balanced between actual barracks and other types of facilities—unlike this year's project at Schofield Barracks?

    Answer. Yes. Each phase will provide complete and usable facilities, and is balanced in the type facilities provided. Each phase includes the following type facilities as part of the project scope: barracks, soldier community buildings, battalion headquarters, and company operations facilities. The brigade headquarters and brigade dining facility are part of the first phase.

Maryland—Aberdeen Proving Grounds: Ammunition Demilitarization Support ($1,850,000)

    Question. Would this work be required if there were no chemical demilitarization program?

    Answer. No, this project would not be required.

    Question. If not, why isn't this project combined with the ''Phase I'' demil project?

 Page 429       PREV PAGE       TOP OF DOC    Segment 2 Of 3  
    Answer. The support work is physically separate from the containment area of the demilitarization facility and can be accomplished more cost effectively as a separate contract to a small contractor.

Maryland—Aberdeen Proving Ground: Ammunition Demilitarization Facility Phase I ($26,500,000)

    Question. This project differs greatly from the earlier chemical demilitarization projects, in that it will provide a pilot test of an alternative to incineration. As such, your estimates are based upon the best available data and costs are adjusted for risk associated with design and construction of a first-of a kind plant. Describe for us the precautions you will take to protect the government from cost overruns in construction by cost-reimbursable design-build contract, beyond merely providing ten percent for contingency.

    Answer. The U.S. Army Corps of Engineers (USACE) Resident Engineer's Office will be staffed with qualified personnel and will carefully monitor and manage the design-build contract, utilizing ''Earned Value Management System'' procedures.

    The contract requirements have been written to require the contractor to complete the design for each construction package prior to start of construction of the particular construction package. Part of the requirement is for the contractor to provide an intermediate design with an associated detailed cost estimate and a final design package with a detailed cost estimate. The Government will thus have two opportunities to review, evaluate and agree or disagree with the cost estimate. Once the Government is satisfied with the final design and cost estimate, the design package will be authorized for construction.

 Page 430       PREV PAGE       TOP OF DOC    Segment 2 Of 3  
    As part of ''Earned Value Management'', the USACE resident office will continuously evaluate the cost and schedule performance of the contractor and take appropriate management measures to require compliance and performance.

    The contract will be structured as cost plus award fee. The award fee criteria will include evaluation of cost and schedule performance.

    Question. The total cost for this construction project is estimated to be $184,500,000, plus an additional $62,593,000 in equipment to be procured from the R&D account. Will the contract include provisions for termination for the convenience of the government under any circumstances?

    Answer. Yes, the contract will include a clause, termination for convenience of the government.

    Question. Both this project and the project at Newport, Indiana will utilize neutralization followed by biodegradation. Will the two plants be substantially identical?

    Answer. The two plants, the Aberdeen Chemical Demilitarization Facility (ABCDF) and the Newport Chemical Demilitarization Facility (NECDF) are not substantially identical. While both plants will use neutralization-based primary treatment processes, the treated agent and reaction chemistry of these primary treatment processes are substantially different for each facility. The ABCDF will use aqueous hydrolysis of HD mustard agent and the NECDF will use caustic hydrolysis of VX nerve agent. Additionally, the secondary post-treatments at these facilities are completely different technologies: the ABCDF wil use biodegredation to treat the HD mustard agent hydrolysate and the NECDF will use Super Critical Water Oxidation to treat the VX nerve agent hydrolysate.
 Page 431       PREV PAGE       TOP OF DOC    Segment 2 Of 3  

    Question. If so, why not execute one first, to benefit from ''lessons learned'' before proceeding with the second plant?

    Answer. Since the treatment processes of the ABCDF and NECDF are dissimilar (as noted in the previous question), i.e., different agents and different post-treatment approaches, there is very limited value in delaying the execution of one of the facilities to benefit from ''lessons learned'' at the other. Additionally, executing these two projects in series would increase the project schedules, increase the total cumulative risk due to longer stockpile storage, and not produce substantive experiences that could be applied to the follow-on facility.

Maryland—Fort Detrick: Physical Fitness Training Center ($3,550,000)

    Question. This project is conjunctively funded with the BRAC account and the MilCon account, with the MilCon account providing 54 percent of the total construction cost. How was ratio between the two accounts determined?

    Answer. You are correct, the project is conjunctively funded between MilCon Program Number (PN) 046358 (Physical Fitness Training Center) Program Amount (PA) $3,550K and Base Realignment and Construction (BRAC) PN 48153 (Physical Fitness Training Center) PA $3,050K. The total cost of the project is $6,600K. The split of construction funds was based on the existing military and dependent populations at Fort Detrick (1330 people) and the BRAC military and dependent population from Fort Ritchie (1147 people). The total population is 2,477 people. This gives a ratio of 1330/2477 or 54% for MCA and 1147/2477 or 46% for BRAC.
 Page 432       PREV PAGE       TOP OF DOC    Segment 2 Of 3  

    Question. When was the existing gym condemned as a result of structural storm damage?

    Answer. The existing gym was condemned on 27 November 1995.

    Question. Why wasn't a new start reprogramming submitted, citing existing authority to replace damaged or destroyed facilities?

    Answer. To replace the existing damaged facility under 10 USC 2854 would have required a higher priority project to be canceled in-order to fund the replacement gym.

Missouri—Fort Leonard Wood: Engineer Qualification Range ($5,200,000)

    Question. Was there an engineer qualification range at Fort Belvoir that met this requirement before the relocation of the engineer school to Fort Leonard Wood?

    Answer. No. This is a new range based upon new requirements.

    Question. Why can't the design of this project be performed entirely in-house, perhaps at the engineer school itself?

    Answer. Project design could not have been given to the Fort Leonard Wood Director of Public Works or the Engineer School since neither organization has the excess capability, nor are they set up to do military construction design work. The U.S. Army Corps of Engineers, Kansas City District, is performing the design. The District is working very closely with the Engineer School to make sure the range meets the school's expectations.
 Page 433       PREV PAGE       TOP OF DOC    Segment 2 Of 3  

New York—United States Military Academy Cadet Physical Development Center ($12,000,000)

    Question. Please submit for the record one or more forms 1391 that describe the full scope of work for all phases of this project.

    Answer. Three DD Forms 1391 are provided below which describe the scope of work in each phase.
    "The Official Committee record contains additional material here."

New York—United States Military Academy Cadet Physical Development Center ($12,000,000)

    Question. What will be the final cost per square foot for this entire project, and how does this compare with other recently constructed physical development centers?

    Answer. The requirement for the physical development center if 42,033 square meters (452,439) square feet) and at $85 million averages $188 per square foot. If compares favorably in size, scope, and cost with existing facilities at other military academies and universities.

    Question. How does the cost per square foot compare with the physical fitness center at Fort Detrick, Maryland that is requested this year?

    Answer. The physical fitness center at Fort Detrick is 3,479 square meters (37,448 square feet) and at $6.6 million averages $178 per square foot.
 Page 434       PREV PAGE       TOP OF DOC    Segment 2 Of 3  

    Question. How is this project connected with athletic conference facility requirements?

    Answer. There are no program spaces within this project that are exclusively used for NCAA competition or training. The primary purpose of all program space within the cadet physical development center is to promote cadet physical development instruction. Although the primary use is cadet physical development instruction, shared use for cadet varsity athletic teams (NCAA sports) is considered in the functional design layout to meet sanctioned NCAA size requirements.

    Question. Submit for the record the economic analysis that has been prepared in evaluating this project.

    Answer. The economic analysis is provided below.
    "The Official Committee record contains additional material here."

North Carolina—Fort Bragg: Whole Barracks Complex Renewal ($47,000.000)

    Question. Will this project replace the worst existing barracks at Fort Bragg?

    Answer. Yes. The installation has identified the barracks in this area as the facilities most in need of repair. This project replaces hammerhead style barracks constructed between 1951–1953.
 Page 435       PREV PAGE       TOP OF DOC    Segment 2 Of 3  

    Question. If not, why were these barracks selected for replacement?

    Answer. Not applicable. These barracks were most in need of repair.

    Question. Submit for the record a list of the planned projects to correct all existing barracks deficiencies at Fort Bragg.

    Answer. The fiscal year 1999–2003 Future Year Defense Program (FYDP) identifies whole barracks complex renewal projects for Fort Bragg in each year of the FYDP. Those projects are identified below, starting with fiscal year 2000. The form 1391 identifies the remaining permanent party barracks requirement at Fort Bragg as 7,781. Of this amount, 4,894 will be handled by Military Construction, Army (MCA) funds and the remainder through the operations and maintenance, Army funded Barracks Upgrade Program. The projects identified below will take care of 1,960 of the MCA requirement. The remaining MCA requirement will be programmed in subsequent years to achieve our goal to buyout barracks to the Department to Defense 1+1 standard by 2008.

Table 41



North Carolina—Fort Bragg: Deployment Staging Complex ($30,000,000)

    Question. To what extent is it necessary to coordinate this project with the hydrant fuel system project at Pope Air Force Base that is requested this year?

    Answer. While Fort Bragg coordinates with Pope Air Force Base on projects and joint concerns on a weekly basis, these two particular projects have no impact on one another, except that they both support the deployment of military forces from Pope Air Force Base.
 Page 436       PREV PAGE       TOP OF DOC    Segment 2 Of 3  

    Question. Are these facilities being sized to accommodate C–5s, C–130s, C–141s, and C–17s?

    Answer. All of the outload facilities for personnel, equipment, and cargo at Fort Bragg are planned to accommodate the Army's strategic mobility requirements using all types of the Air Force aircraft or commercial aircraft to meet both the strategic mobility airdrop and airland scenarios.

    Question. Describe the dud clearance that is required.

    Answer. A portion of the complex will be located on a site that was formerly used as a training range. This site is required to be cleared of all unexploded or dud ordinance prior to construction on the site.

    Question. What is the total cost of the entire outload complex, by fiscal year?

    Answer. The total cost of the entire complex is estimated at $95.7 million. Phases by fiscal year are:

    Phase 1a in fiscal year 1996 for Departure/Arrival Staging Area is $12.7 million.

    Phase 1b in fiscal year 1999 completes Departure/Arrival Staging Area is $30.0 million.
 Page 437       PREV PAGE       TOP OF DOC    Segment 2 Of 3  

    Phase 2 in fiscal year 2000 for Heavy Drop Rigging Facility is $30.0 million.

    Phase 3 in fiscal year 2001 for Ammunition Holding Area is $13.0 million.

    Phase 4 in fiscal year 2002 for Petroleum, Oils and Lubricants (POL) storage complex is $10.0 million.

Oklahoma—McAlester Army Ammunition Plant: Ammunition Containerization Complex ($10,800,000)

    Question. By how much do the exiting facilities fail to meet the requirements?

    Answer. Current facilities can only outload 58 munitions containers per day against a requirement of 400 containers per day to support the Army's strategic mobility program. The existing facilities fail to meet the strategic mobility outloading requirements by 342 containers per day.

    Question. Will the existing facilities be incorporated into this new complex?

    Answer. No. The existing facilities are dispersed throughout the installation. However, they will be used to outload 58 munitions containers per day. The requested project will construct new facilities to handle the balance of the strategic mobility requirement to outload 342 munitions containers per day.
 Page 438       PREV PAGE       TOP OF DOC    Segment 2 Of 3  

    Question. If so, will they be modernized within the cost of this project?

    Answer. No. The existing facilities will not be modernized within this project. Existing facilities are being maintained and repaired through Operations and Maintenance. Army (OMA) projects identified within the Army's strategic mobility program.

    Question. If not, what is the planned utilization of the existing facilities?

    Answer. McAlester will continue to utilize its existing facilities to outload 58 munitions containers per day.

    Question. Briefly describe the ammunition infrastructure improvement project that is programmed for fiscal year 2000.

    Answer. This project will construct approximately 3.5 miles of railroad track in the container rail yard to accommodate the stockpile of flat railcars required to be on hand in case of a major contingency. In addition, this project will include construction of new magazine roads and maintenance and repair of exiting magazine roads accessing approximately 66 ammunition magazines.

Oklahoma—Fort Sill: Tactical Equipment Shop Phase I ($13,800,000)

 Page 439       PREV PAGE       TOP OF DOC    Segment 2 Of 3  
    Question. How many phases are there to this project, and at what cost in which fiscal years?

    Answer. This is the first of a 12 phase effort to provide adequate motor pools for Fort Sill's major tactical unit, III Corps Artillery, which does not currently have one single motor pool that meets its requirements. Each complete and usable phase will support either two battalions, or one battalion. Phase II is in fiscal year 2002 at $13,000,000; and Phase III is in fiscal year 2003 at $14,600,000. Subsequently phases are in the Army's Long Range Program. The total cost of all 12 phases is $165,000,000.

    Question. Why were advance appropriations not requested for the future phases?

    Answer. Advanced appropriations were not required because each phase will provide a complete and usable facility.

    Question. Will this first phase provide a complete and usable facility?

    Answer. Yes.

Oklahoma—Fort Sill: Whole Barracks Complex Renewal ($20,500,000)

    Question. Will this project renovate existing buildings, or demolish the existing structures and provide replacement facilities.
 Page 440       PREV PAGE       TOP OF DOC    Segment 2 Of 3  

    Answer. The project builds new barracks and a soldier community building (SCB) in an area of the installation where World War II wood facilities were demolished previously. Once construction is complete, the installation plans to convert the Old Hammerhead style barracks over to operations facilities. The additional heating, ventilation, and air conditioning (HVAC) for the SCB, identified on the form 1391, will be for the existing SCB, constructed as part of a fiscal year 1996 military construction project. The additional HVAC supports the barracks modules constructed in this project which are supplied with HVAC from the SCB.

    Question. If it is a demolition/replacement project, what is the estimate for demolition that is funded in this project?

    Answer. There is no demolition funded in this project because demolition has already been accomplished in the project's construction footprint.

    Question. If it is a renovation, what engineering analyses have been performed to assure that the existing buildings, built in 1954 and remodeled in 1975, are structurally sound?

    Answer. The project does not involved renovation.

    Question. What accounts for the increase in cost of $3,500,000 for this project from last year?

 Page 441       PREV PAGE       TOP OF DOC    Segment 2 Of 3  
    Answer. The original project was included in the second year of the fiscal year 1998/1999 biennial budget submission. At that time the cost was not yet based on a parametric estimate. The parametric estimate is now complete, and the budget request was adjusted accordingly.

Oregon-Umatilla Depot Activity: Ammunition Demilitarization Facility Phase IV ($50,950,000)

    Question. The Umatilla chemical demilitarization project is a site-adapted standard design. The form 1391 indicates that the total design cost was $11,410,000, equally divided between contract and in-house design, compared with the total project cost of $187,000,000. Is that the design-cost of the entire Umatilla project, or just phase IV?

    Answer. The design cost identified is the total design cost of the entire project, not just Phase IV alone.

    Question. If that is the design cost for the entire project, that comes to 6.1% for design. How does that compare with the design cost for the chemical demilitarization plants at Johnston Island, Tooele, and Anniston?

    Answer. The total design costs including in-house management costs, architect and engineer design costs, and U.S. Army Corps of Engineers District design costs for Johnston Atoll were $5,200,000 or 11%; Tooele were $13,596,000 or 7.55%; for Anniston $13,620,000 or 8.5%.

Texas—Fort Hood: Railhead Facility, Phase I ($17,500,000)
 Page 442       PREV PAGE       TOP OF DOC    Segment 2 Of 3  

    Question. Why is this project divided into two phases?

    Answer. This $32.5 million project was divided into two phases because the construction duration exceeded 24 months and only $17.5 million could be placed in the first year. Advance appropriations are also requested in the amount of $15 million in fiscal year 2000 because this project cannot be divided into separate complete and usable projects.

    Question. Will there be any difficulty continuing to operate and meet current mobility requirements while this project is being executed?

    Answer. No. Operations on the existing rail loading facility will not be affected during construction of the new railhead. The new railhead will be sited away from the existing railhead.

    Question. What is the estimated initial operational capability date for the completed new railhead?

    Answer. The estimated operational capability date for the project is the 3rd quarter, fiscal year 2001.

Texas-Fort Sam Houston: Whole Barracks Complex Renewal ($21,800,000)

    Question. The form 1390 shows that a dining facility is included in the fiscal year 2000 program. Will that facility service soldiers housed in these barracks?
 Page 443       PREV PAGE       TOP OF DOC    Segment 2 Of 3  

    Answer. No. Soldiers in these barracks will use an existing dining facility adjacent to the project. The dining facility project indicated on the form 1390 will service soldiers housed in an area approximately 3 miles away from this barracks project. The dining facility was originally programmed for fiscal year 2000. Subsequent to the preparation of the form 1390, the project was placed in fiscal year 2001 based on revised Army priorities.

    Question. If so, why isn't it included in this project?

    Answer. The dining facility is not associated with this project.

    Question. The form 1391 shows that this project will use a standard or definitive design that was most recently used at Fort Jackson. However, the total design cost will be $3,600,000, or 16.5% of the project cost. Why is the cost of a standard design so high?

    Answer. The estimated design cost on the form 1391 is in error. The U.S. Army Corps of Engineers has determined that the current design cost for this project is estimated at $1,440,000.

Utah—Toole Army Depot: Ammunition Containerization Complex ($3,900,000)

    Question. By how much do the existing facilities fail to meet the requirement?

 Page 444       PREV PAGE       TOP OF DOC    Segment 2 Of 3  
    Answer. Current facilities can only outload 20 munitions containers per day against an outloading requirement of 310 containers per day to support the Army's strategic mobility requirement. The existing facilities fail to meet the strategic mobility requirement by 290 munitions containers per day.

    Question. Will the existing facilities be incorporated into this new complex?

    Answer. Yes. This project will expand the two existing outloading sites to meet the strategic mobility requirement.

    Question. If so, will they be modernized within the cost of this project?

    Answer. Yes. The cost to modernize the existing sites is included in the cost of this project.

    Question. If not, what is the planned reutilization of the existing facilities?

    Answer. We plan to use the existing facilities for outloading of munitions.

Virginia—Charlottesville: National Ground Intelligence Center (NGIC) Facility (46,200,000)

 Page 445       PREV PAGE       TOP OF DOC    Segment 2 Of 3  
    Question. This project falls under the Military District of Washington. Does the Army consider this to be a new installation, or is it a subpost of Fort Belvoir?

    Answer. Charlottesville is a satellite installation of Fort Belvoir.

    Question. The remarks section of the DD Form 1390 states that ''the estimated cost to remedy the deficiencies in all existing permanent and semi-permanent facilities at this installation is $157,504K based on the Installation Status Report information on conditions as of October 1996''. To what installation does this information apply . . . Fort Belvoir or the existing GSA-owned facilities used by NGIC?

    Answer. Fort Belvoir.

    Question. The 1390 shows an area construction index of 0.00. Does the Army have a basis for an index for the Charlottesville area?

    Answer. Since there is no specific index for Charlottesville, the average for Virginia would be used—0.90.

    Question. Can all of this construction be put in place during fiscal year 1999?

    Answer. No, this project will require 30 months for construction.

 Page 446       PREV PAGE       TOP OF DOC    Segment 2 Of 3  
    Question. If not, why was this project not phased?

    Answer. Phasing was not an option because the project is one building which can not be divided into complete and usable phases. Additionally, incremental funding, with advance appropriations, was not feasible because the project was funded with National Foreign Intelligence Program funds, which are fenced by Executive Order and under the control of the Director of Central Intelligence.

    Question. What is the cost of the access road to the building site?

    Answer. There is no cost to the Government. As part of the purchase agreement, the original landowner agreed to construct, maintain, and turn over to the Virginia Department of Transportation the access road to our site. The site is adjacent to Route 29, 10 miles north of Charlottesville.

    Question. Does the Army hold title to the land on which this project will be built?

    Answer. Yes.

    Question. Upon completion of this project, will all existing facilities be turned back to GSA?

    Answer. Yes.
 Page 447       PREV PAGE       TOP OF DOC    Segment 2 Of 3  

    Question. What is the nature and extent of GSA's involvement in this construction project?

    Answer. There is no GSA involvement in this project. At one time this was a GSA project. They were involved in the site selection and schematic study in 1995 and 1996 but were unsuccessful at getting the project budgeted.

Virginia-Fort Eustis: Whole Barracks Complex Removal ($36,531,000)

    Question. The form 1391 for this project states that ''upon completion of this project, the remaining permanent party requirement is 962 personnel at this installation.'' Does this mean that there is a remaining deficit of 962 barracks spaces, or does it mean that there will be 962 people in barracks that do not meet standards?

    Answer. After completion of this project, there will be approximately 962 personnel in barracks that do not meet the 1+1 standard, if soldiers are assigned to barracks to achieve maximum utilization, i.e. one soldier per room.

    Question. What is the program for eliminating this deficit/deficiency?

    Answer. There is a barracks project at Fort Eustis programmed in fiscal year 2001 at $49,000,000, in the current Future Year Defense Program (FYDP). An additional project will be programmed after fiscal year 2003 to complete the barracks requirements at Fort Eustis.
 Page 448       PREV PAGE       TOP OF DOC    Segment 2 Of 3  

Washington-Fort Lewis: Central Vehicle Wash Facility ($4,650,000)

    Question. From block 9 on the form 1391, what is a bird bath?

    Answer. A bird bath is essentially a basin in which track vehicles can drive back and forth to loosen excess dirt attached to vehicles.

    Question. Describe the size and operations of the 6 bird baths, and how they will improve the efficiency of the existing wash facilities.

    Answer. The bird baths are roughly 12 meters by 42 meters. Track vehicles drive though bird baths to loosen excess dirt attached to vehicles; water cannons are utilized to loosen additional debris. Vehicles then enter the existing wash facility to complete the washing of vehicles. The designed system essentially provides the ''prewashing'' to knock off the big dirt then vehicles go thru the existing wash facility for clean up. Efficiencies are gained in the reduced time that vehicles will utilize the wash facilities and by eliminating the time that vehicles need to be rewashed due to vehicles not being cleaned properly.

Washington-Fort Lewis: Consolidated Fuel Facility ($3,950,000)

    Question. Why is this project requested in the Army's budget, rather than that of Defense Logistics Agency (DLA)?

    Answer. This project, which supports training, would not have ranked high enough by DLA to obtain near-term funding because DLA's priority is toward strategic deployment petroleum facilities. Nevertheless, it was a priority with the Army, so it was budgeted in the Army's military construction program.
 Page 449       PREV PAGE       TOP OF DOC    Segment 2 Of 3  

    Question. Isn't DLA the ''consolidated fuels manager'' for the Department of Defense?

    Answer. Yes. However, the Services are not prohibited from budgeting their own resources for critical petroleum facilities.

Washington-Fort Lewis: Close Combat Tactical Trainer Building ($7,600,000)

    Question. This is the only non-chemical demilitarization ''new mission'' project in the Army's military construction request. It will provide a $7,600,000 building to house equipment valued at $16,763,000 that will be acquired under the procurement account. Will this complete the entire program for construction and procurement under this new mission?

    Answer. Yes.

Washington-Fort Lewis: Tank Trail Erosion Mitigation—Yakima ($2,000,000)

    Question. This is the fourth of ten phases. Please provide for the record a table that will show the amounts appropriated to date for tank trail erosion mitigation, and the annual amounts programmed for the future, by fiscal year, including a brief description of the scope of work for each increment.

    Answer. Each increment is part of a total program to upgrade existing dirt roads to crushed rock, with improved drainage.

Table 42


 Page 450       PREV PAGE       TOP OF DOC    Segment 2 Of 3  

    Question. At the end of these ten phases, will permanent improvements have been achieved, or will this become a recurring annual requirement?

    Answer. At the end of the ten phases, permanent improvements will have been achieved. Therefore, no additional MCA funding is anticipated. However, a recurring OMA requirement will be needed to maintain the tank trails.

    Question. Could the Army National Guard (ARNG) perform this work, as part of a road-building exercise during annual training, rather than going abroad for such training?

    Answer. Yes, ARNG engineer units have the capability to perform this work provided that funds were available for the leasing of commercial equipment such as tracked excavators and rock crushers. As a matter of fact, Active duty engineer forces contributed about 25% of the work to complete the Phase 1 project. A contractor accomplished the remainder of the work including the supply of crushed rock. The installation decided to accomplish the subsequent phases totally by contract for a variety of reasons: (1) the Phase 1 experience showed that it was impractical to expect the work could be accomplished entirely by engineer units without some level of contract support, (2) the difficulty in scheduling engineer units to do the work and being assured that a higher priority mission would not impact their availability; and (3) the work must be accomplished during May, June, and July due to the weather conditions.

Belgium-Belgium Various: Child Development Center ($6,300,000)

 Page 451       PREV PAGE       TOP OF DOC    Segment 2 Of 3  
    Question. Will this project be located in Mons, Belgium?

    Answer. This project is in Mons, Belgium. The approved site is located in the ''hub'' of the community. It will be directly across the street from the DODDS school, in the housing area and in the same proximity as the existing child development center.

    Question. The existing facilities are at maximum capacity of 132 children and there is an excess demand waiting list of 109 children, for a total of 241 children. This project will provide a 198-child capacity. How was this facility sized?

    Answer. The facility was sized in accordance with the Army standard formula for determining child development center capacity. The 198 capacity child development center takes into account programmatic considerations such as shift workers. Additionally, the capabilities of the other Army Child Development Services delivery systems of Family Child Care and Supplemental program Services are considered in the sizing calculation.

    Question. Will it fully meet the requirement?

    Answer. Yes. The facilities usage plan provides the blueprint to fully meet the needs of the Mons community.

    Question. Will the facility be used exclusively by U.S. dependents?

    Answer. This facility will continue to serve primarily US members (98%) and meet their mission essential (full day) child care needs. Two percent of the population that use the child development center are the international military family members assigned to SHAPE. However, they primarily use the hourly care program.
 Page 452       PREV PAGE       TOP OF DOC    Segment 2 Of 3  

Germany-Schweinfurt: Whole Barracks Complex Renewal ($18,000,000)

    Question. Do residual value negotiations come down to the level of individual buildings—will the cost of this project be documented for any future residual value negotiations that may occur?

    Answer. Yes, residual value negotiations/settlements are made building by building. Yes, the costs of all projects are documented for future residual value negotiations.

    Question. According to page xl of the justification material, the Army plans to use the DOMFIRA account for real property maintenance on facilities in Europe, but not for new construction. Submit for the record a copy of the existing authorization for this account including the U.S. Code citation.

    Answer. P.L. 101–510, Sec 2921.(c) states: ''ESTABLISHMENT OF SPECIAL ACCOUNT.—(1) This is established on the books of the Treasury a special account to be known as the ''Department of Defense Overseas Military Facility Investment Recovery Account.'' Any amounts paid to the United States, pursuant to any treaty, status of forces agreement, or other international agreement to which the United States is a party, for the residual value of the real property or improvement to real property used by civilian or military personnel of the Department of Defense shall be deposited into such account. (2) Money deposited in the Department of Defense Overseas Military Facility Investment Recovery Account shall be available to the Secretary of Defense for payment, as provided in appropriation Acts, of costs incurred by the Department of Defense in connection with facility maintenance and repair and environmental restoration. . .''
 Page 453       PREV PAGE       TOP OF DOC    Segment 2 Of 3  

    Question. Wouldn't be possible to execute this Conn Barracks/Schweinfurt project under this DOMFIRA account, and to budget for the five barracks restoration projects shown on page xl within the RPMA/O&M account?

    Answer. There is no DOMFIRA currently available and we have no expectation for future cash settlements at this time.

    Question. If so, why did you decide one way instead for the other?

    Answer. For a barracks restoration to be eligible for DOMFIRA/OMA funding it must meet certain criteria. The new work element must be under $500,000 (or $1,000,000 when life/safety issues are involved). The Army has many barracks where the cost of the new work element exceeds the DOMFIRA/OMA limitation and their restorations must be funded using the MILCON appropriation.

Germany-Wuerzburg: Child Development Center ($4,250,000)

    Question. How long is the current waiting list for placement in the existing child development center?

    Answer. The present waiting list is approximately 65 to 70 children.

    Question. The existing facilities are at maximum capacity of 117 children. This project will provide a 145-child capacity. How was this facility sized?
 Page 454       PREV PAGE       TOP OF DOC    Segment 2 Of 3  

    Answer. the facility was sized in accordance with the Army standard formula for determining child development center capacity. The 145 capacity child development center takes into account programmatic considerations such as shift workers. Additionally, the capabilities of the other Army Child Development Services delivery systems of Family Child Care and Supplemental Program Services are considered in the sizing calculation.

    Question. Will it fully meet the requirement?

    Answer. The new 145 capacity child development center along with the other assets and delivery systems in the community will allow the child care requirements to be full met.

    Question. Will this facility be used exclusively by U.S. dependents?

    Answer. This child development center will be used exclusively by US family members.

Korea: Camp Humphreys Whole Barracks Complex Renewal ($8,500,000)

Korea-Camp Stanley Whole Barracks Complex Renewal ($5,800,000)

Korea-Camp Castle Whole Barracks Complex Renewal ($18,226,000)

 Page 455       PREV PAGE       TOP OF DOC    Segment 2 Of 3  
Korea: Camp Casey Whole Barracks Complex Renewal ($13,400,000)

    Question. The form 1390 states that the cost to remedy the defiencies in all existing permanent and semi-permanent facilities in Korea is $1,271,308,000. Will these four barracks projects provide a dollar-for-dollar reduction in this backlog?

    Answer. Yes, these projects will provide a dollar-for-dollar reduction in the backlog.

    Question. Provide for the record a map that will show the location of these four installations.

    Answer. A map showing the location of the four installations is provided below.
    "The Official Committee record contains additional material here."

    Question. Are each of these four projects in full agreement with the classified Theater Master Plan dated July 31, 1997?

    Answer. Yes, all projects are in agreement with the Theater Master Plan.

Kwajalein—Kwajalein Atoll: Power Plant, Roi Namur Island ($12,600,000)

    Question. Will this project include equipment provided from other appropriations—generators and so forth?
 Page 456       PREV PAGE       TOP OF DOC    Segment 2 Of 3  

    Answer. The generators are an integral part of this project and are included as a bid item in the construction contract. As such, funding for the generators is included in the total military construction cost estimate for the project. Approximately $1 million of Other Procurement, Army (OPA) funding will be provided for information/communication support.

    Question. The Ballistic Missile Defense Organization has a project in the budget request this year that includes physical security improvements. Is physical security a concern at the Roi Namur power plant site?

    Answer. No, physical security is not a concern at Roi Namur. The Ballistic Missile Defense Organization's requested security project is located at Fort Wingate, New Mexico.

    Question. How will the KREMs radars be powered while this two-year phased project is under construction?

    Answer. The existing power plant will continue to be used. It is anticipated that maintenance and repairs will extend current plant operations until the new power plant comes on line at which time the existing facilities will be demolished.

Worldwide Various-Minor Construction: Unspecified Minor Construction ($10,000,000)

    Question. Provide for the record a ten year history of amounts that have been requested and appropriated for unspecified minor construction.
 Page 457       PREV PAGE       TOP OF DOC    Segment 2 Of 3  

    Answer. The information follows:

Table 43



Worldwide Various—Planning and Design: Planning and Design ($41,819,000)

    Question. Will this funding level meet the known requirements for the fiscal year 1999 program, including the necessary work on projects programmed for fiscal years 2000 and 2001?

    Answer. Yes.

Worldwide Various—Host Nation Support: ($20,450,000)

    Question. Provide for the record a table that will show the expected distribution of this amount among the three efforts—criteria package preparation, design surveillance, and construction surveillance—and that will also show the expected distribution by country.

    Answer. A preliminary distribution of host nation support funds is as follows:

Table 44



Barracks

    Question. Provide for the record a chart that will show the Army's barracks construction program at the time the ''one plus one'' standard was approved, and the current program through completion, broken out by location in U.S., in Germany, and in Korea.
 Page 458       PREV PAGE       TOP OF DOC    Segment 2 Of 3  

    Answer. The Army's program began prior to the approval of the ''one plus one'' standard in fiscal year 1996 by the Office of the Secretary of Defense. The following chart lays out the construction portion of the Army's Whole Barracks Renewal Program.
    "The Official Committee record contains additional material here."

Family Housing Inventory

    Question. Provide for the record a chart that till show the average number of family housing units supported for fiscal year 1996, 1997, and 1998, and those expected to be supported in fiscal year 1999, broken out into government owned (U.S. and foreign), leased (U.S. and foreign), and privatized under Capital Venture Initiatives.

    Answer. The information on Average Inventory follows.

Table 45



Chemical Demilitarization

    Question. Submit for the record a chart which will show unobligated balances available, by fiscal year and by location, and the maximum amount of construction that could be put in place at these locations through the end of fiscal year 1999. We would appreciate any comments you may wish to add.

    Answer. The following is a chart showing unobligated balances of MILCON funds as of March 1998:

Table 46


 Page 459       PREV PAGE       TOP OF DOC    Segment 2 Of 3  

    The maximum amount of construction that could be put in place and cover long lead construction material, equipment, supplies, subcontracts and the Army Corps of Engineers contract oversight costs at these locations through the end of fiscal year 1999 is as follows:

Table 47


    Question. What benefit has been received from the unusual ''limited notice to proceed'' at Anniston? Why are you planning to use the same procedure at Pine Bluff?

    Answer. The Anniston systems contract employed a unique technique, a ''Limited Notice to Proceed'' (LNTP) provision. The LNTP allows for pre-construction activities (i.e., construction planning, subcontracting, document preparation and submittal of required plans) but prohibits any construction activities until the necessary environmental permits are received.

    The LNTP provision in the Anniston systems contract provided an opportunity for the on-site government team and the systems contractor (SC) to develop a working relationship and prepare to manage the contract activities. It also allowed the SC to fully participate in the environmental permitting process for the facility and develop a strong community relationship through well-planned outreach efforts before construction began. It also allowed the government sufficient time to review the SC submissions without impacting the contractor's schedule, especially during the period following contract award. The SC awarded subcontracts, set up administrative procedures and cost collection and accounting systems, and prepared contractual deliverables before the start of the construction phase of the project.

 Page 460       PREV PAGE       TOP OF DOC    Segment 2 Of 3  
    As numerous benefits were gained during the LNTP period under the Anniston contract and since award of the Pine Bluff SC is also expected before receipt of environmental documentation, the systems contract for the Pine Bluff facility will also contain a LNTP provision. (The systems contract for the Pine Bluff facility was awarded in July, 1997 with a LNTP provision. In November 1997, the General Accounting Office upheld the protest of an unsuccessful offeror; and a decision is expected in late-March 1998.) As was the case in Anniston, the Pine Bluff SC will participate fully in the environmental permitting and public outreach process through interaction with local officials and the public before actual construction begins. While construction efforts will be held in abeyance until receipt of necessary environmental documentation, which is expected in 4QFY98, the following activities are planned:

    The SC will provide bid bond documentation and insurance certificates, prepare contract deliverables and obtain government approval for planned construction work, select and award subcontracting efforts. Other deliverables include such items as the planned contract Work Breakdown Structure, Systems Safety Program Plan, Quality Assurance/Quality Control Program Plan, Accident Prevention Plan, Emergency Response Plan, and Facility Security Plan. The SC will submit subcontractor plans to support the above deliverables, obtain approval of lay down plans, and submit a construction Network Analysis System which must be approved before any progress payments can be made.

    Question. For the record, describe the ''P.L. 104–208 baseline restrictions'' and how they are being addressed?

    Answer. Public Law 104–208 prohibits the obligation of funds for the construction of baseline incineration facilities at the Pueblo, CO, and Blue Grass, KY, storage sites until 180 days after the Secretary of Defense submits a report on the effectiveness of each demonstrated alternative technology to incineration for destruction of assembled chemical munitions. Reports are scheduled to be provided to Congress in December 1998 and April 1999. Consistent with Public Law 104–208, the Program Manager for Assembled Chemical Weapons Assessment reports directly to the Under Secretary of Defense (Acquisition and Technology).
 Page 461       PREV PAGE       TOP OF DOC    Segment 2 Of 3  

    Environmental permitting activities and documentation for incineration-based disposal facilities at Blue Grass and Pueblo are continuing in the event no viable alternatives are identified. In this case, a decision by Congress to precede with the baseline incineration process is required not later than June 30, 1999, in order to meet Chemical Weapons Convention disposal deadline of April 29, 2007.

    Question: Please provide for the record a chart which will show the amounts (by location) and timelines for the entire Military Construction cost of the Chemical Stockpile Disposal Program.

    Answer. See attached chart.

Table 48



    Question. What is you assessment of the current safety status of the unitary chemical munitions stockpile?

    Answer. Experience and testing suggests that some stockpile deterioration has occurred over time, but that it is not dramatic. Munitions leakage is occurring, but with the possible exception of the GB-filled M55 rockets, the rate of leakage does not appear to be increasing at present. It is the Army's assessment that, in general, the stockpile is stable. The primary sources of risk remain external events, such as earthquake, lightning, tornado and airplane crash. Handling of the stockpile items to accomplish any reconfiguration poses a significant risk to the public, the workforce and the environment.

 Page 462       PREV PAGE       TOP OF DOC    Segment 2 Of 3  
    The Army is confident that there are adequate resources (processes, equipment, facilities, manpower) in place to ensure the safe and environmentally sound storage of the chemical stockpile through fiscal year 2007. We continue to analyze storage risks, leaking munitions, propellants and explosives used in chemical munitions, and the condition of the storage structures to further improve our chemical surety processes and enhance safe storage.

    Question. What is the timetable and what are the milestones for completion of the chemical demilitarization program?

    Answer. Following is the Chemical Stockpile Disposal Program schedule and major milestones.

Table 49



    Question. Provide for the record a chart showing, by location, the facilities and operational costs for the chemical demilitarization program.

    Answer. The requested information is shown in the table below. Facilities costs include the cost of designing, construction and equipping each of the chemical demilitarization and support facilities. Operational costs include all costs to test and operate each facility as well as training and all other support costs related to the operation of each facility. Site-specific chemical stockpile emergency preparedness (CSEP) costs are also included. Non site-specific programmatic costs are not included.

Table 50



    Question. What is the timetable for the funding of all construction phases, for the obligation of funds, and for the construction and operation of the chemical demilitarization facilities at locations for which military construction funds are requested—Pine Bluff, Arkansas; Newport, Indiana; Aberdeen, Maryland; and Umatilla, Oregon?
 Page 463       PREV PAGE       TOP OF DOC    Segment 2 Of 3  

    Answer. The construction projects for Pine Bluff, Newport, Aberdeen, and Umatilla are incrementally funded based on funding needed to support the construction effort at each site during that particular year and represents the maximum amount of construction that could be put in place based on the specific site award date. The following table provides the obligation plan for those funds through the end of construction.

Table 51



    The schedule for construction, systemization, and operational phases of the projects is provided below.

Table 52



    Question. What is the total design life of each of these demilitarization facilities?

    Answer. Plant design is not driven by ''life'' criteria but rather by environmental and safety requirements to achieve maximum protection as required by law. This equates to a design life of five years of continuous operation. With prudent maintenance and equipment replacement, the facility's life can be extended significantly. However, the cost of extending the life of a facility is dependent upon local ambient weather conditions; for example, the warm salt air environment at the Johnston Atoll Chemical Agent Disposal System greatly accelerates corrosion of exposed equipment and structures.

    Question. Are the sums requested equal to the amount of construction that can be put into place during fiscal year 1999?
 Page 464       PREV PAGE       TOP OF DOC    Segment 2 Of 3  

    Answer. Yes, the funds requested in fiscal year 1999 equal the amount of construction that can be placed during the twelve-month period.

    Question. What are the current annual costs of maintaining the stockpile?

    Answer. The current estimate for maintaining (storing) the chemical stockpile at all storage sites, including ancillary costs of treaty compliance and emergency preparedness is approximately $120 million.

    Question. What is the total ''sunk cost'' for the chemical demilitarization program through the end of fiscal year 1998?

    Answer. The Army projects that approximately $4.6 billion will be obligated for the Chemical Demilitarization Program by the end of fiscal year 1998. This figure includes $4.3 billion for the Chemical Stockpile Disposal Program and $0.3 billion for the Non-Stockpile Chemical Materiel Program.

    Question. No funds are requested for chemical demilitarization planning and design costs, which were funded in the past under the ''Defense-wide'' account. What is the current unobligated balance of previously appropriated funds, by fiscal year, and what is the plan for obligating these funds?

    Answer. The current (March 1998) unobligated balance of previously appropriated Military Construction Defense wide funds for chemical demilitarization planning and design costs include $5.9 million (fiscal year 1996), $4.12 million (fiscal year 1997), and $9.2 million (fiscal year 1998) for total of $19.22 million. The obligation plan for these funds is as follow:
 Page 465       PREV PAGE       TOP OF DOC    Segment 2 Of 3  

    $3.2 million will be obligated during the remainder of fiscal year 1998 to support the U.S. Army Corps of Engineers' (COE's) Huntsville Center in-house activities and to support additional design efforts for known changes for Anniston, Umatilla, and Pine Bluff.

    $7.0 million is planed to be obligated during fiscal year 1999 to support the COE's Huntsville Center in-house activities and to initiate design efforts to update the Pueblo and Blue Grass designs.

    $9.0 million will be obligated in fiscal year 2000 to support the COE's Huntsville Center in-house activities and to finalize design efforts for the Pueblo and Blue Grass designs.

Advance Appropriations

    Question. At this point in the record, please submit charts displaying the projects and the amounts for which advance appropriations are requested—tables 1 and 2, which appear in the justification material on pages xxviii and xxix.

    Answer. The tables are provided below.
    "The Official Committee record contains additional material here."

    Question. Is it correct that the only precedent for enacted advance appropriations in the Military Construction appropriations bill was for the construction of Fort Drum?
 Page 466       PREV PAGE       TOP OF DOC    Segment 2 Of 3  

    Answer. That is correct. The ''Military Construction Appropriations Act of 1987'' included advance appropriations for Fort Drum for fiscal years 1988 and 1989. This action was accompanied by full authorization in the ''National Defense Authorization Act for FY 87.''

    Question. Provide for the record a table displaying the advance appropriations that were provided for Fort Drum.

    Answer. Fort Drum advance appropriations included:

    FY 88, $221,000,000;

    FY 89, $214,000,000.

    Question. What benefits were realized from the Fort Drum advance appropriations, compared with the well-established method of full authorization and incremental annual appropriations?

    Answer. Advance appropriation allowed the pace of construction to continue without having to slow the contractor to keep pace with the funding stream. The momentum of construction was such that immediate use of the appropriation at the start of the fiscal year was required to make payment to the contractor. Under the established Congressional method of full authorization and incremental annual appropriations, contractor work would have been stopped or slowed if there had been a Continuing Resolution or delay in approving the fiscal year 1988 budget.
 Page 467       PREV PAGE       TOP OF DOC    Segment 2 Of 3  

    Question. Did the Fort Drum advance appropriations avoid change orders, overruns, claims, and so forth?

    Answer. No, advance appropriations did not avoid change orders, overruns, and claims.

    Question. What benefits are expected from the requested advance appropriations for fiscal years 2000, 2001, and 2002, and what is the basis for this expectation?

    Answer. Advance appropriations permit the obligation of funds at the start of each fiscal year and do not slow contractor execution. This becomes critical if budget approval is delayed or operates under a Continuing Resolution. If annual appropriations bills are enacted prior to the fiscal year, the benefits of advance appropriations are not expected to be greater than the method of full authorization and incremental annual appropriations. However, under OMB policy, the Army must request full project funding to build complete and usable facilities.

    Question. Would advance appropriations have any impact on the way the Army builds its POM of FYDP?

    Answer. Advance appropriations would allow the Army to develop a program that could include more projects than would be possible if we had to include a large appropriation request against a single project.

KOREAN CURRENCY FLUCTUATION
 Page 468       PREV PAGE       TOP OF DOC    Segment 2 Of 3  

    Question. Provide for the record a monthly listing of currency exchange rates between the won and the dollar for each of the past twelve months.

    Answer. The won to the dollar exchange rates for the past twelve months are as follows:

    1997:

MAR—882.62

APR—895.57

MAY—894.67

JUN—891.40

JUL—893.09

AUG—898.71

SEP—912.50

OCT—929.42

 Page 469       PREV PAGE       TOP OF DOC    Segment 2 Of 3  
NOV—1035.22

DEC—1494.04

    1998:

JAN—1707.30

FEB—1628.42

    Question. What impact has this had on the Combined Defense Improvement Program and on the Republic of Korea Funded Construction Program?

    Answer. For the Combined Defense Improvement Program (CDIP), the currency fluctuation does not impact since all CDIP construction contracts are awarded and administered by the Republic of Korea (ROK) government in their local currency, the Won. The ROK Funded Construction (ROKFC) program provides dollars to the U.S. Forces Korea for the design and construction of projects. The economic crisis in Korea has included hyperinflation in most construction materials and fuel related costs. Thus, for the ROK Funded Construction (ROKFC) program, the currency fluctuation savings gained from devaluation will be negated by increased inflation.

    Question. What impact has this had, in turn, on the requirement for Host Nation Support planning and design?

 Page 470       PREV PAGE       TOP OF DOC    Segment 2 Of 3  
    Answer. There is no impact on the requirement for Host Nation Support Planning and Design since it is expected that future Burdensharing levels for Host Nation funded construction will be no less than the levels we have received in the past.

PREPOSITIONING IN EUROPE

    Question. What is the current status of the Army's review of requirements for prepositioning in Europe?

    Answer. As part of the recurring Total Army Analysis (TTA) process, force structure and equipment needs, derived from all the unified Commander-in-Chief (CINC) requirements, are reviewed and validated. Army prepositioned equipment requirements, specific to Europe, will be included in the upcoming TAA–07 process. Additionally, U.S. European Command (EUCOM) is also reviewing their long-term prepositioned equipment needs to support NATO commitments and contingency operations. The results of this review, scheduled for release later this year, will be incorporated in future Army requirement analysis.

    Question. Is it possible that some Prepositioned Organization Materiel Configured in Unit Sets (POMCUS) sites will be closed or relocated?

    Answer. The Army does not have an approved plan to close or relocate any of the current prepositioned equipment sites in Europe. U.S. European Command (EUCOM) and Army Material Command (AMC) have conducted various studies to analyze operational efficiencies and cost effectiveness among the various support installations. The most recent review should be completed later this year. The recommendations of this analysis will be balanced against strategic requirements and NATO commitments. Any change in these installations will be thoroughly reviewed and coordinated with all organizations and countries involved.
 Page 471       PREV PAGE       TOP OF DOC    Segment 2 Of 3  

    Question. What is the current plan for further development at Camp Darby?

    Answer. NATO has approved the funding ($40 million) for a capability package known as Theater Reserve Unit/Army Readiness Package—South (TRU/ARPS) at Camp Darby, Livorno, Italy. Project includes construction of seven new controlled humidity warehouses (CHW); building a new maintenance facility; POL pump station; hazardous waste storage facility; battery activation facility; armament maintenance and storage facility; site communications; vehicle wash racks; vehicle grease racks; and upgrade of 10 existing warehouses to CHW. Project is in the 35% design phase which is planned to be complete 3rd Quarter of Fiscal Year (FY) 1998; the entire project was to be completed FY 2002.

INTERNATIONAL MILITARY HEADQUARTERS

    Question. Is the Army devoting any resources over the Future Years Defense Plan for planning and/or design efforts for NATO headquarters in Europe, or for the expansion of NATO headquarters in connection with NATO enlargement?

    Answer. The Army is not resourcing any planning and design efforts for NATO headquarters in Europe, or for the expansion of NATO headquarters in connection with NATO enlargement over the Future Year Defense Plan.

PRIVATIZATION OF UTILITY SYSTEMS

 Page 472       PREV PAGE       TOP OF DOC    Segment 2 Of 3  
    Question. Describe in some detail the Army's ongoing efforts to privatize utility systems.

    Answer. The Army goal, as revised in accordance with the recent Defense Reform Initiative, is to privatize 100% of the natural gas distribution, electric distribution, water, and wastewater systems by 1 January 2000, except those needed for unique security reasons or where privatization is uneconomical. Through Fiscal Year 1997, the Army had addressed 45 of its 265 systems either by (a) establishing that system was already privately owned or privatization was uneconomical or (b) completing conveyance of the system to a regional or municipal utility or qualified investor-owned company. Economic analyses leading to a privatization decision are under way or will be initiated in FY 98 for all remaining systems to assist in meeting the goal.

    Question. Are you seeking any legislative language to change existing authority?

    Answer. No, the Army is not seeking any legislative language to change existing authority.

    Question. Can you document any savings or avoided costs, in net present value or are costs merely being amortized through future rate payments?

    Answer. Capital investment costs for privatization of utility systems are being amortized through future rate payments.

FAMILY HOUSING—NEW CONSTRUCTION
 Page 473       PREV PAGE       TOP OF DOC    Segment 2 Of 3  

    Question. What meaning does the form 1391 have for new construction of family housing units, when the Army is vigorously pursuing whole-installation privatization—how can you assure that projects will be delivered at full scope and within the appropriate amounts?

    Answer. The forms 1391 were developed to correct existing conditions as stated on each form, with costs based on the Tri-Service Cost Model for each specific project site. During development of each such project, no consideration was given to the costs or other requirements associated with privatization of the whole installation. However, because of the potential to leverage these funds more than three-fold, if analysis indicates that privatization should be pursued, then these construction funds would be transferred to the Family Housing Improvement Fund for project scoring. If, on the other hand, analysis indicates that privatization is not feasible, then these construction projects will be promptly advertised and awarded, using standard turnkey procurement methods.

    Question. How are you planning to incorporate these projects, as well as other previously appropriated projects that have not been executed, into RFPs for whole-installation privatization?

    Answer. By leveraging the Military Construction funds we will be able to accomplish not only the original scope of work, but also revitalize all of the installation's existing inventory to meet contemporary standards.

    Question. Why are projects requested at locations that are early candidates for privatization—why not let developers take on the burden of demolition and replacement construction right away, as they will for the next fifty years?
 Page 474       PREV PAGE       TOP OF DOC    Segment 2 Of 3  

    Answer. If the analysis shows that privatization is feasible at a location that has a military construction project programmed, the funds will be leveraged to accomplish a Capital Venture Initiative project. However, if privatization does not prove to be the best approach, the original project can still be executed.

    Question. What is the basis for the cost estimate for each of these projects, military specifications under turnkey contracting, or some other method?

    Answer. The turnkey method of procurement will be used to award these projects. The cost estimate for each of these projects was developed using the Tri-Service Cost Model and site specific infrastructure costs estimates. This is the same method that has been used to develop family housing new construction projects in prior fiscal years.

    Question. How is it possible to do a cost estimate for new construction projects at locations that will be Capital Venture Initiative locations . . . how is it possible to say what new units will cost?

    Answer. Each project was developed to correct existing conditions as stated on each form, with costs based on the Tri-Service Cost Model for each specific project site. During development of each such project, no consideration was given to the costs or other requirements associated with privatization of the whole installation. This is the same procedure that has been used to estimate new construction projects for prior fiscal years. If analysis indicates that privatization should be pursued, then these construction funds would be transferred to the Family Housing Improvement Fund for project scoring. If, on the other hand, analysis indicates that privatization is not feasible, then these construction projects will be promptly advertised and awarded, using standard turnkey procurement methods.
 Page 475       PREV PAGE       TOP OF DOC    Segment 2 Of 3  

    Question. How will existing section 801 units be rolled into whole-installation privatization agreements?

    Answer. Section 801 housing is already privatized and will not be included in the Capital Venture Initiatives agreements.

Family Housing—Post Acquisition Construction

    Question. Provide for the record a ten-year history of amounts that have been requested and appropriated for post-acquisition construction of family housing.

    Answer.

Table 53



West Virginia—Camp Dawson (Kingwood): Readiness Center ($4,465,000)

    Question. Provide for the record a detailed description of the differences, if any, between this project and the project that was authorized and appropriated last year titled ''Armed Forces Reserve Center'' at a cost of $6,828,000?

    Answer. The presently planned project, an Armed Forces Reserve Center (AFRC), was authorized and appropriated in fiscal year 1998, at $6.828 million. An AFRC requires two or more reserve components to utilize the facility. In this instance the AFRC will be housing elements of the West Virginia Army National Guard and the U.S. Army Reserve. The $6.828 million represents 100% Federal funding of a larger facility.
 Page 476       PREV PAGE       TOP OF DOC    Segment 2 Of 3  

    Because the Army Reserve had reservations to this ARFC at the time the fiscal year 1999 President's budget was prepared, the Army National Guard proceeded ahead with a request for only a Readiness Center. This project planned at $4.465 million would have housed only West Virginia Army National Guard elements. The $4.465 million represents 75% Federal funding of a smaller facility. The Army Reserve has now agreed to share this facility and it will be an ARFC.

    Question. Is it correct that this project is not required if H.R. 2531 (the line item veto disapproval bill) becomes law?

    Answer. Yes. This project was included in the fiscal year 1999 budget because of the line item veto action. Now that Congress has acted to restore the line item veto projects, the funds for this project are no longer required in fiscal year 1999.

Utah—Salt Lake City: USAR Center/Organizational Maintenance Shop/Direct-Support-General Support/Equipment Concentration Site (13,200,000)

    Question. Is it correct that this project is not required if H.R. 2631 (the line item veto disapproval bill) becomes law?

    Answer. No. Assuming that the state of Utah provides only $500,000 as they have currently offered as assistance to the Army Reserve for this project, the Army Reserve needs $4,590,000 from the FY 99 appropriation for the Ft. Douglas project. That amount is a more accurate estimate of costs of the project, land acquisition, site preparation, relocation, interim lease, and environmental costs. Without an additional appropriation, all these costs will be taken from the $12,714,000 FY 98 project appropriation, resulting in a reduced project scope or reprogramming requirement.
 Page 477       PREV PAGE       TOP OF DOC    Segment 2 Of 3  

    Question. Provide for the record a detailed description of the differences, if any, between this project and the project that was authorized and appropriated last year titled ''Camp Williams: USARC/OMS'' at a cost of $12,714,000?

    Answer. The projects are the same. However, the $12,714,000 was based on the assumption that the state of Utah would provide land, site preparation, relocation costs, interim lease costs, and environmental cleanup costs of the land to be conveyed to the state of Utah. The State has offered only $500,000 to the Army Reserve to pay for those costs.

    Question. Bring us up to date with plans to vacate the current site, relocate to interim leased facilities, and construct permanent facilities at Fort Douglas, Camp Williams, or elsewhere.

    Answer. The Army Reserve will turn over the 10 acres on Ft. Douglas to the state of Utah no later than 30 September 1998. Environmental remediation of the 10-acre site is underway and should be completed this summer. The units and activities currently occupying the site will relocate to interim leased facilities no later than 30 September 1998. Suitable facilities have been located for leasing, and actions are in progress to secure the lease. Design has been initiated for the permanent facility construction, and the project is 10% designed. A reasonable contract award date is September or October 1998 with construction to begin soon thereafter.

    [CLERK'S NOTE.—End of questions for the record submitted by Chairman Packard:]
 Page 478       PREV PAGE       TOP OF DOC    Segment 2 Of 3  

    [CLERK'S NOTE.—Subseqent to final Congressional action overriding line-item veto of fiscal year 1998 projects, the Army National Guard and the Army Reserve revised their Future Years Defense Program (FYDP). The following information supercedes that which appears in Part 1 of this year's hearings volume, at pages 552, 553, and 606 through 609.]
    "The Official Committee record contains additional material here."

    [CLERK'S NOTE.—Question for the record submitted by Mr. Hobson.]

ACTIVE AND RESERVE COMPONENTS FUNDING

    Question. My feeling is that the army, historically, has not done as good a job with its Guard and Reserve units as some other Services. I think that's a real problem in readiness. When the Army has to go, the Army Guard and Reserve have to go as well. However, they get a lot of criticism because they're not ready. The Army has to look at the changes in the world today and, like it or not, treat the Guard and Reserve as a true part of the total force. In the future, there is going to be more, not less, dependence on the Guard and Reserve. I think the Army is a little slow in recognizing that. What in the budget would make me think otherwise?

    Answer. The involvement of the Reserve Components in determining the allocation of installations resources as part of the Total Army Force is a success story. We believe the concept of parity is fully embodied in the development process of the Army's installations programs. Each component is represented on each of the main decision making bodies involved in the planning and budget process. Nevertheless, the Army has only been able to fund its highest priority programs—statutory requirements (such as environmental and Chem Demil); Barracks; and Strategic Mobility. Beyond this, all components are funded equally, for example, in revitalization military construction we have funded the Active at 13%, the Guard at 12%, and the Reserve at 38% of requirements. Included in the budget request is $24.6 million for Guard readiness centers (armories), training sites, and ranges. The budget request also includes $44.5 million for Army Reserve centers and maintenance facilities.
 Page 479       PREV PAGE       TOP OF DOC    Segment 2 Of 3  

    [CLERK'S NOTE.—End of questions for the record submitted by Mr. Hobson.]
      

Tuesday, March 3, 1998.

DEPARTMENT OF THE NAVY

WITNESS

HON. ROBERT B. PIRIE, JR., ASSISTANT SECRETARY OF THE NAVY, INSTALLATIONS AND THE ENVIRONMENT

Statement of the Chairman

    Mr. PACKARD. Ladies and gentlemen, we want to welcome some of our young people from Spotsylvania, I believe it is, down south a few miles in Virginia. This is a hearing with the Navy and the Marine Corps on military construction issues. It is a very significant part of our committee responsibilities to service the needs of the Navy and the Marine Corps.

    With us this afternoon is Secretary Robert Pirie, a man of great experience. If any of you have had the time, particularly on the committee, to read his resume, you will find that he has had significant experience, having spent a career in the Navy, and then having been involved in many, many other remarkable assignments which I will not deliberate on, Mr. Secretary. But we are delighted to have you here.
 Page 480       PREV PAGE       TOP OF DOC    Segment 2 Of 3  

    We are very much looking forward to your testimony. I have read it through, and it is very thorough and very complete and very well done and well organized. As you might suspect, I am concerned and disappointed, of course, in the lower budget levels that we are seeing from all the Departments in the Services, the Navy and the Marine Corps not excepted.

    We have made a strong effort on this subcommittee to increase our effort to catch up on quality of life facilities for our men and women in the services—housing and child care centers and hospitals and dental centers, and just the entire facility issue that our men and women live with.

    And it is not just a matter of convenience for our service men and women, but it is a retention issue. It even reaches a point of readiness in many instances. If we do not have adequate facilities, adequate runways, adequate barracks, adequate activities and facilities for our men and women to be not only happy but successful in the services, then that affects readiness in many instances.

    As we decrease the budget and have decreased the budget over the last few years, it is very possible we may be reaching the point where we might be affecting readiness, not just the quality of life issues for the service men and women.

    By comparison, the budget request last year included 11 barracks projects and we enacted 16 barracks projects. This year we have significantly less being requested. Only one for sailors overseas, four for the Marine Corps, and five barracks for the Navy, compared to 16 last year that we appropriated for.
 Page 481       PREV PAGE       TOP OF DOC    Segment 2 Of 3  

    I am aware also that your budget was prepared and submitted previously to any override efforts in terms of the 38 projects, but even with that calculation we sense that it is still a very significant reduction in this year's budget.

    Yours is not as bad as some of the other departments, frankly. But it is still a significant reduction from last year's appropriated levels, and that we hope you will address as part of your testimony.

    It is a pleasure to have you in this subcommittee hearing—our next hearing will be tomorrow at 9:30 in the morning. We would like to start if we can on time. That will be with the Air Force and the Air National Guard and the Air Force Reserve. It should be an interesting hearing, and we hope the members of the subcommittee will be here.

    With that, Mr. Pirie, I am going to conclude my comments and ask if Mr. Hefner would like to make any, and then we would like to hear from you.

    Mr. HEFNER. Thank you, Mr. Chairman. We are happy to see you before the committee this morning. I will not make a statement. I will just echo what the chairman has said, and let you know that the commitment that this committee has had over the years for quality of life and benefiting our men and women in the services. We have fought this budget every year and we complain about not having a sufficient budget.

    We are probably one of the only budgets in this House that has even remained stagnant or had cuts in real dollars over the past years. So we are going to do whatever we can to ensure we use our scarce dollars wisely. I know you are going to do the same thing, because we both have a commitment to our men and women in the service. We certainly want to maintain our retention edge and we just welcome you to the committee today.
 Page 482       PREV PAGE       TOP OF DOC    Segment 2 Of 3  

    Thank you, Mr. Chairman.

    Mr. PACKARD. Thank you, Mr. Hefner.

    Again, I think you can assume that we have read your testimony, and it is well done. We would hope that you would not just read it to us, but summarize it as you feel you need to, and then we will spend as much time as we can on questions and answers.

    Mr. PIRIE. Thank you. If it is all right with you I would just like to underscore three things in the testimony.

    Mr. PACKARD. Your entire testimony will be entered into the record.

STATEMENT OF HON. ROBERT B. PIRIE, JR.

    Mr. PIRIE. All right. First, Mr. Chairman, our requests for military construction, for family housing and bachelor housing are generally lower this year than they were last year, even without considering the projects added by the Congress in last year's budget.

    I want to emphasize that this reflects the administration's priorities in dealing with strong demands in other program areas. Reflecting on my four years in this job, I recall that in 1994 there was great concern about readiness, in 1995 about quality of life, in 1996 and 1997 about modernization and increasing the size of the modernization accounts and beefing up the investment.
 Page 483       PREV PAGE       TOP OF DOC    Segment 2 Of 3  

    We have yet in my experience to have a year of concern for real property. I am not personally happy about this, but it is not the first time in my experience that the military construction and real property maintenance accounts have been used to pay other bills.

    In any case, our request is a straight forward expression of the priorities, and we recognize that there is little flexibility in the balanced budget amendment for funds to be added, and we are prepared to live with what we have asked for.

    Another area I am not personally pleased about is that of family housing and public and private ventures. Our progress in that area has been far less than I had hoped. There are some fairly good reasons for this, but not excuses.

    The source of funding that we have for these public and private ventures for family housing is MILCON projects. As I mentioned I think last year, this is an awkward device. The amount appropriated may bear little relation to the finally negotiated public private venture.

    The fact that there is a project available for obligation and the start of construction may tempt claimants to press on with the MILCON and not wait for a public/private venture to be developed and negotiated. And there is, in fact, not much incentive for the claimants to go to public/private ventures.

    The savings that accrue are centrally managed, and they are invisible to those in the field. And many of our people in the field have yet to be convinced that public/private ventures are the best way to provide housing for our people.
 Page 484       PREV PAGE       TOP OF DOC    Segment 2 Of 3  

    Finally, the many stages of approvals for PPVs tend to dampen enthusiasm for projects like that. I am very concerned and I have put the issue at the top of my priority list, and I expect to make substantial progress this year.

    Finally, and also very high on my priority list is the matter of seeing to completion the BRAC actions of earlier years. We are nearly finished with the MILCON projects required to support BRAC relocations. We are beginning to convey substantial amounts of property to the local redevelopment authorities for community reuse.

    In the coming year, we expect to complete the decision process for 19 bases, clearing the way for conveyance of the property in such places as Naval Station Treasure Island, NAS Alameda, and Naval Shipyard Long Beach.

    Each conveyance has its associated issues. But we and the affected communities are learning how to deal with them somewhat better, and are helping the communities and their local redevelopment authorities to become better informed about the required processes and the constraints under which we must operate.

    I am optimistic that the bulk of the pending disposals will be in place by the end of next year. Thus, our backlog of disposals from previous rounds of BRAC will be cleaned up well before we could have any other disposal pending from future rounds of BRAC.

    Mr. Chairman, with that we would be very glad to hear your questions.

 Page 485       PREV PAGE       TOP OF DOC    Segment 2 Of 3  
    [The information follows:]

PREPARED STATEMENT OF THE HONORABLE ROBERT B. PIRIE, Jr.
    "The Official Committee record contains additional material here."

    Mr. PACKARD. Thank you very much, Mr. Secretary. We did not intend for you to be more brief than you needed to be. I hope we did not crimp your style on that.

    I am going to ask if Mr. Hefner would like to lead us off in the questions. I certainly have many, but I will withhold for the other members.

    Mr. HEFNER. Well, I think I will follow suit.

DEMOLITION OF FACILITIES

    In your testimony you have centralized demolition—increased efforts to get rid of the old unneeded facilities. How much of a backlog in demolition do you have in the Navy?

    Mr. PIRIE. I am not sure what it is in dollar terms. Maybe Admiral Nash can help me with that.

    Admiral NASH. I do not think we have quantified that, sir, but we found that there is payback when the demolition is around three to four years. That is a real quick payback. And we are developing those lists now of what ought to be demolished.
 Page 486       PREV PAGE       TOP OF DOC    Segment 2 Of 3  

    Mr. PIRIE. In fact I think we could commit $25 million immediately to demolition work, and probably not run out of projects.

    Mr. HEFNER. I have some other questions, but I think I will wait until later, since there are quite a few members here that may have some questions. I'll reserve the balance of my time.

    Mr. PACKARD. Thank you, Mr. Hefner. Mr. Porter?

GREAT LAKES NAVAL TRAINING FACILITY

    Mr. PORTER. The junior members appreciate the Chairman and Ranking Member allowing us to ask our questions.

    I am going to be very provincial, Secretary Pirie, and talk about the Great Lakes Naval Training Facility. We understand you have $5.75 million for an applied instruction building modification, $7.4 million for gas turbine training facility, and $7.1 million for a hospital and school addition at Great Lakes.

    What other planned or projected construction is on the drawing board for Great Lakes in the future?

    Mr. PIRIE. I will have to supply you a complete list. The one project that I do know about is a small arms range which has been moved up into fiscal year 00 I believe. Does anybody have the Great Lakes list?
 Page 487       PREV PAGE       TOP OF DOC    Segment 2 Of 3  

    Admiral NASH. Yes, sir, we do. Or do you want us to supply it for the record?

    Mr. PIRIE. What do we have? For 1999, the applied instruction building mods and the gas turbine training facility. We have three BEQs for 2001. The small arms range has been moved up into fiscal year 2000. In 2002 we have drill hall replacement, a recruiting support center extension, and the replacement of the recruiting training center drill hall. And in 2003 a pass security office, an all weather track, and upgrade of the air conditioning systems there. And we will supply this list for the record.

    [The information follows:]

    The Navy has $113 million of projects programmed for Great Lakes in the remaining years of the current FYDP:

Table 54


    Mr. PORTER. Thank you. I am obviously very pleased to see that the Navy is continuing its commitment there at Great Lakes. And thank you for providing that information for me.

    Mr. PIRIE. It is our only troop training facility, as you know, and it is extremely important to us.

    Mr. PORTER. Thank you, Mr. Chairman.

 Page 488       PREV PAGE       TOP OF DOC    Segment 2 Of 3  
    Mr. PACKARD. Thank you. Mr. Olver?

    Mr. OLVER. Thank you, Mr. Chairman.

    Good to see you again, Secretary Pirie. I also will be somewhat parochial at this particular point. Last year in the MILCON budget there was an amount of a little over $4 million that was appropriated to renovate a building at the Westover Air Reserve Base in Chicopee, Massachusetts to accommodate a Marine Corps Air Reserve center that had been part of a BRAC closure—the BRAC closure being South Weymouth.

    And this funding project, as I indicated, was $4 million. The Marine Reserve, Air Reserve unit had been displaced from South Weymouth.

    Now, it is a modernization renovation of an existing facility, which had been an Air Force building, and that would provide the long term training facility for the Marine Aviation units that were transferred.

    Of course, the Marine unit is being provided with alternative space on the Westover Air Reserve Base in the meantime.

    Do you know what it is that is happening with that? Does somebody here know what it is that is happening with that?

    Mr. PIRIE. If it is the one I am thinking about, we got somewhat hung up with a discussion with the Air Force about the condition of the property when they turned it over to us. But I think that has been resolved.
 Page 489       PREV PAGE       TOP OF DOC    Segment 2 Of 3  

    General HAYES. Yes, sir. There is an environmental remediation issue with the Air Force. The dialogue is ongoing. I would have to supply the details for the record, sir.

    Mr. OLVER. Thank you.

    [The information follows:]

    What are the details of the environmental remediation issue with the Air Force on FY 1998 Military Construction, Naval Reserve project P–476 at Westover ARB?

    As reflected in the DD1391 for project P–476, the Air Force, during negotiations regarding renovation of building P1900 at Westover, accepted the fiscal responsibility for all environmental remediation and demolition connected with this project.

    The project cost developed by the Department of the Navy was predicated upon this understanding.

    The Air Force acknowledges this commitment and is now working to provide FY–98 USAF O&M to perform asbestos abatement, lead paint removal and associated demolition. Once this preliminary work is completed, we will execute the FY–98 project.

    General HAYES. The dialogue is engaged. It probably was not engaged as sufficiently as it should have been. As recently as this morning a gentleman appearing at another committee, an Air Force general gave me his card and said we want to step up to this responsibility. So we will pursue that.
 Page 490       PREV PAGE       TOP OF DOC    Segment 2 Of 3  

    Mr. OLVER. The Air Force?

    General HAYES. The Air Force?

    Mr. OLVER. The counterpart of Secretary——

    General HAYES. My counterpart. An Air Force Reserve general testified on his construction, and told me this morning he was well aware of it, and they were going to step up to it, and we are going to pursue that.

    Mr. OLVER. So you think I need not pursue that, that it is being pursued already? Are we any where close to a solution? I am not sure where the misunderstanding occurs.

    General HAYES. I cannot give you the details of the misunderstanding. I will have to provide that for the record, but we will make sure we pursue it.

    Mr. PACKARD. General, would you give us your name for the record?

    General HAYES. I am Brigadier General Mike Hayes, and I am facilities director of the Marine Corps.

    Mr. PACKARD. Thank you.
 Page 491       PREV PAGE       TOP OF DOC    Segment 2 Of 3  

    Mr. OLVER. I will be seeking a direct answer from you, and if I can figure out who the direct counterpart is at the Air Force, to see if we cannot get this sorted out.

    Mr. PIRIE. I would like to be optimistic and assure you that we can resolve this in the wink of an eye, sir. But I have unfortunately been exposed to too many such cases in the past. But we will work hard on this. We will try to come to a conclusion.

    Mr. OLVER. Okay. Thank you.

    Mr. PACKARD. Mr. Hoyer.

    Mr. HOYER. Mr. Chairman, thank you very much, and I appreciate again your courtesy for allowing me to participate in this hearing.

BASE REALIGNMENT AND CLOSURE

    Mr. Secretary, thank you very much for being here. There was much discussion in your testimony about the BRAC process. We have all been through the BRAC process. Caused a lot of disruption, but on the other hand the reports seem uniform that we still have a lot of excess infrastructure.

    My question to you is on page two of your testimony you refer to the QDR, that there was enough excess capacity in DOD infrastructure to warrant two more rounds of closures and realignments similar in size to BRAC rounds in 1993 and 1995.
 Page 492       PREV PAGE       TOP OF DOC    Segment 2 Of 3  

    Let me ask you from the perspective of the United States Navy, the implication here is there is a lot of excess infrastructure. While I may believe that is true across the DOD spectrum, can you give me your analysis for the committee as to what position the Navy might be in as it relates to excess infrastructure?

    Mr. PIRIE. We are, as a matter of fact, even as we speak, supplying information to the Office of the Secretary of Defense. We are going to compile a comprehensive report, and I hope to have that over here by the first of April.

    And I do not really want to leapfrog their conclusions. As you know, in BRAC 1995, some of the recommendations that the Department of Defense made with respect to Navy property to the Commission were not accepted by the Commission.

    Unless the calculations have been changed, which I doubt, I think we would consider there to be excess in those areas, although since we do not have presently a BRAC authority, we have not assembled anybody to examine the old methodology to see if it is warranted.

    We have not had any data calls for certified data or anything like that. So we do not have much to go on.

    Mr. HOYER. Mr. Secretary, let me be a little more specific. My belief is that the Navy contributes a lot at the office.

    Mr. PIRIE. Yes, sir.
 Page 493       PREV PAGE       TOP OF DOC    Segment 2 Of 3  

    Mr. HOYER. To some degree beyond what others saw fit to do for whatever reasons. In that context do you have, without going before the report is issued—I understand that is a difficult position—to the extent you can, what percent, if you can put a percentage on it, of excess capacity and infrastructure that we have in the Navy do you think we have already moved on?

    Mr. PIRIE. Well, as you know, in BRAC 1993 and BRAC 1995, we closed a very substantial number of installations, and did things like all of Alameda and Oakland, all of Charleston. And that very substantially reduced our excess capacity.

    We come down in the bases, something like 14 percent overall across the whole Department of Defense. I would say in the Navy we have probably come down closer to 25 percent, in excess capacity as a whole. So that at the end of the day in BRAC 1995, we felt that, for example, in Naval stations and cruiser equivalent pier space and so forth, we had gotten all the excess capacity there was anywhere to get.

    Mr. HOYER. Thank you. Mr. Secretary, you made a comment that concerns me—two comments. One, it is easy to defer maintenance, because we kid ourselves that it will not matter, and we could get it next year. Of course, next year never comes, because you have the same kind of fiscal crisis.

    At what risk of compounding our costs are we? In other words, the longer you defer, the greater the cost. Where are we in that process? The Chairman and Ranking Member have been just champions on this issue. It is tough, and of course the Chairman has a tremendous problem in terms of the resources available to him.
 Page 494       PREV PAGE       TOP OF DOC    Segment 2 Of 3  

    But I know that he is very concerned about this maintenance issue. Can you tell me at what risk we are at this point?

    Mr. PIRIE. We have piers and runways that are not in good shape and need to be fixed. We have buildings, hangars, for example, that leak, and so forth. We cannot afford to fix all of these in one year.

    We are hoping that our funding line will increase in future years in the five year Defense program, and currently it is planned that way. So if Lucy doesn't jerk the football away from Charlie Brown in future years, we ought to have a little more money to deal with those issues.

    I do not think that maintenance of real property situation at the moment is a crisis or anything like that. But I, like you, would not like to see the real property maintenance backlog grow much more.

RETENTION

    Mr. HOYER. Last question, and thank you, Mr. Chairman, for your courtesy. Last question, Mr. Secretary. This committee has been, the Chairman and the Ranking Member have both been, again, champions of the quality of life.

    Number one, we want to do that for our people. But, number two, as a very practical business judgment, we need to retain people that we spent a lot of money training. To what degree is our quality of life budget risking losing our ability to retain people we have trained and who are doing a good job?
 Page 495       PREV PAGE       TOP OF DOC    Segment 2 Of 3  

    Mr. PIRIE. The risk to retention is across the whole spectrum of things, which include tempo of operations, and military pay, and quality of life and so forth. I do not think that we are contributing to that risk unduly, in the family housing and bachelor housing area.

    But I feel constrained to say that when I was assistant secretary of Defense in the Carter administration, we had enormously severe problems in retention. And at that time I was convinced that although we had a military pay gap of fairly significant proportions, and we had pretty severe tempo of operations, that a major constituent of the problem was that our people had decided they were working for a second class organization, that did not have good facilities, good buildings, good housing, could not supply spare parts to people who needed to fix their equipment and things of that nature.

    I would hate to see us get to that point ever again.

    Mr. HOYER. Thank you, Mr. Chairman. Thank you, Mr. Secretary.

    Mr. PACKARD. Let me return to the first point that Mr. Hoyer brought up, and see if we can kind of wrap that section up. I have a few questions on BRAC, and if anyone wishes, I would be happy to yield if we can just kind of wrap up the BRAC portion of this question and answer session.

    You have been very comprehensive in your written statement on the progress we are making on BRAC, and when we expect to be through with the first four BRAC activities, and when and how we are moving into a net savings, and so forth, and I appreciate the comprehensiveness of your statement on that.
 Page 496       PREV PAGE       TOP OF DOC    Segment 2 Of 3  

    Do we have in place at the present time an evaluation process to determine if in fact we are doing the right thing in closing the bases? Obviously we are. But do we have a process where we can evaluate cost, savings, the transfer of missions to other areas in the services, the conveyance of properties and all of this?

    Each of these processes are fraught with a lot of problems. I see some of them, even out in my part of the country, where it is not a simple process of closing a base, conveying property to the successor, be in another Federal agency or a State or local agency or whether it be private sector.

    Do we have an evaluation process as we are completing the transfer of these bases to the private sector or to other places?

    Mr. PIRIE. If I understand the question properly, what you are asking about, is there a fundamental requirements statement that lays out all the real property that we require to support the operating forces?

    Mr. PACKARD. That may be, but I am more interested, frankly, Mr. Secretary, in as we wrap up a base closing, or even a realignment, do we just walk away from it once that is done, and forget it? Or do we have a process where we sit down or have a team that sits down and evaluates how that went, what went wrong, what could have been done better?

    Is it working? Is it all coming together? And where there are problems—and there are—are they being worked out or resolved or discussed to avoid them in other BRAC processes? You are talking about two more BRACs in the future, 2001 and 2003, if that comes about, which I think most of us feel it won't—at least not on that schedule.
 Page 497       PREV PAGE       TOP OF DOC    Segment 2 Of 3  

    But if it does, we ought to learn from what we have gone through. Do we have a process to evaluate that?

    Mr. PIRIE. We do. The Office of the Secretary of Defense has as committee that calls together all of the military departments and they lay problems and issues on the table that we are confronting. And we learn from each other's experiences and so forth.

    In my office we have been doing a great deal of learning in trying to bring along the folks in the field who have to implement these things. The latest information about what we found that works and what does not work.

    Mr. PACKARD. Are we transferring that to the existing closing processes that we might profit from it?

    Mr. PIRIE. Oh, yes. Yes, sir.

    Mr. PACKARD. I would hope that we would not retrace the same problems. I know there are guidelines that are in law as to the process, but the fact is where we have problems, at El Toro, for example, or any other base closing, that we ought to learn from that so we do not repeat the problems.

    Mr. PIRIE. One of the things that we have learned is that it is very important for us to start the discussions with the local redevelopment authority very early and to help them see the nature of the process that we have to go through.
 Page 498       PREV PAGE       TOP OF DOC    Segment 2 Of 3  

    Certainly the most frustrating thing for all of the communities and local redevelopment authorities is getting through the environmental impact statement business and so forth. And some mayors have expressed themselves as very impatient with that stuff.

    Unfortunately it is hard, sometimes, to tell whether they are impatient directly with us, or they are impatient with the operation of the laws. We are getting a lot smarter about how to do that, but it is a two way street. The local redevelopment authorities have to recognize that constraints about the clean up process, things that we must do and things that we are not allowed to do. And it is a matter of working together.

    In general it is improving.

    Mr. PACKARD. Is it? Thank you for that. In the conveyance of property as a result of BRAC, do you consider that DOD drives a hard bargain? Do they handle it like a business? Are they concessionary in terms of other government agencies that want to reuse the property? What is your primary focus in the conveyance of property?

    Mr. PIRIE. Well, there is a hierarchy of needs.

    Mr. PACKARD. I know there is.

    Mr. PIRIE. And other DOD entities have first priority, other government agencies priority after that. Then comes the homeless, and so on. In general, under the modes of conveyance that we have, we are required to obtain fair market value for the property.
 Page 499       PREV PAGE       TOP OF DOC    Segment 2 Of 3  

    In the case of economic development conveyances, we can soften the fair market value requirement if there is a public purpose to be served, particularly job creation in conveying the property under liberal terms and so forth.

    Mr. PACKARD. Most agencies, including the private sector, kind of think that they ought to get the property free. There is no way that we are going to reach a point where we see a savings if that is the general attitude.

    Mr. PIRIE. The fact that communities expect to get the property free or for a dollar bedevils us and makes it very hard for us to get even a reasonable or substantially discounted fair market value.

    So that impression certainly makes it difficult for us.

    Mr. PACKARD. What can you do? What should be done to dispel that general attitude?

    Mr. PIRIE. We have to start educating them, as I say, early in the process. And I think if members can tell their constituents that we simply cannot give this property away, that would be helpful, too.

    Mr. PACKARD. The last question on that section, the BRAC section, that I have is in the environmental clean up area. Most bases that are being closed or realigned that do not have air facilities on it, the clean up may not be quite as difficult.
 Page 500       PREV PAGE       TOP OF DOC    Segment 2 Of 3  

    But virtually any base that has had aviation activities on it, there are some major clean up requirements. Those generally are not inexpensive.

    Have you calculated, or do you have a feel for how large this problem is for the Navy and the Marine Corps and are we going to be doing the clean up ourselves, or are we going to pass that on to the new property owner or what?

    Mr. PIRIE. Well, we are required to clean up the environment.

    Mr. PACKARD. Before conveyance?

    Mr. PIRIE. Yes. And in some cases we don't have to clean it up before conveyance, but we do have to have a remedy in place that will ultimately result in it being cleaned up. Or some arrangement where for value received somebody else cleans it up, and we discount the value of the property, or something of that kind.

    We can come to an arrangement like that. But the responsibility is ultimately ours. We can never step away from that.

    We figure that for—I think it is in my statement somewhere—we figure that for clean up of the remaining BRAC actions we need something on the order of $3 billion or so. $2.1 billion I am much more reliably informed.

    And that is a big amount of money, but is not an undoable amount.
 Page 501       PREV PAGE       TOP OF DOC    Segment 2 Of 3  

    Mr. PACKARD. Are you able to avoid litigation in the process of clean up?

    Mr. PIRIE. Generally.

    Mr. PACKARD. That is one of the major problems of Superfund.

    Mr. PIRIE. Yes. Well, when you get to litigation, of course, everything stops until you can settle that, and the clock keeps ticking, and it costs more money. Generally we have been reasonably successful.

    Mr. PACKARD. Thank you. Mr. Cramer?

    Mr. CRAMER. Thank you, Mr. Chairman.

PUBLIC/PRIVATE VENTURES

    I apologize for having to come in late. I may end up being redundant with some issues that I would like to take this opportunity to bring up. But I was reading through your public/private ventures there, and your statement that you are pleased with the completion of the first two PPV efforts, but you must admit that subsequent progress has been less than you contemplated.

    Would you tell me a little bit more about that, and why that progress has not been as you thought it would be, if that is a fair way to put it?
 Page 502       PREV PAGE       TOP OF DOC    Segment 2 Of 3  

    Mr. PIRIE. Yes, sir.

    Mr. CRAMER. Maybe what I am looking for is what have you learned from this process, and how will you use that?

    Mr. PIRIE. In some ways the broad variety of authorities that we have to create public/private ventures, the fact that we can contribute land and Federal property, the fact that we can contribute to a public/private venture a guarantee of occupancy, the fact that there are a variety of kinds of things that can be done with a developer to create a public/private venture—this richness of availability of different means has kind of made it more complicated than it should have been.

    It is like a kid in a candy store. We have not been able to decide which methods to use. And we have not established any real good models. The two models that we have, the South Texas model, and the Washington State model are okay as far as they go, but I do not think that is going to be the way we do it everywhere.

    In fact, every place is really substantially different.

    Mr. PACKARD. Would the gentleman yield for a moment? Under the two that you are talking about, you were working with different authorities. And today we have different authorities than that. How do you relate your experience there to what your experience appears to be now?

 Page 503       PREV PAGE       TOP OF DOC    Segment 2 Of 3  
    Mr. PIRIE. The initial set of authorities came with some money, specifically directed toward that, and a fairly constrained mode of getting into this venture. So we essentially followed the cookbook and developed these projects. It was not easy. It took a couple of years just to go from concept to a request for proposals.

    But that was relatively simple. We are talking about far more complicated deals now, and a general reluctance everywhere to make a 20 to 50 year mistake. We really want to work these things out and get them right.

    But they are beginning to come into shape. With the Marine Corps we have a public/private venture in Albany, Georgia that has just gotten through all the wickets, which involves contribution of property, both land and houses, that are not on the Marine Corps base, as our stake in a venture that would have the developer build houses on the base for occupancy by Marine Corps personnel. He would build them and maintain them for some period of time.

    Mr. CRAMER. Are you discouraged? Are you about to decide that it is more trouble than it is worth? Or is it just so unique to every situation that it is too time consuming to come up with an application that——

    Mr. PIRIE. We are not discouraged. I think ultimately this will work. I think ultimately it is a good deal for the taxpayer and for the sailors and marines. But it has been an adventure getting everybody on board, figuring out what we want to do, and also bringing the community of developers into the picture.

    Mr. CRAMER. Thank you.
 Page 504       PREV PAGE       TOP OF DOC    Segment 2 Of 3  

    That is all I have, Mr. Chairman.

    Mr. PACKARD. Thank you.

    Mr. Hefner?

BASE REALIGNMENT AND CLOSURE-ENVIRONMENTAL CLEANUP

    Mr. HEFNER. If I could just follow up. If I understand it right, if we decide to close a base, God forbid, say Fort Bragg——

    Mr. PACKARD. That's a terrible idea. [Laughter.]

    Mr. HEFNER. Okay, Fort Meade or wherever.

    Mr. HOYER. What was that? [Laughter.]

    Mr. HEFNER. That's another bad example, isn't it? Let's start again.

    It seems to me that transfering a facility is a bit cumbersome. When talking about transfers I know that we tried to transfer the Presidio to the Interior Department we had some problems.

 Page 505       PREV PAGE       TOP OF DOC    Segment 2 Of 3  
    The Interior Department says they would love to have it, but we do not have the money to clean it up. So you clean it up, and then you can give it to us. And then of course the private sector, says, hey, if you are going to clean it up, give us the chance to bid on it or get into the process.

    It seems we need to work out a situation where it would be good practical business to transfer a facility from, say, the military to the Interior Department or whoever. Then let them utilize their appropriations or have a special appropriation for clean up in their agency. For example say we are closing it down and we are going to transfer it to the Interior Department, and out of their pot they clean it up.

    And they have to meet the same guidelines. Does that make any sense?

    Mr. PIRIE. That would simplify our problem tremendously. At the moment we take our own BRAC money and we clean it up, and then we attempt to persuade Interior, for example, that we have cleaned it up to standards that meet their approval.

    Mr. HEFNER. The fact remains somebody has got to clean it up before you can do anything with it.

    Mr. PIRIE. Yes, sir.

    Mr. HEFNER. Because you have closed the base, it has got to be cleaned up before you can use it for a shopping mall or anything else. Somebody is going to have to pay for that clean up.
 Page 506       PREV PAGE       TOP OF DOC    Segment 2 Of 3  

    If it makes sense to transfer it to the Interior Department or the Forest Service, or whoever, it seems to me that there could be a simple formula that could be worked out that says we are going to transfer this 5,000 acres to the Interior Department. It will be a simple transfer, but you have to take on the responsibility and be obligated to clean it up in the same amount of time that it would take you to clean it up.

    This does not make a lot of sense to me.

    Mr. PIRIE. It is extremely complicated, and the formula that you propose would be terrific, as long as we could come to an agreement about what the clean up consisted of.

    For example, Naval Air Station Adak in Alaska. We are in the process of completing the clean up there as far as we are required to clean it up, which includes petroleum in the ground and things of that kind.

    But there is a discussion with the local redevelopment authority for Adak about whether we are obligated to knock down a bunch of buildings that they do not want to use in the future.

    Mr. HEFNER. Who is this property going to?

    Mr. PIRIE. The local redevelopment authority wants to convey the property to the Aleut Corporation that represents the interests of a substantial number of Aleut Indians. It is a rather complicated transfer because Adak is essentially a National Refuge, and it should revert to the Interior Department.
 Page 507       PREV PAGE       TOP OF DOC    Segment 2 Of 3  

    The Interior Department does not want it.

    Mr. HEFNER. Well, they don't want it, but I am saying when it is in everybody's interest and the—Interior Department wants it, and we want to get rid of it. It seems to me that this is an isolated instance that you are talking about.

    Mr. PIRIE. It is. It is a very unusual and vexatious case.

    Mr. HEFNER. It seems to me this is human nature. If you want to transfer it to the private sector, naturally they want it all cleaned up. They also want all the buildings out of it, where they can come in and survey it and put a mall there.

    But there ought to be some simple way you could say, okay, we are going to transfer this, but you know what the rules are. Here is how it has to be cleaned up if you are going to put a mall or whatever there, under the EPA guidelines, and let the folks that acquire it clean it up, and work out the price where they would be responsible for it.

    Because you no longer need it, and this is something that actually is not your field, is it?

    Mr. PIRIE. Oh, yes. We do the clean ups of existing bases.

    Mr. HEFNER. I know you can. But that is not what you are set up to do.
 Page 508       PREV PAGE       TOP OF DOC    Segment 2 Of 3  

    Mr. PIRIE. That is right. I mean, for the people who are going to come in and develop it they ought to have the liberty to develop it as suits their needs. Generally in 90 percent of the cases we are able to come to an accommodation.

    For example, if it is going to be—if it is an air station that is going to be an airport, we clean it up to the standards that are appropriate to an airport, not to national park standards.

    Generally that works. Occasionally, as in the case of Adak that I cited, it is complicated and it has more features than we really want.

    Mr. HEFNER. This may sound simplistic, but if you clean up an area, and then you sell it to a corporation or whoever. When they begin to develop it, before they can get loans and cash to do the development, they are going to come in and demand another check. They are going to do drillings and additional testing and it may require more clean up than you have already done.

    It would just seem to me that if you are going to do that, people that are going to eventually use it in the end, would want the responsibility of cleaning it up where it would be sufficient to use once you have conveyed it to them.

    Mr. PIRIE. Generally we start the process with an environmental baseline survey. And the people who are going to be the redevelopment authority are in on that, and also the local regulators. So we try to establish what is there to a fair degree of accuracy to really start the process.
 Page 509       PREV PAGE       TOP OF DOC    Segment 2 Of 3  

    Clearly if something that nobody knew about over the course of time develops, then we are responsible to come back in and clean that up. There is no way to get out of that under current law.

    Mr. HEFNER. I have no further questions.

    Mr. PACKARD. Mr. Olver.

FORT DEVENS, MASSACHUSETTS

    Mr. OLVER. Thank you, Mr. Chairman.

    These exchanges are giving me time to find other things to discuss here. I notice in the 1999 Defense program that there are a couple of projects in my State, Massachusetts, Army Reserve Fort Devens, $3.3 million, and then Navy Reserve, it lists Lawrence, $840,000.

    Well, I now discover that the Navy Reserve Lawrence is really Naval Reserve for a Marine unit that is presently at Lawrence, but to move it to Devens. Now, that makes me begin to suspect that there is a master plan someplace of consolidation or rearrangement of Reserve units to the Reserve base at Devens, which was the subject of BRAC 1991, I guess it was.

    Where do I find a document which shows which pieces? Do I have to squirrel these together from Navy documents and Army documents? Or is there something that says what Devens is doing, and what is the phasing for what is supposed to happen there?
 Page 510       PREV PAGE       TOP OF DOC    Segment 2 Of 3  

    Mr. PIRIE. I think we can provide you with what the Navy's plans and what the Marine Corps plans are, and so forth. But what is coming into Devens, I think that is largely a matter of the Army's responsibility.

    [The information follows:]

    What consolidation or rearrangement of Reserve units is occurring at the Reserve base at Fort Devens, MA that resulted in the relocation of the Ordnance Maintenance Control Team from Lawrence, MA discussed in the budget justification for your FY 1999 Military Construction, Naval Reserve Project p–477?

    The Naval Reserve has no short-term nor future plans to relocate or consolidate drilling Naval Reserve Units to Fort Devens, MA.

    The Marine Corps Reserve will relocate the Ordnance Maintenance Contact Team. It is presently located in the former Naval Marine Corps Reserve Center in Lawrence, MA and is planned to move to Fort Devens, MA upon completion of project P–477.

    In addition, Weapons, Headquarters and Service Company, First Battalion, Twenty Fifth Marine Regiment will relocate from Camp Edwards, MA to Fort Devens upon completion of project P–508 at Fort Devens. This project was funded by Congress in FY 1996 for construction originally at Camp Edwards. Environmental problems delayed project execution at that site, so funding was subsequently reprogrammed for execution at Fort Devens, MA.

 Page 511       PREV PAGE       TOP OF DOC    Segment 2 Of 3  
    Mr. OLVER. But if this is going into Devens, this is at sufferance from the Army for you? They are suffering you at Devens? What is this?

    Mr. PIRIE. Well, it is a closing Army base. And they are making——

    Mr. OLVER. There was an active Army——

    Mr. PIRIE. It was an active Army installation.

    Mr. OLVER. But this is a unit which is a Reserve Naval unit, a Reserve Marine unit.

    General HAYES. This is a Marine maintenance unit, sir. I cannot speak to it in terms of movement though.

    Mr. OLIVER. In terms of what?

    General HAYES. In terms of movement relative to BRAC process. But that project is specifically, the $840,000, is to rehabilitate a building, as I understand it, for a maintenance unit for the service support group.

    Mr. OLVER. Well, fine, but if it is going to be at Devens, I assume it will be by way of some agreement with the Army. Would that be correct?

 Page 512       PREV PAGE       TOP OF DOC    Segment 2 Of 3  
    General HAYES. Yes, sir.

    Mr. OLVER. So should not there somewhere be a document where I could see what is happening at Fort Devens since we are now putting units from various forces together? Is this the end of it? Is this going to be the last of these that happens bringing Naval Reserve pieces to that particular site?

    How would I know that?

    Mr. PIRIE. I do not really know quite how to answer your question. I do not think we have any plans to move other Navy or Marine Corps units to Fort Devens.

    The Army may have a consolidated list of activities that they expect to move in there over the course of the next few years, which might include other Army activities or other Air Force activities.

    Mr. OLVER. I suspect because that is a ground training base that it probably would not have—well maybe it would have Air Force activities, you might be right. So I should ask that of the Army because they are in control of the base and if there is anything that is in anywhere in the books of Naval plans that go beyond this, they would know about that?

    Mr. PIRIE. Oh, definitely. Because one of the requirements of the base closure process is that application has to be made to the closing activity for space in the activity for other DOD stuff.
 Page 513       PREV PAGE       TOP OF DOC    Segment 2 Of 3  

FUTURE YEAR DEFENSE PLAN

    Mr. OLVER. Okay. For the 1999 FYDP, the items that are here in the construction plan, are those all design?

    Mr. PIRIE. All items?

    Mr. OLVER. Yes.

    Mr. PIRIE. 1999?

    Admiral NASH. They are under design.

    Mr. OLVER. They are all under design? All of them would be under design?

    Admiral NASH. Yes, sir.

    Mr. OLVER. Would they be being designed with 1998 monies?

    Admiral NASH. It could be.

    Mr. OLVER. I guess what I am trying to explore is whether—let me get out what I am really after. I am curious whether you have enough money to design the projects in the future, and I am sort of curious about what the phasing is of design for projects which are in the 1999 FYDP, in the plan, and if you have FYDP 2000 items, is there design money in this year's—in the 1999 budget—to cover the design for the year 2000 items?
 Page 514       PREV PAGE       TOP OF DOC    Segment 2 Of 3  

    Admiral NASH. You give us money the year ahead for design.

    Mr. OLVER. Are we always adequate? Do we always give you enough to do the design for the next years?

    Admiral NASH. We have in the past been able to manage to get designs done. We also have new ways of doing things, where we can get design and construction done at the same time, which speeds up the process.

    So as far as I know we have had adequate design funds and we continue to get them—because that is part of what we are asking for.

    Mr. OLVER. So you have new ways of getting design and construction done in the same appropriation I guess is what you mean there.

    Admiral NASH. Yes, sir.

    Mr. OLVER. Are any of these 1999 planned programs ones that have the design and the construction money included and expected to be in one year?

    Admiral NASH. I will take that for the record. We will give you a list.

    Mr. OLVER. Yes.
 Page 515       PREV PAGE       TOP OF DOC    Segment 2 Of 3  

    [The information follows:]

    The planning and design funding being requested in the FY 1999 budget will be used to design projects currently in the FYs 2000 and 2001 programs.

    Mr. OLVER. I guess I would like to be able to map out for my own mind how these things move, how the design for items are done along the way. And we do add-ons in many of these instances.

    Admiral NASH. Yes, sir.

    Mr. OLVER. And if we add on, then you will not have had design to do the add-ons in an orderly manner, would you?

    Admiral NASH. We manage that together.

    Mr. OLVER. You manage those together.

    Admiral NASH. We manage those together.

    Mr. OLVER. Do the designs and the add-ons and the construction together in a year where we give you that money in the one year and it can be done in that year, or least into the process of whatever the reserve of monies into the next fiscal year applies?

 Page 516       PREV PAGE       TOP OF DOC    Segment 2 Of 3  
    Admiral NASH. And there are formulas for how to get enough design money the year ahead, and if there is additional design money required we manage that.

    Mr. OLVER. I was afraid it was going to be something like——

    Mr. PACKARD. Would the gentleman yield?

    Mr. OLVER. Yes, by all means.

    Mr. PACKARD. My observation is that, number one, there is flexibility enough in the system where they can design and have access to design money perhaps in the early years of the FYDP, first and second year perhaps.

    Admiral NASH. Yes, sir.

    Mr. PACKARD. I would assume that it would be quite unusual to have design work going on in the fourth and fifth year of a FYDP, simply because that may never come about. It may never actually be built out.

    Back to the other question of add-ons. We select very carefully add-ons that are executable, and that usually means that they can be designed, and contracts can be let within the year.

    Every one of the projects of the 38 that were line item vetoed last year, the reason given that they were lined out because most of them were not executable was not true. They were executable. We selected them on that basis.
 Page 517       PREV PAGE       TOP OF DOC    Segment 2 Of 3  

    And that means that they can do the design work, and can do the contract work within the year that they are approved for funding. So I think we have been careful on the committee not to approve add-ons that are not executable.

    Mr. OLVER. Well, would we be doing design for, say, 2001 or 2002 projects out of monies that are in this year's 1999 design allowance? Is that likely?

    Admiral NASH. We do some levels of design farther out, and as we get closer we do more designs. We do not do a whole design on a project that might not occur.

    So a few years out we are looking at what we have—project cost estimates, and trying to get some kind of idea, and then as we get closer to the year we are doing more design. So it's a process that we manage, and that's what I meant in terms of we manage the process. And it has to be ready to execute.

    Mr. PIRIE. Some of these projects require a good deal less design than others. Like dredging projects. They are big, expensive MILCON projects, but they do not require much design.

    The replication of a missile bunker next to one that you built last year does not require a lot of design, because it is the same building that you built last year. Some of these things take a lot. Some do not.

    Mr. PACKARD. Would the gentleman yield one more time?
 Page 518       PREV PAGE       TOP OF DOC    Segment 2 Of 3  

    Mr. OLVER. Sure.

LOST DESIGN

    Mr. PACKARD. I understand that we do have shelves of design work that has been done over the years on projects that have never been built and probably never will be built. That is a waste of money—millions and millions of dollars worth.

    We would hope that we could avoid that as much as possible because politics change, priorities change, a variety of things change. A project that you pushed through to have design work done, and they start the design work four years early, and then you are not here to see it through, and it no longer has the political support, and it may never be built as a result.

    So those are things that I think they are trying to avoid, and we want to avoid as much as possible. So the design work generally today, we would hope, means we are moving forward to construction.

    Mr. OLVER. It would seem to me to make better sense, as you put it, that those that are executable within the design and construction for the same project, for the very reason that you describe—well, I am just trying to learn how this process functions. Thank you.

    Mr. PACKARD. Thank you.
 Page 519       PREV PAGE       TOP OF DOC    Segment 2 Of 3  

    Mr. Hoyer, would you like another round?

    Mr. HOYER. If I could, Mr. Chairman.

    Mr. PACKARD. Of course.

    Mr. HOYER. Thank you, sir. I want to observe that I have spent a lot of time, effort and money trying to avoid not being here to follow through on my own projects. [Laughter.]

PRIVATIZATION

    Mr. HOYER. A capital investment that I think is very important.

    You mentioned in your statement, obviously in terms of savings, in house and outsourcing or contracting out. On page 4 of your statement you make a comment that I thought was interesting.

    And I would like to get your view on it, in which you indicate that there is a 30 percent savings whether it is done in or out, if in fact if you do it in you do it smart, that is to say, reinvention or new ways. You say new most efficient organization.


Next Hearing Segment(3)